Родительские собрания, сценарии к дню именинника, материалы к математическому бою, неделе математики, к кружку, к урокам математики
план-конспект занятия по теме

Соболева Галина Петровна

Сценарии к дню именинника, материалы для родительских собраний для 5-6 классов. Материалы к кружку по математике для 5-6 классов. Математический бой для 8-11 классов. Для недели математики подходит материал Школьная математическая регата 7-9 классы и КВН для 5-6 классов, софизмы, презентация о Лобачевском. Методические разработки могут стать элементами уроков

Скачать:

ВложениеРазмер
Файл День зимнего именинника. Сценарий праздника для ребят 5 класса25.56 КБ
Файл День осеннего именинника. Сценарий праздника для ребят 5 класса22.08 КБ
Файл День весеннего именинника. Сценарий праздника для ребят 5 класса25.82 КБ
Файл Организация учебной домашней деятельности учащихся. Материалы для родительского собрания, 5 класс35.05 КБ
Файл Организация учебной домашней деятельности. Материалы для родительского собрания, 5 класс78.57 КБ
Файл Дети и родители. Материалы для родительского собрания, 5 класс22.02 КБ
Файл Дети и родители. Материалы для родительского собрания, 5 класс163.42 КБ
Файл Здоровые дети в здоровой семье. Материалы для родительского собрания, 5 класс20.87 КБ
Office presentation icon Здоровые дети в здоровой семье. Материалы для родительского собрания, 5 класс777.74 КБ
Microsoft Office document icon Бранные слова в речи ребёнка. Материалы для родительского собрания, 6 класс45.5 КБ
Файл Математический бой 8-11 классы. Правила1.33 МБ
PDF icon Математический бой 8-11 классы.Задачи1.34 МБ
Файл Материалы к кружку по математике 5 класс19.65 КБ
PDF icon Задачи с числами. Материалы к кружку по математике 5 класс.233.83 КБ
Файл Задачи, решаемые с конца. Материалы к кружку по математике 5 класс19.33 КБ
Microsoft Office document icon Запись цифр и чисел у народов мира. Материалы к кружку по математике 5 класс947 КБ
Файл Круги Эйлера. Материалы к кружку по математике 5 класс91.02 КБ
Файл Магические квадраты. Материалы к кружку по математике 5 класс313.24 КБ
Файл Математические ребусы. Материалы к кружку по математике 5 класс18 КБ
Файл Математические фокусы. Материалы к кружку по математике 5 класс18.61 КБ
Файл Приёмы быстрого счёта. Материалы к кружку по математике 5 класс26.72 КБ
Файл Софизмы. Материалы к кружку по математике 5 класс17.29 КБ
Microsoft Office document icon Старинные задачи. Материалы к кружку по математике 5 класс50 КБ
Файл Числа великаны. Материалы к кружку по математике 5 класс34.45 КБ
Файл Школьная математическая регата 7-9 классы. Материалы к неделе математики. 228.8 КБ
Файл Материалы к неделе математики. Сценка "Деление"14.98 КБ
Файл О софизмах. Материалы к неделе математики. 106.96 КБ
Файл О Лобачевском. Материалы к неделе математики.461.77 КБ
Файл Блиц-турнир для зрителей. Школьной математической регаты. Неделя математики. 16.28 КБ
Файл Конкурс болельщиков. К школьной математической регате. Материалы к неделе математики. 683.83 КБ
Файл Приветствие команды ПУПС. Математический КВН 5 класс.Материалы к неделе математики. 11.37 КБ
Файл Свойства параллелограмма. Тест. Геометрия 8 класс. 11.8 КБ
Файл Свойства параллелограмма. Таблица свойств. Геометрия 8 класс. 12.11 КБ
PDF icon Итоговое повторение в таблицах.Геометрия 7 класс. 1.38 МБ
Файл К уроку геометрии 8 класс13.01 КБ
PDF icon Тест по начальным геометрическим сведениям. Геометрия 7 класс. 1.46 МБ
Файл Час весёлых и находчивых. 5-6 классы. Неделя математики. 27.29 КБ
PDF icon Тест по теме "Натуральные числа и шкалы". Математика 5 класс2 МБ
Файл Математика 6 класс. Презентация к теме "Моделирование"74.81 КБ
Файл "Промежутки".Проверочная работа. Математика 6 класс. 30.15 КБ
Файл "Вычитание рациональных чисел". Проверочная работа. Математика 6 класс. 22.21 КБ
Файл "Формулы сокращённого умножения". Проверочная работа. Алгебра 7 класс. 12.6 КБ
Файл "Формулы сокращённого умножения". Самостоятельная работа. Алгебра 7 класс. 164.25 КБ
Файл Статистика. Алгебра 9 класс. 48.67 КБ
Файл Теория вероятностей. Алгебра 9 класс.89.67 КБ
Файл Экспериментальные данные. Алгебра 9 класс. 52.07 КБ
Файл "Масленица". Сценарий праздника для 1 и 5 классов27.27 КБ

Предварительный просмотр:

Сценарий

«День Зимних Именинников»

Цели:

Поздравить зимних именинников с днем рождения;

Создание в коллективе благоприятного психологического климата;

Воспитание творческих способностей, умений и навыков коллективной творческой деятельности.

Ход мероприятия:

Под музыку П.И.Чайковского «Декабрь»

Ведущий 1: Добрый день, дорогие друзья!  Мы сегодня собрались здесь, чтобы поздравить наших милых именинников, родившихся зимой, которые стали старше  на целый год.

Ведущий 2: И снова сказка начинается

                    Когда приходит к нам зима.

                    Всё белым снегом одевается,

                    И в шапках снеговых дома.

                   Зима лишь только начинается…

Ведущий 1: Декабрь – первый месяц зимы. В народе его называют студень. Он землю студит, реки, и озёра льдом мостит, поля серебристым покрывалом укрывает.

Ведущий 2: Коньки и санки так нам нравятся,

                 И лыж стремительный полёт

                 И, наконец, с лесной красавицей

                Приходит праздник – Новый год!

Ведущий 1: Тех, кто родился в декабре,

                    Поздравим нынче с днём рожденья!

Приглашаем занять почетные места наших именинников декабря Сорокина Валера, Углова Ксюша, Шалкин Данила, Соболева Аня

(Музыка становится громче.  Выходят именинники декабря)

Приветствуем их аплодисментами

Под музыку П.И.Чайковского «Январь»

Ведущий 2: Месяц зимний,

                   Месяц снежный

                  Открывает первым год,

                  Вновь со снегом и морозом

                  В гости к нам январь идёт.

                 Он на реках и озёрах –

                 Всюду выстроил мосты

                 И одел пушистым снегом

                Все деревья и кусты.

Ведущий 1: Январь – в народе просинец – морозные узоры на окнах рисует. А просинцем называют его потому, что красит он лёд в просинь – синим цветом.

Ведущий 2: Кто родился в январе,

                  Все сюда идите,

                 Поскорее всем ребятам

                 Себя вы покажите!

 (Громче становится  музыка, выходят именинники  января)

Ведущий 1: Приветствуем Исхакова Серафима  и Авдеенкова Никиту аплодисментами.

Под музыку П.И.Чайковского «Февраль»

Ведущий 1: Февраль гуляет во дворе,

                   И щиплет уши детворе,

                   И красит щёки докрасна.

                   Уйдёт февраль – придёт весна!

Ведущий 2: Февраль в народе называют снежень: он приходит с метелями да вьюгами. Это последний месяц зимы.

Ведущий 1: В день четвёртый февраля

                    Родилась девчонка!

                    Подросла и расцвела,

                    Не узнать ребёнка.

 Кто это?  

        И мы приглашаем февральскую именинницу   Колосову Женю.

(Под музыку выходит Колосова Женя)

Ведущий 2:

Праздник мы сейчас откроем

Игры, пляски здесь устроим.

Повернитесь все друг к другу

И пожмите руки другу.

Руки вверх все поднимите

И вверху пошевелите

Именинникам УРА!

Поздравлять вас всех пора!

Ведущий 1: Поздравляем вас, именинники, с днём рождения, который вы ждали много дней, целый год. Посмотрим на эти красивые, умные, добрые лица. Хотим пожелать вам всего, всего хорошего, конечно, здоровья, исполнения ваших желаний и прекрасного настроения. Пусть этот час подарит вам радость общения и яркие впечатления.

Ведущий 2:

Чтоб всегда под счастливой звездою

Вас судьба по дороге вела.

В доме чтоб полноводной рекою

Жизнь спокойно и мирно текла.

Пусть Ваш дом лишь друзья посещают,

Стороною обходят ненастья,

От души мы добра Вам желаем,

Долгой жизни, здоровья и счастья!

И давайте все вместе скажем им «поздравляем».

Все ребята кричат три раза «Поздравляем»! Выносятся три торта со свечками. Звучит музыка Светиковой С. «День рождения»

Ведущий 2: На день рождение принято дарить подарки. Вот и мы сегодня приготовили подарки для наших зимних именинников. Девочки Степанова Ксюша, Глотова Марина, Вудрицкая Саша и Беляева Настя исполнят для вас песню О друзьях.

Поют девочки.

Ведущий 1. Игошина Даша и Голубева Таня споют песню «С Днём рожденья»

Поют девочки

Затем ведущий проводит конкурсы.

Спой песню

А теперь именинники порадуют нас своими ангельскими голосками. Они нам споют по одному куплету  песни «В траве сидел кузнечик», но с заменой всех гласных на одну гласную, а какую, ребята вытянут жребий. Одни поют,  заменяя на «А» первый куплет, другие – на «У» второй куплет, третьи на «О» третий куплет.

«Загадки для именинников».  

Кто из именинников  отгадает больше загадок, тому приз

Растёт она вниз головой

Не летом растёт, а зимой

Чуть солнце её припечёт

Заплачет она и умрёт. (Сосулька)

Белое покрывало на земле лежало,

Лето пришло, оно всё сошло. (Снег).

Гуляет в поле, а не конь.

Летает на воле, да не птица. (Вьюга).

И не снег, и не лёд

А серебром деревья уберёт. (Иней).

«Жил я посреди двора,

Где играет детвора, но от солнечных лучей

Превратился я в ручей». (Снеговик).

Он вошёл – никто не видел,

Он сказал – никто не слышал.

Дунул в окна и исчез,

А на окнах вырос лес. (Мороз).

Рассыпался горох на семьдесят дорог:

Никто его не собирает. (Град).

Рыбам зиму жить тепло:

Крыша: толстое стекло. (Лёд.)

Запорошила дорожки,

Разукрасила окошки,

Радость детям подарила

И на санках прокатила. (Зима).

Красавица какая!

Стоит, светло сверкая,

Как пышно убрана…

Скажите, кто она? (Ёлка новогодняя).

Есть, ребята, у меня

Два серебряных коня.

Езжу сразу на обоих.

Что за кони у меня? (Коньки)

Поздравление – загадка

Именинников приглашаем сесть на стулья спиной к зрителям. Дети по очереди подходят со спины и поздравляют своими словами именинников, а именинники должны отгадать, кто их поздравляет.

Конкурс “Прищепки”

Наши дети очень шустрые, быстрые, ловкие и я предлагаю ещё один конкурс. Участвуют по две пары от именинников и одноклассников.

Выходят 2 пары. Одному участнику из пары завязывают глаза, а на другого хаотично крепят прищепки. По команде ведущего участник с завязанными глазами должен снять все прищепки со своего напарника. Побеждает та пара,  у которой не останется ни одной прищепки.

Игра “Сюрприз”

Все дети встают  в круг. По команде, когда заиграет музыка, свёрток передают по кругу, когда затихает музыка, у кого в руках оказывается свёрток, должен его развернуть, как только музыка заиграет, свёрток опять передаётся по кругу. У последнего окажется приз.

Перед чаепитием ведущий читает стих, дети подбирают рифму.

Я захотел устроить бал,

И я гостей к себе… (позвал).

Купил муку, купил творог,

Испёк рассыпчатый… (пирог).

Пирог, ножи и вилки тут,

Но что-то гости не… (идут).

Я ждал, пока хватило сил,

Потом кусочек… (откусил).

Потом подвинул стул и сел

И весь пирог в минуту… (съел).

Когда же гости подошли,

То даже крошек не… (нашли).

Приглашаем всех на чаепитие.



Предварительный просмотр:

День  осеннего именинника

5 класс.

Цель: сплотить коллектив, воспитание доброго и внимательного отношения

друг к другу, развитие творческих способностей учащихся.

Оформление клуба: воздушные шары, поздравительные рисунки

компьютер.

Ведущий 1- Игошина Дарья; Ведущий 2 – Потанин Кирилл; Ведущий – Классный руководитель

Ведущий 1: Добрый день! Сегодня мы собрались вместе, чтобы поздравить

наших именинников, которые родились летом или осенью : Глотову Марину, Трухина Родиона, Степанову  Ксюшу, Голубеву Таню, Звонова Влада, Вудрицкую Сашу, Гурина Колю, Луконина Диму, Волнову Настю,  Затравина Ваню.  День рождения – один из самых главных праздников жизни каждого человека, ведь в этот день появляется новый человек.

Ведущий 2: -Что такое день рождение?

Я отвечу без сомненья:

День подарков, пирогов,

День улыбок и цветов!

И у нас для вас, именинники, – есть небольшой сюрприз.

Все вместе поют:

С Днём рождения вас,

С Днём рождения вас,

С днём рожденья, одноклассники,

С днём рождения вас!

В это время выносят торт со свечами (10 штук по количеству именинников). Именинники все вместе задувают свечи.

 Ведущий  1: Желаем вам

От солнца – тепла,

От людей – добра,

От мамы с папой нежности,

От друзей – любви и верности.

Ведущий 2: Все сегодня должны быть веселыми. Давайте запустим ракету веселья – ногами топаем и руками хлопаем…

Ведущий 1:  Итак, начинаем веселье. Поздравительный номер – выступление ребят нашего класса. Песня «Из чего же…»

Ведущий: Класс делится на две команды: именинников и одноклассников.

Наш первый конкурс называется «Разминка».

1. Конкурс «Загадки про слова»

- Бусы женщин украшают, все об этом знают.

Чтоб мужчинам их носить, нужно что-то отцепить.

Что? Скорее говори! Я считаю: раз, два три…(отцепить букву Б)

- Если шарф укоротить, чем тогда он может быть? (шаром)

- Львиную гриву щеткой чесали, на пол из гривы две буквы упали.

Кто догадается, кто же поймет, что вместо гривы на леве растет? (ива)

- Есть такая шапка, дети. Больше всех она на свете.

Папа наш такой большой, тонет в шапке с головой!

Кто же быстро догадается, как та шапка называется? (папаха)

- Что добавить нужно в сок, чтобы сыпаться он мог? (песок)

- С буквой Г я по небу лечу, с буквой В – детишек лечу. (грач – врач)

- С буквой Б – я емкость для солений, с буквой Т – в конце я предложений. (бочка-точка)

Ведущий 2:  А сейчас Сценка "Наши падежи"

 

Действующие лица: учитель и ученик Петров

 

Учитель:  Петров, выйди к доске и запиши небольшой рассказ, который я тебе продиктую.

Ученик выходит к доске и готовится писать.

Учитель(диктует): “Папа и мама ругали Вову за плохое поведение. Вова виновато молчал, а потом дал обещание исправиться.”

Ученик пишет под диктовку на доске.

Учитель: Прекрасно! Подчеркни в своём рассказе все имена  существительные.

Ученик подчёркивает слова: «папа», «мама», «Вову», «поведение», «Вова», «обещание».

Учитель: Готово? Определи, в каких падежах стоят эти существительные. Понял?

Ученик: Да!

Учитель: Начинай!

Ученик: “Папа и мама”. Кто? Что? Родители. Значит, падеж родительный.

 Ругали кого, чего? Вову. “Вова” - это имя. Значит, падеж  именительный.

 Ругали за что? За плохое поведение. Видно, что-то натворил. Значит, у “поведения” падеж творительный.

 Вова молчал виновато. Значит, здесь у “Вовы” падеж винительный.

 Ну, а “обещание”, конечно, в дательном падеже, раз Вова его дал !

 Вот и всё!

Учитель: Да-а, разбор получился оригинальный! Неси-ка дневник, Петров. Интересно, какую отметку ты предложил бы себе поставить?

Ученик:  Какую? Конечно, пятёрку!

Учитель: Значит, пятёрку? Кстати, в каком падеже ты назвал это слово - “пятёрку”?

Ученик:  В предложном!

Учитель: В предложном? Почему же?

Ученик:  Ну, я же её сам предложил!

 

Ведущий: Продолжаем дальше конкурс:

Вопросы:

1. Что находится посередине земли? ( буква М)

2. Какой месяц короче всех? ( май)

3. Что можно приготовить, но нельзя съесть? (уроки)

4. Чем кончается день и ночь? (мягким знаком)

5. Что станет с зеленым платком, если он упадет в Красное море? (он станет мокрым).

6. Сколько месяцев в году имеют 28 дней? (все месяцы)

7. Что стоит между окном и дверью? (буква «И»)

8. Может ли страус назвать себя птицей? (нет, т.к. он не умеет говорить).

Ведущий: Наш третий конкурс.

 Из подручных средств нарядить  именинников. (5 минут)

Ведущий: фото на память – запомним их такими.

Ведущий 2: И снова нас радуют девочки: Соболева Аня и Колосова Женя с песней «Ты да я да мы с тобой»

Ведущий1:  Гороскоп – поздравление.

Без гороскопа никак нам не обойтись! Только гороскоп будет праздничный, необычный. Что же ждет наших новорожденных в новом году.

( Бумажные карточки с предсказаниями и бумажные карточки с названиями предметов).

Все помещается в сумочку и именинники, вытаскивая карточку с названием предмета узнает о своем гороскопе. Все зачитывается громко для всех.

Ведущий 2: Фейерверк - ваша жизнь заискрится всеми цветами

Хлопушка в вашей жизни произойдет какое-то оглушительное событие

Письмо вас ожидают приятные известия

Цветок вы станете еще милее и красивее

Гриб вас ожидают приятные и полезные прогулки по лесу

Часы - в вашей жизни произойдут радостные перемены

Очки рекомендуется меньше времени проводить у телевизора, а больше – с друзьями на

свежем воздухе

Конфета у вас начнется новая сладкая жизнь

Книга вы будете стремиться к знаниям и преуспеете в учебе

Стрела вы будете очень настойчивы в достижении цели

Подарок жизнь приготовит для вас приятный сюрприз

Солнце этот год будет для вас удачным

Ведущий: конкурс «Змейка»

Все присутствующие образуют цепочку. Первый – «голова», последний

- «хвост». Включается музыка, и гусеница начинает двигаться вперед. При этом голова может двигаться как хочет, - может махать руками, делать резкие выпады, идти гусиным шагом, что угодно. Остальные должны повторять. Когда «голова» устанет, она поворачивается к следующему игроку  и переходит в хвост, после чего все начинается с новой «головой» и новыми приколами.

Ведущий: ну и какой же день рождения без угощений?

Наш праздник продолжается чаепитием.



Предварительный просмотр:

День Весеннего именинника

5 класс.

Цель: сплотить коллектив, воспитание доброго и внимательного отношения

друг к другу, развитие творческих способностей учащихся.

Ведущий 1:

Отступила зима, прилетели грачи,

Распустились зелёные листья.

Много красок вокруг: просто это весна

Всё раскрасила солнечной кистью.

И сегодня, в прекрасный весенний денёк,

Мы ребят своих вновь поздравляем,

И сегодня опять дружный наш коллектив

Им успехов и счастья желает.

Ведущий 2. Приглашаются именинники: Потанин Кирилл, Разинов Саша, Бородинов Толя, Степанов Женя, Гришин Кирилл, Беляева Настя, Игошина Даша, Савельев Ваня

            Наблюдая с просторов Вселенной,

Как на землю приходит весна,

Я решил вам цветок необычный

На ваш праздник в подарок прислать.

На созвездии Дней Рождений

Распустился он в солнечный день.

Не похож он на розы и маки

И совсем не похож на сирень.

Я надеюсь, его вы узнали?

На ромашку земную похож,

Пожеланье себе ты узнаешь,

           Только лишь лепесток оторвёшь

Пожелания на лепестках ромашки:

  1. Тебе желаем доброты и понимания,
    Друзья пусть любят и в делах везёт,
    Исполнятся заветные желания.

    2) Тебе желаем счастья и удачи,
    Больших успехов, преданных друзей.
    Пусть будет жизнь на радости богаче,
    Полна хороших встреч и ярких дней!

    3) Мы желаем лететь стремительно вперёд,
    Не опасаясь поворотов, тебя всегда удача ждёт!

    4)  Пусть не будет на сердце печали,
    Пусть всегда окружают друзья.
    Мира, радости, счастья желаем,
    И успехов во всём и всегда.

    5) Пусть этот день, как песня соловья,
    Перечеркнёт всех хмурых дней ненастье.
    Пусть жизнь твоя, как ранняя весна,
    Приносит каждый день в ладонях счастье!

    6) От души желаем радости, везенья,
    Много добрых и счастливых дней,
    Чтоб отличным было настроенье,
    Жизнь - полна улыбками друзей.

7) Желаем успехов, удачи,
Легко отдыхать, и трудится с отдачей,
Чтоб радость в судьбе твоей чаще встречалась,
Чтоб всё выходило и всё получалось!

                                               

8) Желаем, чтобы люди ценили тебя,

Друзья, - чтоб тобою гордились,

Чтоб жизнь молодая цвела и цвела,

И годы, - чтоб счастливо длились.                                                

                                                                       

Ведущий 1: При рождении  новорожденный получает имя. Давайте спросим наших именинников, что они знают о своих именах. Что означают ваши имена? Почему вам дали именно такое имя? Давайте вместе узнаем и запомним, что означают ваши имена.
  
  Кирилл  — По-гречески правитель.

Анатолий -  По-гречески  восход солнца

Александр — По-гречески мужественный защитник

Евгений – по-гречески благородный

Иван – по - древне еврейскому Милость божья

Анастасия -  по-гречески воскресшая

Дарья – по древне персидскому  сильная, побеждающая

Ведущий 2. Теперь для вас, именинники, песня в исполнении группы ребят. (Песня на мотив Г. Гладкова “Чунга-чанга”.)
День рожденья с детства любишь ты.
В день рожденья – дарятся цветы.
В день рожденья – водят “Каравай”.
Вместе с нами дружно запевай.
 
Припев: 
Веселятся дружно дети. 
Праздник любят все на свете.
Только раз в году бывает день рожденья!
Поздравленья и подарки,
И наряды очень ярки –
С нетерпеньем ждем всегда мы День рожденья!


В день рожденья – танцы, песни, смех.
В день рожденья ждет тебя успех.
В день рожденья – сбудутся мечты.
В день рожденья так прекрасен ты!
 
Припев: тот же.

Ведущий 1. О человеке можно рассказать многое, зная только дату его рождения.

Ведущий 2. Ой, это не новость... Я думаю, что даже ребята знают, что такое знаки зодиака!

Ведущий 1. Ребята, а вы знаете, какие знаки зодиака приходятся на весенние месяцы?

Реакция зала.

Ведущий 2. Чтобы узнать поближе своих одноклассников и друзей, мы вам немного расскажем про их темперамент и характер.

Ведущий 1. Первый наш знак - это Рыбы (с 21 февраля по 20 марта). Этот знак покровительствует Потанину Кириллу, Бородинову Толе, Разинову Саше. Рыб нелегко понять или описать. Сами они видят насквозь людей, с которыми общаются, бывают хранителями их тайн. Многое знают и многое требуют, производят впечатление мечтательных идеалистов, легкоранимы, порой пугливы и озабочены.

Для развития своей личности стремятся к одиночеству. Многие Рыбы - мистики с глубоким пониманием жизни, в них есть что-то от восточных мудрецов.

Они больше думают, чем действуют. Подвержены частой смене настроения.

Любят Рыб за мудрость и доброту. Для Рыб важно преодолеть свою робость и сомнения, научиться верить в себя.

 Рыбы - знак Воды. Предпочитают уклоняться, обходить препятствия, нежели бороться с ними, но главное умение - проникать, продвигаться, несмотря ни на что, и затоплять, и побеждать.

Ведущий 2 .Овен, рожденный с 21 марта по 20 апреля. Гороскоп для Гришина Кирилла, Степанова Жени, Беляевой Насти, Игошиной Даши

Овен - впечатлительная личность с сильной волей. Часто бывает индивидуалистом, его трудно не заметить. Быстро схватывает суть явлений. На него можно положиться, он дает искренние, дружеские советы.

Верит, что жизнь в его руках, нетерпелив, не верит в судьбу. Для него вся жизнь в действии. Что на уме, то и на языке - он не умеет лгать.

В его натуре есть что-то детски непосредственное, что сразу бросается в глаза. Его либо любят, либо ненавидят. Он плохо разбирается в людях, совершенно не замечает нюансов человеческих характеров. Отсюда частые разочарования при общении с людьми.

Совет Овнам: думайте, прежде чем говорить. Далеко не каждый может ладить с Овном - больше терпения и меньше высокомерия.

Овен - самый огненный знак, неугасимый, неутомимый. Горячий, вспыльчивый, живой ум, сообразительный. Хорошо умеет принимать решения, динамичен, разговорчив, бодр и оптимистичен, но нетерпелив и самоуверен.

Ведущий 1.  Телец (с 21 апреля по 20 мая) - это соль земли, ответственный и зрелый человек, которого уважают и обожают. Крепость для своих близких; мягкий, добрый и ласковый человек. Голова у него всегда «в порядке».

Принимает людей такими, какие они есть, обладает огромной силой самоконтроля. Чувствительная струнка скрыта от посторонних.

Тельцом управляет больше чувства, нежели разум, он верен, ценит друзей, требователен, у него приятные характер и манеры. Его трудно вывести из равновесия. Но в гневе Телец похож на разъяренного быка.

Телец - знак Земли, непоколебим, олицетворяет силу, незыблемую, как скала, пока не затрагивает его вулканическое нутро. Практичный, надежный, последовательный, упорный, трудолюбивый, но жесткий, черствый, чаще пессимист.

Ведущий 2. Шуточное развлечение «Вопросы – Ответы»

Одному человеку ведущий дает тянуть из конверта бумажку с вопросом, другому – с ответом. Игрок задает второму игроку вопрос, и получает на него ответ. И так далее, по очереди, пока конверты не опустеют. Это всегда очень смешно.

Вопросы:

1. Скажи, ты всегда такой красивый?

2. Скажи, ты списываешь на уроках?

3. Скажи, ты стираешь двойки в дневнике?

4. Любишь ли ты делать подарки?

5. Часто ли ты падаешь с кровати во сне?

6. Когда тебя не видят, ты ковыряешься в носу?

7. Ты поёшь в туалете?

8. Ты любишь объедаться тортами на ночь?

9. Правда, что по понедельникам ты ешь только соленые огурцы?

10. Правда ли, что ты хочешь сменить цвет волос на фиолетовый?

11. Правда ли, что ты спишь только в розовой пижаме со слонами?

12. Правда ли, что ты купаешься с резиновыми уточками?

Ответы:

1. Я не представляю свою жизнь без этого.

2. Нет, я очень стеснительный человек.

3. Только после того, как получу двойку.

4. Конечно, вместо того, чтобы делать уроки.

5. Когда прогуливаю математику.

6. Да, часами, особенно в темноте.

7. Ну надо же! Как вы угадали?!

8. В принципе нет, но как исключение – да.

9. К этому у меня склонность с детства.

10. Если не видят родители.

11. По субботам это у меня – необходимость.

12. Это давно уже мое самое большое желание.

Игра “Цветочная лотерея”.

Нужно заранее вырезать круги из бумаги и написать на них названия цветов, приготовив и бумажки в корзинке. Бумажные круги нужно разложить на пол.

Под музыку дети танцуют вокруг кругов, не наступая в них. Когда музыка замолкает, каждый ребенок занимают любой из кругов. Ведущий, не глядя, вытягивает из корзинки бумажку с названием цветка и зачитывает его. Кто наступил на кружок с таким же названием цветка, проиграл.

БИТВА ВОЗДУШЫХ ШАРИКОВ.

Каждому игроку (8 человек именинников) к правой ноге (к щиколотке) привязывается воздушный шарик. После стартового сигнала все участники стараются проткнуть шарики других игроков и уберечь свой. Участники, у которых воздушный шарик лопнул, выбывают из игры. Последний человек, оставшийся в игре, объявляется победителем.

Нитка у шарика должна быть не длиннее 30 см.

Конкурс «Фруктовый сад»

Чтоб выросли фрукты и ягоды,

Деревья должны расцвести.

Пока же попробуйте в ложечке

маленькой

Апельсин до конца донести.

На линии старта выстраивается две команды по 5 человек.

Игроки должны пронести большой апельсин в маленькой ложечке до

линии финиша. Побеждает та команда, которая сделает это быстрее

Конкурс  «Наш реактивный шарик»

Весной так дышится легко,

Пищит в лесу комарик.

Вздохнём и конкурс проведём

«Наш реактивный шарик».

Вам надо лишь по шарику

Надуть и отпустить.

Тот победит, чей шарик

Подальше улетит.

Участники конкурса становятся в одну линию. По команде ведущего они начинают надувать шарик. Ведущий считает до десяти. На счёт «десять» все игроки выпускают свои шарики. Побеждает тот игрок, чей шарик улетел дальше.

Ведущий 1. День рожденья шагнул за порог.

                    Где же наш именинный торт?

  По запаху слышу – он тут!

  Красивый и сладкий торт нам несут!

 Скорее в круг вставайте,

  И торт именинный встречайте!



Предварительный просмотр:

Нормы СанПина

Независимо от продолжительности учебной недели число уроков в день более 6 уроков - в 5-м классе. У нас: 4 дня по 6 уроков и 1 день (пятница) 5 уроков. Итого: 29 часов. Норма!

Для предупреждения переутомления и сохранения оптимального уровня работоспособности организуют облегченный учебный день - четверг или пятница.

 Объем домашних заданий (по всем предметам) должен быть таким, чтобы затраты времени на его выполнение не превышали (в астрономических часах) в  5 классе - 2 ч.

Шкала трудности предметов, изучаемых в 5  классе

Общеобразовательные предметы

Количество баллов

 (ранг трудности)

Биология

10

Математика

10

Иностранный язык

9

Русский язык

8

Обществоведение

6

История

5

Труд

4

Литература

4

ИЗО

3

Физическая культура

3

Музыка

2

Подготовка домашних заданий.

При выполнении обучающимися домашних заданий (самоподготовка) следует соблюдать следующие рекомендации:

- приготовление уроков проводить в закрепленном учебном помещении, оборудованном мебелью, соответствующей росту обучающихся;

- начинать самоподготовку в 15 - 16 часов, так как к этому времени отмечается физиологический подъем работоспособности;

- ограничивать длительность выполнения домашних заданий, чтобы затраты времени на выполнение не превышали (в астрономических часах): в  5 классе - 2 ч.

- предоставлять по усмотрению обучающихся очередность выполнения домашних заданий, рекомендуя при этом начинать с предмета средней трудности для данного обучающегося;

- предоставлять обучающимся возможность устраивать произвольные перерывы по завершении определенного этапа работы.

Нормы домашнего задания

Объем домашних заданий не должен превышать 50% объема аудиторной нагрузки.

Организация помощи ребенку в приготовлении домашних заданий               родителями

Родители всегда пытаются помочь своим детям в приготовлении уроков. Эта помощь варьируется от отдельных кратких объяснений до полного выполнения задания вместо ребенка. В любом случае возникает масса проблем. В попытках справиться с ними родители используют различные методы: подкуп, угрозы, наказания, рассуждения. Мотивы взрослых вполне объяснимы. Разве можно найти что-то плохое в естественной родительской озабоченности? Однако очень часто родители только усугубляют школьные стрессы своих детей.

Если принять во внимание требования, предъявляемые в школе к выполнению заданий, беспокойство многих ребят относительно одобрения своих действий со стороны учителей и страх потерпеть неудачу перед классом, то становится ясно, что школьники надеются спокойно отдохнуть дома. Однако этого не происходит - родители «стоят у них над душой» во время выполнения домашних заданий, не позволяя расслабиться. Это сродни ситуации, когда работающий человек, получив долю стресса на рабочем месте, возвращается домой в такую же напряженную обстановку. Подумайте, сколько времени вы сможете выдержать подобную ситуацию?

Какой должна быть атмосфера домашних уроков, и чем она должна отличаться от школьной? Прежде всего, занятия дома должны быть лишены школьной напряженности, ребенок может встать и подвигаться, как ему хочется. Родитель в роли учителя не ставит оценок. Дома можно не знать, не уметь, не понимать, не успевать, ошибаться - никто не поставит двойку, не сделает запись в дневнике.


Безоценочность рождает обстановку свободы, спокойствия, творчества, безопасности, в которой можно учиться новому в ситуации поддержки и веры в успех, а не стресса. Именно в такой атмосфере у ребенка развивается и усиливается интерес к новым знаниям и умениям, а не стремление получить хорошую оценку любой ценой.

Еще один плюс: домашние занятия могут организовываться с учетом индивидуальных особенностей работоспособности ребенка. Для некоторых детей характерна так называемая эпизодическая работоспособность, при которой ребенок может удерживать внимание на учебных задачах в течение 7-10 минут, а затем отвлекается, давая своей нервной системе время для восстановления. Пятиминутный перерыв для отдыха - и школьник готов продолжать заниматься. В школе нет возможности предусматривать подобные перерывы для каждого ученика, а дома родители для собственного ребенка могут организовать индивидуальный режим обучения, учитывающий пики и спады его работоспособности. Особенно полезен индивидуальный подход к  гиперактивным или слишком тревожным детям.

Если вы решили помогать ребенку в приготовлении домашних заданий, стоит запастись терпением и выдумкой, чтобы превратить занятия не в мучительную процедуру, а в увлекательный способ общения и познания, приносящий истинное удовольствие и пользу ребенку и вам. Представьте, что вы вместе отправляетесь в непростое путешествие из пункта "Не могу, не знаю, не умею" в пункт "Могу, знаю, умею!". Причем главная роль принадлежит не вам - вы только сопровождаете отважного маленького путешественника, однако наблюдать, направлять, помогать намного сложнее и ответственнее, чем делать самому. Поэтому вам понадобится больше выдержки, сил, уверенности в успехе, чем ребенку. Чтобы облегчить вашу миссию, ниже привожу основные правила организации индивидуальной помощи ребенку дома, которые способны принести ему действительно пользу, а не вред.

1. Ниже приведен список приемов, которые необходимо знать родителям для оказания помощи школьникам в процессе выполнения домашних заданий.

1)
 Выполнение распорядка дня. Многим детям помогает соблюдение четкого распорядка дня. Для ребят слишком велико бремя ответственности, если им приходится самостоятельно решать, когда приниматься за выполнение домашней работы. Детям надо заранее решить, делать ли им уроки сразу после прихода из школы или, например, после ужина. Однако, после того как примерное время выполнения домашних заданий определено, нужно придерживаться расписания настолько четко, насколько это возможно.

Четкое выполнение распорядка дня поможет справиться с такой проблемой, как «вылавливание» детей и «усаживание» их за работу. Через некоторое время домашняя работа станет просто естественной частью их ежедневного расписания. Замечу, что время, отведенное на ее выполнение, не должно ничем прерываться. Телефонные звонки, телевизионные передачи и все остальное может подождать, пока работа не будет завершена.

Вечером родителям следует обязательно проверить выполненную домашнюю работу. Многие дети очень волнуются, боясь принести в школу задания, выполненные с ошибками. Поэтому очень важно, чтобы родители регулярно проверяли работу. Такое поведение взрослых порождает у ребенка ощущение завершенности работы, является признаком доброжелательного внимания родителей, а также формирует чувство безопасности и уверенности, что задания выполнены без ошибок. Эту уверенность школьник принесет с собой в класс, а значит, будет чувствовать себя спокойнее, когда станет выполнять классную работу. Однако если ребенок не понял какой-то определенный материал, то родителям нужно сообщить об этом учителю.

2)
 Распределение заданий по степени сложности. Для некоторых детей является проблемой сделать выбор, с какого задания начинать выполнение домашней работы. Они могут очень долго мучиться над ее решением. Чтобы подсказать, с каких заданий лучше начинать - легких или трудных, надо понаблюдать, как ребенок включается в работу и насколько быстро утомляется. Если он начинает работать сразу и без затруднений, но подъем быстро сменяется спадом, посоветуйте ему начинать с более трудных заданий. Если раскачивается медленно, но эффективность работы постепенно нарастает, можно начать с более легких уроков.

3)
 Выполняйте домашние задания вместе с ребенком, а не вместо него. Ответственность за их выполнение в любом случае остается на ученике, а не на родителях. Постарайтесь убедить ребенка в том, что добросовестное приготовление уроков значительно облегчает выполнение классных заданий, что дома можно выяснить все то, о чем он не смог спросить в школе и без стеснения потренироваться в том, что пока не получается. Все-таки главное для нас - не пятерки любой ценой, а интерес ребенка к познанию и уверенность его в своих силах. Поэтому, если уж вы помогаете делать уроки, то добиваетесь не того, чтобы работа была сделана без единой помарки и ошибки, а чтобы к ребенку пришло понимание материала.


4)
 Родителям не следует сидеть рядом с ребенком все время, пока он делает уроки. Многие родители скажут, что их дети не умеют самостоятельно работать и им приходится сидеть рядом с ними. На самом деле это не так. Видя, что родители готовы постоянно находиться рядом, дети сознательно решают ничего не делать самостоятельно. Кое-кто просто перестает работать, как только родитель отходит. Часто такие несамостоятельные школьники не справляются с классной работой, принося недоделанные задания домой.

Если родители уже оказались в такой ситуации, не следует немедленно менять установившийся порядок вещей. Надо двигаться постепенно. Несколько дней подряд взрослым целесообразно садиться как можно дальше от ребенка, выполняющего домашнее задание. Постепенно родителям следует увеличивать расстояние между собой и школьником, пока он не станет работать полностью самостоятельно.

5)
 Выполняйте с ребенком только то, что задано в школе. Не стоит перегружать школьника дополнительными заданиями. Помните, ребенок находится в школе 6-7 часов, а затем его "рабочий день" продолжается, когда он выполняет уроки дома. Жизнь ребенка не должна состоять из одной умственной деятельности! Ребенок - целостное существо, и для его полноценного развития важны движение, общение, труд и творчество.

6)
 Ограничьте время на приготовление уроков в целом и время совместной вашей работы. Договоритесь с ребенком, что будете работать вместе 1-1,5 часа, но с полной отдачей. Выполнение уроков в течение двух и более часов истощает силы ребенка, в результате умственную работу он связывает только с негативными эмоциями. Ограничение времени совместной работы учит ребенка ценить вашу поддержку и помощь.

 Не следует разрешать школьнику сидеть за уроками весь вечер напролет. Иногда родители разрешают ребенку сидеть над домашним заданием несколько часов подряд. Это вполне нормально, если школьник все это время действительно работает и задание, в самом деле, требует столько времени на выполнение. Однако если взрослый видит, что за час или два ребенок почти не продвинулся в его выполнении, то надо прекратить бесполезное занятие. В таком случае целесообразно написать учителю записку, объясняющую происшедшее, или встретиться с педагогом.

Каковы причины неспособности школьника справиться с заданием дома?

  • Он мог просто не понять новый материал, и поэтому был не в состоянии выполнить домашнее задание.
  • Возможно, у ребенка уже сформировалось чувство беспомощности. В этом случае, если он будет долго сидеть над заданием, велика вероятность, что оно будет выполнено родителями.
  • У ребенка могут быть серьезные проблемы с обучением в целом, например, по причине несформированности необходимых учебных умений и навыков.
  • Ученик не в состоянии справиться с большим объемом задания.
  • Если вы считаете, что ваш ребенок испытывает перегрузку по какому-либо из учебных предметов, обратитесь за разъяснением  и помощью к учителю -  предметнику или завучу.

    7)
    Выполняя домашние задания с ребенком, придерживайтесь удобного для него темпа. Если вы будете торопить и подгонять, это лишь создаст нервозную обстановку, не способствующую мыслительной работе, но вряд ли увеличит скорость выполнения. Призывы: "Внимательнее", "Не отвлекайся!" тоже не принесут пользы. Ребенок отвлекается потому, что его нервная система требует времени для восстановления. Если ребенок продолжает заниматься под принуждением, то он устанет гораздо быстрее. Заметив признаки утомления (ребенок роняет карандаш или ручку, не будучи голодным, просит есть или пить, задает отвлеченные вопросы, берет в руки посторонние предметы и т. д.), предоставьте ему возможность отвлечься от занятий на 5-7 минут, затем вновь вовлеките в учебный процесс.

    8)
     Не ставьте перед ребенком несколько разноплановых задач одновременно. Например, сидеть прямо, писать красиво, думать быстро. Постоянное торпедирование командами-приказами: "Выпрямись", "Не дергай ногой!", "Не кусай губы!", "Пиши ровно" - приводит к прямо противоположному результату: ребенок тут же отвлекается, и ему трудно вновь сосредоточиться. Выделите для себя ту главную задачу (прочитать и разобраться с текстом, понять решение задачи, грамотно переписать упражнение и т. д.), а остальные отодвиньте на другое время.

    9)
     Помощь при заучивании информации из учебника. Требуйте от своего ребенка внимательного прочтения инструкций по выполнению учебных заданий, формулировки вопросов. Учите его детальному изучению содержания материалов учебника, его справочных материалов, правил и инструкций.

    Часто школьники не знают, на что им надо обращать внимание в процессе чтения учебного текста. В большинстве учебников в конце каждого параграфа есть вопросы. Родителям стоит обсудить их с ребенком до того, как он начнет читать учебник. Используя такую тактику, школьник будет знать, на какую важную информацию надо обращать внимание. Некоторые дети стремятся запомнить абсолютно все, что написано в учебнике. Тогда целесообразно дать такому ребенку карандаш и попросить его отметить слово или предложение, которое, по его мнению, является ответом на один из вопросов.



    Если ребенок сделал ошибку, то взрослый должен помочь ему найти ее и исправить. Ваша цель - упростить задачу для ребенка, построить ему такую новую мини-задачу, которая окажется ему по силам и поможет понять причину совершенной ошибки. Например, если ребенок при сложении 27 и 15 получает 32, вы его спрашиваете: "А сколько будет 17 и 15?". Получив ответ в новой задаче - 32, ребенок натыкается на противоречие: 15 складывается с разными числами, а ответ один! Так он приводит его к обнаружению ошибки. 

    11)
     Сначала проверить то, что выполнено правильно. Как правило, родители в первую очередь обращают внимание на ошибки своих детей. Взрослым стоит взять за правило отмечать, как хорошо школьник выполнил те задания, которые сделаны без ошибок. А относительно заданий, в которых допущена ошибка, сказать ребенку: «Я думаю, что если ты еще раз проверишь этот пример, то у тебя может получиться несколько другой ответ». Это побудит ученика вернуться к заданию без отвращения и чувства бессилия. Если родитель начнет проверку домашнего задания с разбора ошибок, да еще и рассердится, то ребенок, вместо исправления недочетов, будет переживать, что вызвал неудовольствие взрослого.

    Иногда целесообразно проверять домашнее задание по частям, сразу после того как школьник решил задачу, выполнил упражнение. Для многих детей важно немедленно получить подтверждение, что все сделано без ошибок или как можно быстрее исправить погрешности в работе. Таким образом, ребенок достаточно быстро может получить обратную связь относительно хода выполнения уроков. У него возникает желание хорошо выполнить следующее задание. Если же ребенок начал что-то делать неправильно, то он имеет возможность сразу обнаружить и объяснить ошибку. Это означает, что школьнику не придется потом переделывать все задание целиком.

    Когда ребенок может полностью перейти на самоконтроль, зависит от его индивидуальных особенностей. Хорошо, чтобы такое умение выработалось к моменту его перехода в среднюю школу. Ученик средних и старших классов должен уже владеть способами самопроверки с помощью логических и образных схем.

12) Хвалите ребенка за своевременно и качественно выполненное домашнее задание. Демонстрируйте его достижения перед другими членами семьи, братьями и сестрами. Не сравнивайте его умения с умениями других детей. Поощряйте упорство и проявление характера достижения цели.

Какие бы отметки ребенок ни получил, все же сейчас он готов заниматься - поддержите его в этом своей похвалой и одобрением. Важно, чтобы ребенок поверил, что вы его продолжаете любить независимо от оценок, полученных в школе. Радуйтесь каждый раз, когда заметите в его глазах интерес к знаниям, пусть даже при выполнении простых задач. Помните, человеку нравится то, что у него получается. Задавайте ребенку задания и вопросы по силам.
Даже маленький успех, увеличенный родительским вниманием и одобрением, окрыляет, разжигает интерес, рождает желание решать более сложные учебные задачи.

13)
 Исключите из своей речи негативные оценочные высказывания. Подобные фразы никоим образом не стимулируют умственную деятельность ребенка, но значительно ухудшают его эмоциональное состояние. Как правило, они произносятся под действием накопившихся за время занятий эмоций раздражения, обиды, пренебрежения, страха, злости, а иногда и ненависти. Если вы решили помочь ребенку, оставьте на заранее оговоренное время все свои дела и заботы. В противном случае возникает ситуация, когда перспектива заняться более привлекательным делом приводит к тому, что взрослому становится жалко "тратить драгоценное время" на обучение ребенка ("я бы уже за это время "), и в результате вырываются обидные (и отнюдь не способствующие повышению умственной активности) слова: "Ну неужели нельзя это было сделать правильно с первого раза'", "Это неправильно, думай еще!"

Иногда взрослые употребляют в общении с ребенком фразы, подобные следующим: "Ты просто не хочешь думать!", "Ты очень невнимателен" или "Если будешь так стараться, никогда не получишь ничего больше двойки!" Это так называемые формулы прямого внушения, которые можно распознать по часто используемым в них словам: "всегда", "никогда", "опять", "все время", "вечно". Иногда в них содержатся негативные характеристики ребенка: "Ты невнимательный", "Ты ленишься", "Ты нытик". Еще родители могут показывать ребенку его малопривлекательное (по причине сегодняшних неудач в учебе) будущее. Стоит ли говорить, что такие фразы нужно по возможности исключать из родительского репертуара общения с ребенком.

14)
 Родителям стоит обращать внимание на невербальные сигналы, которые они посылают своим детям в ходе выполнения домашнего задания. Родители часто говорят, что никогда не впадают в гнев и не кричат на своих детей, когда помогают им делать уроки. Однако общение происходит не только на вербальном уровне. Известно, что невербальные способы передачи информации являются не менее значимой частью общения. Поэтому очень многие сигналы, особенно негативные, могут быть переданы довольно просто, даже если родители сами об этом не подозревают.

Гримасы, напряженность позы, вздохи, поднятые брови и другие проявления «языка тела» - все это является невербальными ответами на промахи детей. Если они достаточно чутки, то быстро примут эти сигналы. Это только добавит напряженность в ваши взаимоотношения, связанные с домашней работой.

15) Для того, чтобы облегчить своему ребенку выполнение домашних заданий, покупайте ему энциклопедии, словари и справочные пособия по различным предметам, справочники  на информационных носителях.

16) Формируйте привычку доводить начатое дело до конца, даже если придется чем-то жертвовать.


Домашние задания и... хорошее настроение.

У ребенка выработается и затем сохранится положительный настрой на приготовление домашних заданий, если вы (родители):


- с самого начала дадите ему понять, что его уроки столь же важны, сколько и самые серьезные дела взрослых; никто не имеет права оторвать школьника от его дела, послав в магазин или включив телевизор;


- в своей семье будете поддерживать атмосферу уважения к умственному труду;


- вернувшись домой с работы или, наоборот, встречая ребенка из школы, не станете начинать общение с вопроса об уроках; найдете другую форму приветствия;


- не будете стоять у ребенка "над душой", пока он не начнет делать уроки или в процессе работы;


- никогда не станете использовать выполнение домашних заданий как средство наказания за проступки;


- постараетесь не напоминать ребенку о его многочисленных прошлых промахах и неудачах и не напугаете предстоящими трудностями; сформируете отношение к трудностям как к чему-то вполне преодолимому;


- проверяя сделанное, не будете злорадствовать по поводу ошибок ("Я так и знал, что ты их насажаешь!");


- в случае, если ошибки действительно есть, все равно найдете возможность похвалить ребенка за затраченные усилия; отметите любые, даже незначительные успехи ("Сегодня эта буква у тебя получается лучше, чем вчера", "Ты сегодня так старался!").


Родительское собрание

«Организация домашней  учебной  работы  школьников»

Классный руководитель 5 класса

Соболева Галина Петровна

Ёлнать, 2013г.


Предварительный просмотр:


Подписи к слайдам:

Слайд 1

Организация домашней учебной работы школьников

Слайд 2

Общеобразовательные предметы Ранг трудности (количество баллов) Биология 10 Математика 10 Иностранный язык 9 Русский язык 8 Обществознание 6 История 5 Труд 4 Литература 4 ИЗО 3 Физическая культура 3 Музыка 2

Слайд 3

Объём домашних заданий не должен превышать 50% объёма аудиторной нагрузки Затраты времени на выполнение не должны превышать 2 астрономических часа

Слайд 4

Занятия дома должны быть лишены школьной напряжённости ребёнок может встать и подвигаться, как ему хочется д ома можно не знать, не уметь, не понимать, ошибаться – никто не поставит двойку

Слайд 5

Приёмы для оказания помощи школьникам в процессе выполнения домашних заданий Выполнение распорядка дня Распределение заданий по степени сложности Выполняйте домашние задания вместе с ребёнком, а не вместо него Родителям не следует сидеть рядом с ребёнком всё время, пока он делает уроки Выполняйте с ребёнком только то, что задано в школе Ограничьте время приготовления уроков. Не следует разрешать школьнику сидеть за уроками весь вечер

Слайд 6

Причины неспособности школьника справиться с заданием дома Не понял новый материал Сформировалось чувство беспомощности Не сформированы необходимых учебных умений и навыков Не в состоянии справиться с большим объёмом задания

Слайд 7

Приёмы для оказания помощи школьникам в процессе выполнения домашних заданий 7 . Выполняя домашние задания с ребёнком, придерживайтесь удобного для него темпа 8. Не ставьте перед ребёнком несколько разноплановых задач одновременно 9. Помощь при заучивании информации из учебника 10. Сначала проверить то, что выполнено правильно 11. Хвалите ребёнка 12. Исключайте из своей речи негативные оценочные высказывания 13. Родителям стоит обращать внимание на невербальные сигналы, которые они посылают свои детям 14. Формируйте привычку доводить начатое дело до конца, даже если придётся чем-то жертвовать

Слайд 8

Домашние задания и … хорошее настроение С самого начала дайте понять ребёнку, что его уроки столь важны, сколько и самые серьёзные дела для взрослых В своей семье поддерживайте атмосферу уважения к умственному труду Встречая ребёнка из школы не начинайте общение с вопроса об уроках Не напоминайте ребёнку о его многочисленных прошлых промахах и неудачах, не пугайте предстоящими трудностями В случае, если ребёнок допустил ошибки, всё равно найдите возможность похвалить его за затраченные усилия



Предварительный просмотр:

Родительское собрание «Дети и родители: идеалы и реальность»

        Как редко ребёнок бывает

таким, как  нам хочется …

Януш Корчак

Со школьниками было проведено тестирование. Которое выявило среди них визуалов, аудиалов и кинестетиков.

Эти слова означают особенности восприятия и переработки информации людьми различного возраста.

Визуал - человек, воспринимающий большую часть информации с помощью зрения.

Аудиал - тот, кто получает основную информацию через слух.

Кинестетик - тот, кто воспринимающий информацию через другие ощущения (обоняние, осязание и др.) и с помощью движений.

Такие особенности психики проявляются уже в дошкольном возрасте и играют большую роль в налаживании контактов ребенка с окружающими, в успешности обучения, получения новых навыков и умений. Как распознать у человека ведущий канал восприятия и переработки информации? Часто достаточно понаблюдать!

ДОШКОЛЬНЫЙ ВОЗРАСТ

Малыш-визуал при восприятии информации больше полагается на зрительные ощущения. Он быстро запоминает цвет, форму, размер. Если малышу предложить новую игрушку, он сначала внимательно рассмотрит ее со всех сторон. Дети-визуалы любят лепить, рисовать, вырезать, складывать пазлы. Их занятия и игры включают в себя взаимодействие глаз и рук. Такие детки предпочитают рассматривать картинки в книжке, чем слушать сказку. Для ребенка-визуала важно, что на нем одето, он сам выбирает цвет колготок, платья, кофточки; девочки трепетно относятся к украшениям, заколкам, бантикам. Малыш-визуал первым делом запомнит и расскажет, кто, в чем пришел сегодня в садик, у кого на площадке была самая яркая игрушка. Визуал довольно аккуратен (для своего возраста), он, например, не полезет в лужу, потому что ботинки будут грязными, некрасивыми. В общении со сверстниками ребенок-визуал сначала наблюдает, а уже потом идет на контакт (совместную игру), может подолгу играть один. В воспитании малыша-визуала необходимо уделять внимание развитию речи (он неразговорчив), коммуникабельности (умению общаться), физической координации.

Дошкольник-аудиал познает окружающий мир через звуки, поэтому раньше других начинает говорить, имеет большой словарный запас. Такие малыши любят слушать музыку, сказки, вести разговоры - обсуждать, доказывать. Придумывает различные сказки, игры. Умеет убеждать, уговаривать. Но в подвижных играх участвовать не любит. Дети-аудиалы очень общительны. Они могут немного отставать от сверстников в развитии навыков, связанных со зрительным (поиск отличий на картинках) и моторным (умение ловко залазить на горку, быстро и долго бегать) восприятием. Поэтому стоит эти навыки развивать с помощью игр, упражнений.

Маленький кинестетик познает мир с помощью прикосновений, движений. У него хорошо развита моторика, он очень много двигается, обычно рано начинает ходить. Такой малыш любит активные игры, связанные с прыжками, бегом, борьбой, ему трудно усидеть на одном месте, долго заниматься одним делом. Малыш-кинестетик любит все трогать, ощупывать; ему особенно важны тактильные ощущения, поэтому чаще ласкайте, прижимайте к себе, целуйте, старайтесь меньше употреблять любимую родителями фразу "Не трогай!"

ШКОЛЬНЫЙ ВОЗРАСТ

Словарь

  • Визуалы в своей речи употребляют существительные, глаголы, прилагательные, связанные со зрением (смотреть, наблюдать, картина, на первый взгляд, красочный, яркий, как видите...).
  • Для аудиала характерны слова, связанные со слуховым восприятием (голос, послушайте, обсуждать, молчаливый, тишина, беззвучный...).
  • Словарь кинестетика в основном включает слова, описывающие чувства, ощущения (схватывать, мягкий, теплый, шелковистый, прикосновение, хороший нюх, ароматный, гибкий...).

Направление взгляда

  • У визуалов при общении взгляд направлен в основном вверх,
  • у аудиалов - по средней линии,
  • у кинестетиков - вниз.

Особенности внимания

  • Кинестетику вообще трудно концентрировать свое внимание, его легко отвлечь чем угодно;
  • аудиал легко отвлекается на звуки;
  • визуалу шум практически не мешает.

Особенности запоминания визуалов, аудиалов и кинестетиков

  • Визуал помнит то, что видел, запоминает картинами (образное мышление хорошо развито).
  • Аудиал - то, что обсуждал, запоминает, слушая.
  • Кинестетик помнит общее впечатление; запоминает, двигаясь, ощупывая, нюхая.

Различия в поведении кинестетиков, аудиалов и визуалов

Ситуация: домашнее задание, которое записано на доске.

  • Визуал послушно откроет дневник и перепишет с доски то, что задано на дом.
  • Аудиал переспросит у соседа и запишет со слуха или не будет вообще ничего писать, а дома позвонит по телефону и узнает у одноклассников, что задано.
  • Кинестетик долго роется в портфеле, находит нужные учебники, в них - заданные страницы и прямо в учебниках отмечает, что задано.

Ситуация: поведение на перемене.

  • Визуал чаще всего остается в классе (если там не очень шумно) либо выходит в коридор и рассматривает информацию на стенах, наблюдает за другими детьми.
  • Аудиал использует перемену, чтоб наговориться и пошуметь.
  • Кинестетику перемена нужна, чтобы размяться, подвигаться.

Конечно же, при обучении, воспитании необходимо учитывать особенности восприятия и переработки информации. Это относится и к родителям, и к педагогам. Важно общаться с ребенком на "его языке".

Замечания воспримутся лучше, если:

  • визуалу - покачать головой, погрозить пальцем;
  • аудиалу сказать шепотом " ш-ш-ш", "тише";
  • кинестетику положить руку на плечо.

При выполнении ребенком домашнего задания рекомендуется:

  • визуалу разрешить иметь под рукой листок, на котором он в процессе работы может чертить, штриховать, рисовать и т.д.;
  • аудиалу не делать замечания, когда он в процессе запоминания издает звуки, шевелит губами;
  • кинестетика не заставлять сидеть долгое время неподвижно, давать возможность моторной разрядки (сходить в другую комнату, встать и попрыгать...).

При общении используйте методы, понятные и близкие ребенку:

  • с визуалом употребляйте слова, описывающие цвет, форму, местоположение, можно выделять цветом пункты в книге, использовать таблицы, схемы, наглядные пособия;
  • с аудиалом используйте вариации голоса (громкость, паузы, интонации);
  • с кинестетиком используйте жесты, прикосновения, довольно медленную речь, позволяйте ему "обыгрывать" информацию.

Любой человек, ребенок в том числе, использует разные каналы восприятия, то есть, мы не можем быть только визуалами, например. Особенности восприятия зависят и от возраста: для маленьких детей свойственно стремление все потрогать, попробовать на вкус, школьники же чаще задействуют слух и зрение.

Учитесь понимать друг друга, тогда в вашей жизни и в жизни ваших детей будет намного меньше проблем!


Предварительный просмотр:


Подписи к слайдам:

Слайд 1

Как редко ребёнок бывает таким, как нам хочется … Януш Корчак

Слайд 4

ШКОЛЬНЫЙ ВОЗРАСТ Словарь Визуалы в своей речи употребляют существительные, глаголы, прилагательные, связанные со зрением (смотреть, наблюдать, картина, на первый взгляд, красочный, яркий, как видите...). Для аудиала характерны слова, связанные со слуховым восприятием (голос, послушайте, обсуждать, молчаливый, тишина, беззвучный...). Словарь кинестетика в основном включает слова, описывающие чувства, ощущения (схватывать, мягкий, теплый, шелковистый, прикосновение, хороший нюх, ароматный, гибкий...).

Слайд 5

Направление взгляда У визуалов при общении взгляд направлен в основном вверх, у аудиалов - по средней линии, у кинестетиков - вниз. Особенности внимания Кинестетику вообще трудно концентрировать свое внимание, его легко отвлечь чем угодно; аудиал легко отвлекается на звуки; визуалу шум практически не мешает . Особенности запоминания визуалов, аудиалов и кинестетиков Визуал помнит то, что видел, запоминает картинами (образное мышление хорошо развито). Аудиал - то, что обсуждал, запоминает, слушая. Кинестетик помнит общее впечатление; запоминает, двигаясь, ощупывая, нюхая.

Слайд 6

Различия в поведении кинестетиков, аудиалов и визуалов Ситуация: домашнее задание, которое записано на доске. Визуал послушно откроет дневник и перепишет с доски то, что задано на дом. Аудиал переспросит у соседа и запишет со слуха или не будет вообще ничего писать, а дома позвонит по телефону и узнает у одноклассников, что задано. Кинестетик долго роется в портфеле, находит нужные учебники, в них – заданные страницы и прямо в учебниках отмечает, что задано.

Слайд 7

Различия в поведении кинестетиков, аудиалов и визуалов Ситуация: поведение на перемене. Визуал чаще всего остается в классе (если там не очень шумно) либо выходит в коридор и рассматривает информацию на стенах, наблюдает за другими детьми. Аудиал использует перемену, чтоб наговориться и пошуметь. Кинестетик у перемена нужна, чтобы размяться, подвигаться. Замечания воспримутся лучше, если: визуалу - покачать головой, погрозить пальцем; аудиалу сказать шепотом " ш-ш-ш ", "тише"; кинестетику положить руку на плечо .

Слайд 8

При выполнении ребенком домашнего задания рекомендуется: визуалу разрешить иметь под рукой листок, на котором он в процессе работы может чертить, штриховать, рисовать и т.д.; аудиалу не делать замечания, когда он в процессе запоминания издает звуки, шевелит губами; кинестетика не заставлять сидеть долгое время неподвижно, давать возможность моторной разрядки (сходить в другую комнату, встать и попрыгать...). При общении используйте методы, понятные и близкие ребенку: с визуалом употребляйте слова, описывающие цвет, форму, местоположение, можно выделять цветом пункты в книге, использовать таблицы, схемы, наглядные пособия; с аудиалом используйте вариации голоса (громкость, паузы, интонации); с кинестетиком используйте жесты, прикосновения, довольно медленную речь, позволяйте ему "обыгрывать" информацию.

Слайд 9

Учитесь понимать друг друга, тогда в Вашей жизни и в жизни Ваших детей будет намного меньше проблем!



Предварительный просмотр:

                           Здоровые дети в здоровой семье

Уважаемые родители! Помните!
Здоровая семья — это крепость, в которой ребенок чувствует себя
защищенным! Это гнездо, в котором ему спокойно и уютно. Семья,
родители должны удовлетворить многие жизненно-важные потребности
ребенка, должны многому научить детей. Ребенок — это пластилин, что
из него вылепишь — то и будешь иметь! Это чистая доска, что
напишешь, — то и останется на всю жизнь! В здоровой семье —
здоровый ребенок! В больной семье — больной!

Рекомендации родителям о здоровом образе жизни ребёнка

Дорогие взрослые! Вашему ребёнку как воздух необходим режим дня, разумно составленный, соответствующий возрастным особенностям. То, что ребёнок привыкает в одно и то же время есть, спать, активно действовать, создаёт благоприятные предпосылки для его всестороннего развития.

 Режим дня должен быть достаточно гибким. В зависимости от условий (домашних, климатических, от времени года, индивидуальных особенностей ребёнка) он может меняться, но не более чем на 30 минут в ту или иную сторону. 

После зарядки ребёнку необходимо принимать водные процедуры (закаляться водой). Начинать надо с самых простых процедур: умывания, обтирания, потом перейти к обливанию, к прохладному душу, а можно и к более сложным — купанию в бассейне или открытом водоёме. 

Снижать температуру воды надо постепенно — с учётом возраста вашего ребёнка, состояния его здоровья, характера процедуры. Проводить процедуры следует систематически, примерно в одни и те же часы, предварительно хорошо настроив ребёнка. В качестве местных закаливающих процедур используются: умывание, обтирание по пояс, мытьё рук и обливание, игры с водой, «топтание в тазу».

Недостаточная двигательная активность – гипокинезия – все больше «молодеет». Она отмечается не только у детей старшего возраста, но все чаще у младших школьников, дошкольников и даже у совсем маленьких детей. Двигательная активность является важнейшим компонентом образа жизни и поведения дошкольников. Она зависит от организации физического воспитания детей, от уровня их двигательной подготовленности, индивидуальных особенностей, телосложения и функциональных возможностей растущего организма. Дети, систематически занимающиеся физкультурой, отличаются жизнерадостностью, бодростью духа и высокой работоспособностью. Физическая культура занимает ведущее место в воспитании детей и немалая роль в приобщении детей с раннего возраста к здоровому образу жизни ложится на плечи родителей.

 

Для снятия возбуждения перед сном можно посидеть на кровати рядом с ребенком; мягко, плавно погладить его по спине вдоль позвоночника, поцеловать, сказать, что завтра все будет хорошо, все у него получиться, что он умный и добрый.

Следить, чтобы пробуждение ребенка было плавным (он должен полежать в кровати не менее 10 минут; ставить будильник в изголовье детской кроватки противопоказано).

 Проводить с ребенком водные гигиенические процедуры, а после проветривания комнаты - утреннюю гимнастику под музыку.

Использовать при приготовлении блюд продукты, богатые минералами и микроэлементами, белками, легкими углеводами, витаминами.

В период риска заболевания ОРВИ давать, в виде добавки к супам, чеснок и зеленый лук.

Обеспечивать ребенка одеждой из натуральных волокон, чтобы она способствовала полноценному кожному дыханию и правильному теплообмену.

Нужно убедительно сказать ребенку, что после физкультуры, необходимо снять спортивную майку и надеть сухую, сменную.

Предупредить ребенка, что сразу после физкультуры нельзя пить холодную воду.

 

Совместный активный досуг:

 

 1. способствует укреплению семьи;

 2. формирует у детей важнейшие нравственные качества;

 3. развивает у детей любознательность;

 4. приобщает детей к удивительному миру природы, воспитывая к ней бережное отношение;

 5. расширяет кругозор ребенка;

 6. формирует у ребенка первичные представления об истории родного края, традициях, культуре народа;

 7. сближает всех членов семьи (дети живут одними задачами с родителями, чувствуют причастность к общему делу).

 

Проводя совместно досуг, у родителей и детей возникает тот духовный контакт, о котором многие родители только мечтают.

 

Рекомендации родителям по организации активного семейного отдыха

 

Проведение совместного семейного отдыха – будь то семейные туристические походы, активный отдых на реке, море, участие в спортивных семейных играх, например, «Папа, мама, я – спортивная семья», велосипедные прогулки в лес и т.д. – благотворно действуют на школьников и их родителей. Родители совместно с детьми могут:

 

1. В зимнее время – совершать лыжные прогулки с детьми, кататься на коньках, на санках, совершать пешие прогулки в ближайший лес, парк, лепить во дворе снежные крепости, фигуры.

 2. В весеннее и осеннее время – брать детей с собой в однодневные туристические походы, совершать совместный отдых на реке,  организовывать совместные дворовые затеи с подвижными играми.

 3. В летнее время – загорать, плавать, устраивать шумные, подвижные игры на улице.

 

В заключение хочется предложить родителям сформулированные семейные правила, касающиеся режима дня, закаливания, питания и других составляющих здорового образа жизни.

 

Семейный кодекс здоровья

 

1. Каждый день начинаем с зарядки.

 2. Просыпаясь, не залеживаемся в постели.

 3. Берем холодную воду в друзья, она дарит бодрость и закалку.

 4. В детский сад, в школу, на работу – пешком в быстром темпе.

 5. Лифт – враг наш.

 6. Будем щедрыми на улыбку, никогда не унываем!

 7. При встрече желаем друг другу здоровья (Здравствуй!)

 8. Режим – наш друг, хотим все успеть – успеем!

 9. Ничего не жевать сидя у телевизора!

 10. В отпуск и выходные – только вместе!

 

 Чтоб здоровье сохранить
Чтоб здоровье сохранить,
Организм свой укрепить,
Знает вся моя семья
Должен быть режим у дня.

Следует, ребята, знать
Нужно всем подольше спать.
Ну а утром не лениться–
На зарядку становиться!

Чистить зубы, умываться,
И почаще улыбаться,
Закаляться, и тогда
Не страшна тебе хандра.

У здоровья есть враги,
С ними дружбы не води!
Среди них тихоня лень,
С ней борись ты каждый день.

Чтобы ни один микроб
Не попал случайно в рот,
Руки мыть перед едой
Нужно мылом и водой.

Кушать овощи и фрукты,
Рыбу, молокопродукты-
Вот полезная еда,
Витаминами полна!

На прогулку выходи,
Свежим воздухом дыши.
Только помни при уходе:
Одеваться по погоде!

Ну, а если уж случилось:
Разболеться получилось,
Знай, к врачу тебе пора.
Он поможет нам всегда!

Вот те добрые советы,
В них и спрятаны секреты,
Как здоровье сохранить.
Научись его ценить!

 


Предварительный просмотр:


Подписи к слайдам:



Предварительный просмотр:

Родительское собрание №1.

6 класс

Бранные слова в речи ребёнка: что делать?

        Бранные, или неприличные, слова рано или поздно появляются в лексиконе каждого ребёнка.

Причины появления бранных слов в лексиконе ребёнка

        Почему дети так охотно повторяют нехорошие выражения? В первую очередь их привлекает эмоциональность, с которой эти слова произносят окружающие. Ругающийся человек выглядит безгранично самоуверенным, его жесты очень выразительны. Вокруг него возникает атмосфера волнения и напряжения. Произнесённые таким тоном слова не могут остаться незамеченными окружающими.

        Собственные наблюдения ребёнка и воспитательные беседы, проводимые родными и учителями, наводят его на мысль, что умение вставить в свою речь крепкое словечко является одним из признаков взрослости. И если родители говорят, что использовать подобные слова можно только взрослым, - естественно, ребёнок, стремящийся во всём походить на старших, будет намеренно употреблять запрещённые выражения.

        В речи родителей и педагогов часто можно услышать слова с частицей «не»: «не ходи туда», «не делай так», «не надо», «нельзя» и т.д. Отрицательных речевых конструкций гораздо больше, чем побудительных. В одном ряду с этими запретами – вето на определённую речевую стратегию: «не ругайся матом», «не выражайся», «не хами». Реакция на резкое подавление и запрет предсказуема: дети начинают ругаться, «выражаться», рассказывая матерные анекдоты, писать матерные слова на стенах и т.д.        

        Заметив, что ненормативная лексика шокирует, дети начинают использовать её, чтобы позлить, подразнить окружающих. В этом случае бранные слова становятся орудием мести.

        Мат используется:

  • для повышения эмоциональности речи;
  • эмоциональной разрядки;
  • оскорбления, унижения адресата речи;
  • демонстрации агрессии;
  • демонстрации отсутствия страха;
  • демонстрации раскованности, пренебрежительного отношения к системе запретов;
  • демонстрации принадлежности к «своим».

Но на самом деле сквернословие отражает скудность лексического запаса говорящего и неумение ориентироваться в ситуации наивысшего эмоционального подъёма (радости или гнева).

Издревле матерщина в русском народе именуется сквернословием. Что такое сквернословие? Это речь, наполненная неприличными выражениями, непристойными словами, бранью. Матерная брань – это не просто набор непристойностей. Подобная лексика свидетельствует о духовной болезни человека. Сократ говорил: «Каков человек, такова и его речь».

История возникновения сквернословия

        В Древней Руси мат являлся заклинанием, формулой против нечистой силы. Через матерную брань люди вступали с ней в общение, как бы настраиваясь на её волну. Но все знали, что нельзя бранить детей матом, - они будут мучимы бесами. Материться в доме нельзя – в нём поселятся бесы. Если выругаться в лесу - леший может обидеться, на берегу реки или озера – оскорбится водяной. Где же человеку можно было выругаться, выплеснуть злость? В поле. Отсюда и выражение «поле брани». Многие думают, что оно означает место битвы, а на самом деле – это поле матерной ругани. Интересна теория происхождения слова «мат». Оно означает «крик», громкий голос в значении звукоподражательном. А подражание было крикам животных – «ма» и «мя» - в брачный период. В Древней Руси считалось неприличным уподобляться скоту и орать благим матом о том, что относится к сфере интимных отношений.

Сквернословие в подростковом возрасте

        Ученики 6-9-х классов переживают кризис подросткового возраста, который приходится на период 12-15 лет и связан с половым созреванием. Он характеризуется стремлением школьника к поиску своего места в жизни. Обратите внимание, что использование жаргона отражает гипертрофированный интерес к половой системе человека, функции деторождения и генитальной тематике. К 12-14 годам учащиеся используют жаргон в основном для выплеска эмоций и анекдотах. Но так как мат довольно ограниченный пласт языка, то постепенно расширяя кругозор и повышая эрудицию, ученики средних классов перестают нуждаться в «словесном мусоре» жаргонизмов. В доверительной беседе выясните, что означают «нехорошие слова» в представлении ребёнка (пусть даже объяснит их значение). Не будьте ханжами во время беседы с подопечными. Кстати, зачастую ребята сами не знают эквивалента высказанных непристойных слов и их действительной сути. Продемонстрируйте вашу досаду только в ограниченности знаний школьника и укажите на стилистическую ущербность матерного арго. Дайте понять, что вы намного больше осведомлены в данной теме, но в силу этических причин не употребляете жаргонизмы(они не интересны, ущербны, утомительны в плане постоянного употребления, это слова с отрицательной энергетикой).

        

Что делать педагогам и родителям

        Бесполезно ругать детей за использование неприличных слов или запрещать их произносить. Это сделает ругательства более привлекательными в глазах ребёнка, он будет употреблять их, но постарается, чтобы вы не слышали. Подросткам надо объяснить, что люди используют ругательства лишь в крайних случаях, когда от отчаяния им уже не хватает сил и слов. Во избежание появления ругательств в лексиконе ребёнка взрослым необходимо следить за собственной речью.

        Ещё одна проблема. Как поступить, если ребёнок спрашивает о значении того или иного бранного слова? Рекомендуется не уходить от ответа. Не все родители готовы к такому свободному обсуждению бранных слов с ребёнком. В данном случае надо честно сказать, что значение этого слова настолько неприлично, что вы не хотите его произносить. И уж совсем не стоит поступать, как Волька из сказки Л. Лагина «Старик Хоттабыч». Он в сердцах обозвал Хоттабыча «балдой», а на вопрос старика, что это значит, объяснил: «Балда- это что-то вроде мудреца». И был очень смущён, когда Хоттабыч публично обратился к нему со словами: «О, превосходнейший в мире балда!» Иногда родители ведут себя так же, придумывая, «культурные» объяснения для ругательств.

        Если ребёнок интересуется, почему окружающие употребляют подобные слова, скажите, например, что так говорят люди несдержанные и невоспитанные, когда хотят обидеть и разозлить человека.

        Поскольку ругательства и сквернословия – это зачастую проявление агрессии, важно помочь ребёнку в освоении методов самоконтроля и адекватного выражения эмоций. А первой реакцией на бранные слова может стать помощь в обозначении чувств подростка: «Я понимаю, что ты говоришь так не потому, что хочешь обидеть ребят, а просто тебя очень сердит то, что происходит. Попробуй не ругаться, а договориться с ними».

        Если же ребёнок вас самих «поймал на слове», надо извиниться, сказать, что, к сожалению, вам не удалось сдержаться, вы поступили плохо. Дайте понять, что искренне раскаиваетесь, и впредь старайтесь держать себя в руках.

        


Предварительный просмотр:

Предварительный просмотр:


Предварительный просмотр:

Задачи с числами

Задача 1

В детском саду на утреннике, детям раздавали конфеты. Всего было 234 конфеты. Мальчикам досталось по 4 конфеты, а девочкам по 5. На следующий праздник решили сделать так, чтобы и мальчикам и девочкам досталось по 6 конфет. Сколько конфет решили раздать детям на следующий праздник, если девочек и мальчиков одинаково?

Решение 

  • 1) 4 + 5 = 9 (конфет) мальчику и девочке;
  • 2) 234 : 9 = 26 – мальчиков и девочек по отдельности;
  • 3) 26 * 2 = 52 (ребенка) всего в детском саду;
  • 4) 52 * 6 = 312 (конфет).

Ответ: в следующий раз решили раздать 312 конфет.

2. Старинная задача.

Пятеро сыновей делили наследство. Наследство состояло из 3-х домов. Так как три дома было неудобно делить между пятью братьями, то решили, что трое старших братьев возьмут себе по дому, и каждый из них выплатит младшим братьям по 800 рублей. После того, как младшие братья разделили деньги между собой, оказалось, что всем досталось поровну от наследства. Сколько стоили дома, если предположить, что они имели равную стоимость?

Решение 

  • 1) 800 * 3 = 2400 (рублей) всего выплатили старшие братья;
  • 2) 2400 : 2 = 1200 ( рублей) досталось каждому из младших братьев;

Ответ: дома стоили по 1200 рублей каждый.

Задача 3

В ведре было 40 кружек молока. После того, как из ведра отлили какое-то количество молока, то в нем осталось молока в 7 раз больше, чем отлили. Сколько молока отлили из ведра?

Решение 

  • Так как молока осталось в 7 раз больше, значит, отлили 1/8 ведра, поэтому:
  • 1) 40 : 8 = 5 (л) молока отлили из ведра;

Ответ: из ведра отлили 5 литров молока.

Задача 4

На двух книжных полках стояли книги, всего 12 штук. Сколько книг стояло на каждой из полок, если после того, как с первой на вторую переставили столько книг, сколько стояло на второй, на обеих полках стало книг поровну?

Решение 

  • 1) 12 : 2 = 6 (книг) стало на полках;
  • 2) 6 : 2 = 3 (книг) стояло на второй сначала;
  • 3) 12 - 3 = 9 (книг) стояло на первой сначала;

Ответ: сначала на первой стояло 3 книги, на второй 9 книг.

Задача 5

У Коли и Саши было всего 8 орехов. Сначала Коля дал Саше столько орехов, сколько было у Саши, потом Саша дал Коле столько орехов, сколько было у Коли. Сколько орехов было у мальчиков сначала, если теперь их стало поровну?

Решение 

  • 1) 8 : 2 = 4 (ореха) стало у каждого мальчика;
  • 2) 4 : 2 = 2 (ореха) Саша дал Коле;
  • 3) 4 + 2 = 6 (орехов) стало у Коли;
  • 4) 6 : 2 = 3 (ореха) Коля дал Саше;
  • 5) 6 – 3 = 3 (ореха) было у Саши;
  • 6) 8 – 3 = 5 (орехов) было у Коли.

Ответ: у Коли сначала было 5 орехов, у Саши 3.

Задача 6

Бабушка посчитала, что если дать внукам по 4 яблока, то 3 яблока останутся, а если дать по 5 яблок, то 2 яблока не хватит. Сколько внуков у бабушки?

Решение 

  • Если бабушка раздаст внукам по 4 яблока, то у нее останется еще 3 чтобы добавить 3 детям по одному яблоку. В этом случае трем детям достанется по 5 яблок, а так как по условию 2 яблока не хватает, если раздавать по 5, значит у бабушки еще 2 внука.

Ответ: у бабушки 5 внуков.

Задача 7

В детском саду на больших пирамидках по 7 колец, на маленьких по 5. Всего 20 пирамидок, на которых всего 128 колец. Сколько больших пирамидок в детском саду?

Решение 

  • 1) 20 * 5 = 100 (колец) если на каждой из пирамидок по 5 колец;
  • 2) 128 – 100 = 28 (колец) остаток;
  • 3) 7 – 5 = 2 (кольца) разница;
  • 4) 28 : 2 = 14 (пирамидок) по 7 колец;

Ответ: в детском саду 14 больших пирамидок.

Задача 8

В магазин привезли всего 20 двухколесных и трехколесных велосипедов. Сколько трехколесных и двухколесных велосипедов привезли по отдельности, если у всех велосипедов вместе 55 колес?

Решение 

  • 1) 20 * 3 = 60 (колес) если все велосипеды трехколесные;
  • 2) 60 – 55 = 5 (колес) разница;
  • 3) 20 – 5 = 15 (велосипедов).

Ответ: в магазин привезли 15 трехколесных велосипедов и 5 двухколесных.

Задача 9

Лена посчитала, что если каждый мальчик в классе принесет 5 рублей, а каждая девочка 3 рубля, то всего соберут 122 рубля. Сколько в классе девочек и сколько в классе мальчиков, если всего в классе 30 детей?

Решение 

  • 1) 30 * 3 = 90 (рублей) если бы каждый принес по 3 рубля;
  • 2) 122 – 90 = 32 (рубля) остаток;
  • 3) 5 – 3 = 2 (рубля) разница;
  • 4) 32 : 2 = 16 – мальчиков;
  • 5) 30 – 16 = 14 – девочек.

Ответ: в классе 16 мальчиков и 14 девочек.

Задача 10

Швейная фабрика закупила 138 метров черной и зеленой ткани, всего на 54000 рублей. Сколько метров каждой ткани по отдельности было закуплено. Если метр зеленой ткани стоит 500 рублей, а метр черной ткани 300 рублей?

Решение 

  • 1) 138 * 300 = 41400 (рублей) было бы потрачено, если бы вся ткань стоила 300 рублей за 1 метр;
  • 2) 54000 - 41400 = 12600 (рублей) остаток;
  • 3) 500 - 300 = 200 (рублей) разница в стоимости за метр;
  • 4) 12600 : 200 = 63 (м) ткани по 500 рублей за 1 метр;
  • 5) 138 - 63 = 75 (м) ткани по 300 рублей за 1 метр.

Ответ: ткани по 300 рублей за 1 метр, купили 75 м; ткани по 500 рублей за 1 метр, купили 63 м.


Предварительный просмотр:


Предварительный просмотр:

Задачи, решаемые с конца

Увлечение математикой часто начинается с размышлений над какой-то новой, интересной, нестандартной и понравившейся задачей. Она может встретиться и на школьном уроке, и на занятии математического кружка, в журнале или книге, ее можно услышать от друга или от родителей. Задачи на логику развивают в человеке сообразительность, интеллект и упорство в достижении цели. Очень часто одна решенная логическая задача пробуждает у ребенка устойчивый и долговременный интерес к изучению математики, желание искать и решать новые логические, нестандартные задачи и задачи повышенной трудности. А это, во многом, и есть главная цель учителя.

Логические задачи – это хороший способ развития умственных способностей. К классу логических задач относятся также задачи на переливания.

Задача 1

Отцу и сыну вместе 65 лет. Сын родился, когда отцу было 25 лет. Какого возраста отец и сын?

Решение

Так как сын родился тогда, когда отцу было 25 лет, то разница в их возрасте будет 25 лет. Тогда 65 – 25 = 40 (лет) – будет удвоенный возраст сына, а значит, сыну будет 20 лет, а отцу 45.

Задача 2

Одну овцу лев съел за 2 дня, волк за 3 дня, собака за шесть дней. За сколько дней они вместе съедят овцу?

Решение

  1. Так как лев съел овцу за 2 дня, то за 1 день он съел ½ овцы.
  2. Так как волк съел овцу за 3 дня, то за 1 день он съел 1/3 овцы.
  3. Так как собака съела овцу за 6 дней, то за 1 день она съела 1/6 овцы.
  4. Вместе лев, волк и собака за 1 день съедят ½+1/3+1/6=1, то есть 1 овцу.

Задача 3

Трое мальчиков имеют по некоторому количеству яблок. Первый мальчик дает другим столько яблок, сколько каждый из них имеет. Затем второй мальчик дает двум другим столько яблок, сколько каждый из них теперь имеет;  в свою очередь и третий дает каждому из двух других столько, сколько есть у каждого в этот момент. После этого у каждого из мальчиков оказывается по 8 яблок.

Сколько яблок было у каждого мальчика в начале?

Решение

Решаем задачу с конца с помощью таблицы.

НОМЕР МАЛЬЧИКА

1

2

3

Число яблок в конце

8

8

8

Число яблок до передачи их третьим мальчиком

8 : 2 = 4

8 : 2 = 4

8 + 4 + 4 = 16

Число яблок до передачи их вторым мальчиком

4 : 2 = 2

4 + 2 + 8 = 14

16 : 2 = 8

Число яблок первоначально

2 + 4 + 7 = 13

14 : 2 = 7

8 : 2 = 4

Таким образом, первоначально яблок у первого, второго и третьего мальчиков было соответственно  13, 7 и 4.

Разминка. Устные упражнения.

Одно из необходимых умений, которое важно для правильного решения текстовых задач,- это внимательное чтение условия   задачи.

  1. Вы – шофер автобуса. В автобусе первоначально было 23 пассажира. На первой остановке вышло 3 женщины и зашло 5 мужчин. На второй остановке зашло 4 мужчины и вышло 7 женщин.
    Сколько лет шоферу?
  2. Какое слово из 11 букв все отличники пишут неправильно?
  3. Продавая в магазине попугая, продавец пообещал, что попугай будет повторять каждое услышанное им слово.  Покупатель очень обрадовался, но придя домой, обнаружил, что попугай «нем как рыба». Тем не менее, продавец не лгал.
    Как это могло быть?
  4. Английский офицер, вернувшийся из Китая, заснул в церкви во время службы. Ему приснилось, что к нему приближается палач, чтобы отрубить ему голову, и тот самый момент, когда сабля опускалась на шею несчастного, его жена, желая разбудить заснувшего, слегка дотронулась до его шеи веером. Потрясение было столь велико, что офицер тут же умер.
    В этой истории, рассказанной вдовой офицера, что-то неладно. Что же именно?
  5. Петя решил купить Маше мороженое, но для его покупки ему не хватало 3 рублей, а Маше всего лишь 1 рубля. Тогда они решили сложить свои деньги, но опять не хватило 1 рубля на покупку даже одного мороженного.
    Сколько стоила порция мороженого?

Начинаем соревнование!

  1. Я задумал число, умножил его на два, прибавил три и получил 17.
    Какое число я задумал?
  2. Однажды черт предложил бездельнику заработать. «Как только ты перейдешь через этот мост, - сказал он, - твои деньги удвоятся. Можешь переходить по нему сколько хочешь раз, но после каждого перехода отдавай мне за это 24 копейки». Бездельник согласился и … после третьего перехода остался без гроша.
    Сколько денег у него было сначала?
  3. Группа туристов отправилась в поход. В первый день они прошли 1/3 пути, во второй - 1/3 остатка, в третий – 1/3 нового остатка. В результате им осталось пройти 32 км.
    Сколько километров был маршрут туристов?
  4. Играя в рулетку, Виктор удвоил количество своих денег, потом потерял 12 рублей. После этого у него осталось 60 рублей.
    С какой суммой он начинал игру?
  5. Над озерами летели гуси. На каждом озере садилась половина гусей и еще полгуся, остальные летели дальше. Все сели на семи озерах.
    Сколько было гусей?



Предварительный просмотр:

            Запись  цифр и чисел у разных народов

Сейчас в большинстве стран мира, несмотря на то, что там говорят на разных языках, считают одинаково, "по-арабски". Но так было не всегда. Еще каких-то пятьсот лет назад ничего подобного и в помине не было даже в просвещенной Европе, не говоря уже о какой-нибудь Африке или Америке. Но тем не менее числа люди все равно как-то записывали. У каждого народа была своя собственная или позаимствованная у соседа система записи чисел. Одни использовали буковки, другие - значки, третьи - закорючки. У кого-то получалось удобнее, у кого-то не очень.Ведь не так-то просто даже имея цифры (значки, которыми записываются числа), записать какое-нибудь число. Для этого нужна система счисления (способ записи чисел с помощью цифр). (Сразу хочу предупредить, что системы счисления бывают непозиционными и позиционными или аддитивными и мультипликативными Но не стоит пугаться таких грозных слов, на деле все очень просто).

Самая простая система счисления была еще у древних людей. Какое число нужно записать, столько сделают засечек на палке, или в кучку камешков положат. Но это удобно, пока числа небольшие. Вы только представьте себе число 1 000 записанное с помощью кучки камушков, а 1 000 000?. Неудобно?Тогда стали люди придумывать как по другому записывать большие числа. Для начала решили, что каждые 10 палочек заменять загогулинкой, и счет пошел легче! Так появилась аддитивная система счисления.Но люди никогда не стоят на месте, они постоянно чего-нибудь изобретают. Не захотелось людям вырисовывать по десятку палочек да загогулинок, и решили каждое круглое число обозначить по-особому. Но для этого потребовалось большое количество цифр-символов, и, чтобы не изобретать велосипед, решили использовать алфавит. Так и появилась на свет алфавитная аддитивная система счисления. Такая система очень долго использовалась по всей Европе, и во многих государствах за ее пределами.

Но далеко не все народы делали свои записи с помощью алфавита или слоговых знаков (об алфавитах и слоговых знаках здесь). В Китае иероглифы не позволили появиться такой системе счисления, и тогда ученые изобрели немного другую систему, названную мультипликативная система счисления. Эта система имела одно очень важное свойство: в ней одна и та же цифра, в зависимости от расположения в записи числа могла иметь разные значения. Именно такой системой счисления мы с Вами сейчас и пользуемся.

Здесь собраны наиболее известные нумерации мира:

 

 Древнеегипетская нумерация

 Древнегреческая нумерация

 Вавилонская нумерация

 Нумерация индейцев Майя

 Старо-Китайская нумерация

 Славянская кириллическая нумерация

 Славянская глаголическая нумерация

 Латинская нумерация

 Современная арабская нумерация

 

Самая простая система счисления

В этой системе счисления для записи чисел используется только одна цифра. Ее можно изобразить в виде , или любой другой фигуры. Числа будут, кружочка палочки  записываться примерно так:

1

 

2

  

3

   

4

    

5

     и т. д.

Такая система счисления использовалась, и до сих пор используется в основном народами, не имеющими письменности.

Но иногда такой системой счисления пользуются и современные люди, например, отмечая зарубками количество прошедших дней, или карандашом отмечая черточками в тетради количество проданных товаров.

Алфавитные аддитивные системы счисления

В этой системе счисления для записи чисел используется уже не несколько цифр, а большая часть алфавита. Все цифры здесь изображаются в точности так же, как и буквы алфавита того народа, который использовал эту систему.

Эта система счисления была очень распространена. Даже по сей день мы часто используем, например, римские цифры.

Такая система счисления уже годится для записи чисел, но она крайне неудобна для счета. Любое из четырех действий арифметики может вызвать затруднение. Для счета здесь нужна большая сноровка.

Вы только попробуйте разделить два вот таких числа:

XCIX и XXXIII

А ведь всего-то это 99 : 33.Удобств для счета, как мы видим ни каких. Такой системой счисления пользовались Римляне, Греки, Арабы, Евреи, Сирийцы, Славяне, Грузины.

Аддитивные системы счисления

В этой системе счисления для записи чисел используется уже не одна, а несколько цифр. Они могут изображаться так, как взбредет в голову, но только разные цифры должны выглядеть по-разному. Например в Египте единицы записывали палочками , а десяток палочек заменяли на изображение пут для коров, десяток пут - одна мерная веревка, и т. д. Для того, чтобы прочесть число, нужно было сложить значения всех цифр. Поэтому такие системы назвали аддитивными (add добавлять, складывать англ.).

 

 

1

2

3

4

9

10

11

Такая система счисления уже годится для записи чисел, но она крайне неудобна для счета.

Вы только попробуйте перемножить два вот таких числа:

И

 2026.Удо×А ведь всего-то это 1457 бств для счета, как мы видим ни каких. Такой системой счисления пользовались Египтяне, Ацтеки, племена Майя.

Системы счисления

Система счисления - очень сложное понятие. Оно включает в себя все законы, по которым числа записываются и читаются, а так же те, по которым производятся операции над ними.

Самое главное, что нужно знать о системе счисления - ее тип: аддитивная или мультипликативная. В первом типе каждая цифра имеет свое значение, и для прочтения числа нужно сложить все значения использованных цифр:

XXXV = 10+10+10+5 = 35; CCXIX = 100+100+10-1+10 = 219;

Во втором типе каждая цифра может иметь разные значения в зависимости от своего местоположения в числе:

(иероглифы по порядку: 2, 1000, 4, 100, 2, 10, 5)
Здесь дважды использован иероглиф "2", и в каждом случае он принимал разные значения "2000" и "20".

 

 10+5 = 2425× 100+2× 1000 + 4×2

Для аддитивной системы нужно знать все цифры-символы с их значениями (их бывает до 4 - 5 десятков), и порядок записи. Например, в Латинской записи если меньшая цифра записана перед большей, то производится вычитание, а если после, то сложение (IV = (5-1) = 4; VI = (5+1) = 6).

Для мультипликативной системы нужно знать изображение цифр и их значение, а так же основание системы счисления. Определить основание очень легко, нужно только пересчитать количество значащих цифр в системе. Если проще, то это число, с которого начинается второй разряд у числа. Мы, например, используем цифры 0, 1, 2, 3, 4, 5, 6, 7, 8, 9. Их ровно 10, поэтому основание нашей системы счисления тоже 10, и система счисления называется "десятичная". В вышеприведенном примере используются цифры 0, 1, 2, 3, 4, 5, 6, 7, 8, 9 (вспомогательные 10, 100, 1000, 10000 и т. д. не в счет). Основных цифр здесь тоже 10, и система счисления - десятичная.

Как можно догадаться, сколько есть чисел, столько же может быть и оснований систем счисления. Но используются только самые удобные основания систем счисления. Как вы думаете, почему основание самой употребительной человеческой системы счисления 10? Да, именно потому, что на руках у нас 10 пальцев. "Но на одной то руке всего пять пальцев" - скажут некоторые и будут правы. История человечества знает примеры пятеричных систем счисления. "А с ногами - двадцать пальцев" - скажут другие, и будут тоже абсолютно правы. Именно так считали индейцы Майя. Это даже видно по их цифрам.

Очень интересно понятие "дюжина". Всем известно, что это 12, но откуда появилось такое число - мало кто знает. Посмотрите на свои руки, вернее, на одну руку. Сколько фаланг на всех пальцах одной руки, не считая большого? Правильно, двенадцать. А большой палец предназначен отмечать отсчитанные фаланги.

А если на другой руке откладывать пальцами количество полных дюжин, то получим всем известную шестидесятеричную вавилонскую систему.

В разных цивилизациях считали по-разному, но и сейчас можно даже в языке, в названиях и изображениях цифр найти остатки совсем других систем счисления, когда-то использовавшихся этим народом.

Так у французов когда-то была двадцатеричная система счисления, поскольку 80 по-французски звучит как "четырежды двадцать".

Римляне, или их предшественники использовали когда-то пятеричную систему, так как V ни что иное, как изображение ладони с отставленным большим пальцем, а X - это две таких же руки.

Мультипликативные системы счисления

В таких системах счисления для записи чисел используется уже определенное количество цифр, которые могут принимать разные значения в зависимости от расположения в записи числа. Все цифры здесь изображаются определенными символами.

Например 0, 1, 2, 3, 4, 5, 6, 7, 8, 9, 10 11, 12, …, 99, 100, 101 …

Запись числа 1999 ×означает, что 1 1000 ×+ 9 100 ×+ 9 10 + 9. Для того, чтобы "собрать" такое число используется умножение (multiplication англ.), из-за чего систему и назвали "мультипликативной".

Такие системы счисления были только у народов с очень хорошо развитой математикой. По сей день мы используем только такую систему счисления.

Такая система счисления годится для записи чисел, и она очень удобна для счета. Любое из действий арифметики и алгебры может быть выполнено легко. Для счета здесь не нужна большая сноровка.

Впервые такая система, вернее ее зачатки появилась в Древнем Вавилоне, почти в то же время она была изобретена в Китае, потом в Индии, откуда перекочевала на Аравийский полуостров, а затем и в Европу. Здесь эту систему счисления назвали Арабской, и под этим именем она разошлась по всему миру. Так что, говоря "арабские числа" надо иметь в виду, ну, хотя бы индийские.

Египетская нумерация

Египтяне придумали эту систему около 5 000 лет тому назад. Это одна из древнейших систем записи чисел, известная человеку.

1. Как и большинство людей для счета небольшого количества предметов Египтяне использовали палочки.

Если палочек нужно изобразить несколько, то их изображали в два ряда, причем в нижнем должно быть столько же палочек сколько и в верхнем, или на одну больше.

10. Такими путами египтяне связывали коров

Если нужно изобразить несколько десятков, то иероглиф повторяли нужное количество раз. Тоже самое относится и к остальным иероглифам.

100. Это мерная веревка, которой измеряли земельные участки после разлива Нила.

1 000. Вы когда-нибудь видели цветущий лотос? Если нет, то вам никогда не понять, почему Египтяне присвоили такое значение изображению этого цветка.

10 000. "В больших числах будь внимателен!" - говорит поднятый вверх указательный палец.

100 000. Это головастик. Обычный лягушачий головастик.

1 000 000. Увидев такое число обычный человек очень удивится и возденет руки к небу. Это и изображает этот иероглиф

10 000 000. Египтяне поклонялись Амону Ра, богу Солнца, и, наверное, поэтому самое большое свое число они изобразили в виде восходящего солнца

Записывались цифры числа начиная с больших значений и заканчивая меньшими. Если десятков, единиц, или какого-то другого разряда не было, то переходили к следующему разряду.

- 1207, - 1 023 029

Попробуйте сложить эти два числа, зная, что более 9 одинаковых иероглифов использовать нельзя.

Новая, или арабская нумерация

Это, самая распространенная на сегодняшний день нумерация. Название "арабская" для нее не совсем верно, поскольку хоть и завезли ее в Европу из арабских стран, но там она тоже была не родной. Настоящая родина этой нумерации - Индия.

В различных районах Индии существовали разнообразные системы нумерации, но в какой-то момент среди них выделилась одна. В ней цифры имели вид начальных букв соответствующих числительных на древнеиндийском языке - санскрите, использующем алфавит "Деванагари".

0

1

2

3

4

5

6

7

8

9

Первоначально этими знаками представлялись числа 1, 2, 3, … 9, 10, 20, 30, …, 90, 100, 1000; с их помощью записывались другие числа. Но в последствии был введен особый знак - жирная точка, или кружок, для указания пустующего разряда; и нумерация "Деванагари" превратилась в поместную десятичную систему. Как и когда совершился такой переход - до сих пор неизвестно. К середине VIII века позиционная система нумерации получает широкое применение. В это же время она проникает в соседние страны: Индокитай, Китай, Тибет, Среднюю Азию.

Решающую роль в распространении индийской нумерации в арабских странах сыграло руководство, составленное в начале IX века Мухаммедом Аль Хорезми. Оно было переведено в Западной Европе на латинский язык в XII веке. В XIII веке индийская нумерация получает преобладание в Италии. В других странах она распространяется к XVI веку. Европейцы, заимствовав нумерацию у арабов, называли ее "арабской". Это исторически неправильное название удерживается и поныне.

Из арабского языка заимствовано и слово "цифра" (по-арабски "сыфр"), означающее буквально "пустое место" (перевод санскритского слова "сунья", имеющего тот же смысл). Это слово применялось для названия знака пустого разряда, и этот смысл сохраняло до XVIII века, хотя еще в XV веке появился латинский термин "нуль" (nullum - ничто).

Форма индийских цифр претерпевала многообразные изменения. Та форма, которой мы сейчас пользуемся установилась в XVI веке.

 

Древнее изображение десятичных цифр не случайно: каждая цифра обозначает число по количеству углов в ней. Например, 0 — углов нет, 1 — один угол, 2 — два угла и т.д. Написание десятичных цифр претерпело существенные изменения. Форма, которой мы пользуемся, установилась в XVI веке.

Латинская (Римская) нумерация

Это, наверное, самая известная нумерация, после арабской. С нею мы достаточно часто сталкиваемся в повседневной жизни. Это номера глав в книгах, указание века, числа на циферблате часов, и т. д.

Возникла эта нумерация в древнем Риме. Использовалась она для аддитивной алфавитной системы счисления

I

1

V

5

X

10

L

50

C

100

D

500

M

1 000

Прежде знак M изображался знаком Ф, потому то 500 и стал изображать знак D как "половина" Ф. Так же построена и пары L и C, X и V.

Записывались цифры числа начиная с больших значений и заканчивая меньшими, слева направо. Если цифра с меньшим значением записывалась перед цифрой с большим значением, то происходило ее вычитание.

CCXXXVII = 100+100+10+10+10+5+1+1 = 237

Но

XXXIX = 10+10+10-1+10 = 39

Есть правило, по которому нельзя записывать подряд 4 одинаковых цифры, такая комбинация заменяется комбинацией с правилом вычитания, например:

XXXX = XC (50-10)

IIII = IV (5-1)

CCCC = CD (500-100)

О происхождении римских цифр достоверных сведений нет. В римской нумерации явственно сказываются следы пятеричной системы счисления. В языке же римлян ни каких следов пятеричной системы нет. Значит, эти цифры были заимствованы римлянами у другого народа (скорее всего этрусков).

Такая нумерация преобладала в Италии до XIII века, а в других странах Западной Европы - до XVI века.

Славянская глаголическая нумерация

Эта нумерация была создана для переписки чисел в священных книгах западных славян. Использовалась она нечасто, но достаточно долго. По организации она в точности повторяет греческую нумерацию. Использовалась она с VIII по XIII в.

1

10

100

1 000

2

20

200

 

3

30

300

 

4

40

400

 

5

50

500

 

6

60

600

 

7

70

700

 

8

80

800

 

9

90

900

 

 

Записывались цифры числа начиная с больших значений и заканчивая меньшими, слева направо. Если десятков, единиц, или какого-то другого разряда не было, то его пропускали

Такая запись числа аддитивная, то есть в ней используется только сложение:

= 800+60+3 = 863

Для того, чтобы не перепутать буквы и цифры, использовались титла - горизонтальные черточки над числами, или точки.

Славянская кириллическая нумерация

Эта нумерация была создана вместе со славянской алфавитной системой для переписки священных книг для славян греческими монахами братьями Кириллом (Константином) и Мефодием в IX веке. Эта форма записи чисел получила большое распространение в связи с тем, что имела полное сходство с греческой записью чисел. Если посмотреть внимательно, то увидим, что после "а" идет буква "в", а не "б" как следует по славянскому алфавиту, то есть используются только буквы, которые есть в греческом алфавите. До XVII века эта форма записи чиcел была официальной на территории современной России, Белоруссии, Украины, Болгарии, Венгрии, Сербии и Хорватии. До сих пор православные церковные книги используют эту нумерацию.

Записывались цифры числа начиная с больших значений и заканчивая меньшими, слева направо. Если десятков, единиц, или какого-то другого разряда не было, то его пропускали. Интереснее всего записывались числа второго десятка:

четыре на десять

Читаем дословно "четырнадцать" - "четыре на десять". Как слышим, так и пишем: не 10+4, а 4+10, - четыре на десять. И так для всех чисел от 11 до 19. Таким образом у славян мы прослеживаем десятеричную систему счисления.

Запись числа, использованная славянами аддитивная, то есть в ней используется только сложение:

= 800+60+3

Для того, чтобы не перепутать буквы и цифры, использовались титла - горизонтальные черточки над числами, что мы видим на рисунке.

Для обозначения больших, чем 900 чисел использовались специальные значки, добавляемые к букве. Так образовывались числительные Тысяща - 1 000, Леон - 10 000, Одр - 100 000, Вран (ворон) - 1 000 000, Колода - 10 000 000, Тьма - 100 000 000.

Со словом "Тьма" связана поговорка "тьма-тьмущая", означающая немыслимо много. В "Слове о полку Игореве" мы встречаем фразу "орда покрыла вороновым крылом", которую можно истолковать как "побила большой силой", где "большой" можно сравнить с полумиллионом человек.

В России славянская нумерация сохранилась до конца XVII века. При Петре I возобладала так называемая "арабская нумерация"

Китайская нумерация

Эта нумерация одна из старейших и самых прогрессивных, поскольку в нее заложены такие же принципы, как и в современную арабскую, которой мы с Вами пользуемся. Возникла эта нумерация около 4 000 тысяч лет тому назад в Китае.

1

6

2

7

3

8

4

9

5

° 

0

 

Записывались цифры числа начиная с больших значений и заканчивая меньшими. Если десятков, единиц, или какого-то другого разряда не было, то сначала ничего не ставили и переходили к следующему разряду. (Во времена династии Мин был введен знак для пустого разряда - кружок - аналог нашего нуля). Чтобы не перепутать разряды использовали несколько служебных иероглифов, писавшихся после основного иероглифа, и показывающих какое значение принимает иероглиф-цифра в данном разряде.

10

100

1 000

 

однатысяча- 1 000; пятьсотенчетыредесяткавосемь- 548

Такая запись числа мультипликативна, то есть в ней используется умножение:

 10+8× 100+4× 1 000 и 5×1

Нумерация индейцев Майя

Эта нумерация очень интересна тем, что на ее развитие не повлеяла ни одна из цивилизаций Старого Света. Однако в ней использованы все те же принципы. Сначала эта нумерация обслуживала пятиричную систему счисления, а потом ее приспособили для двадцатиричной.

1

9

2

10

3

11

4

12

5

13

6

15

7

19

8

0 или 20

 

Записывались цифры числа в столбик, начиная со знаков , затем знаки , а потом больших значений и заканчивая меньшими.

59

16

23

 

20+20+5+5+5+1+1+1+1 = 59; 5+5+5+1 = 16; 20+1+1+1 = 23

Такая запись числа аддитивна, то есть в ней используется только сложение:

Вавилонская нумерация

В древнем Вавилоне примерно за 40 веков до нашего времени создалась позиционная нумерация, то есть такой способ записи чисел, при котором одна и та же цифра может обозначать разные числа, смотря по месту, занимаемому этой цифрой. Наша теперешняя нумерация тоже поместная. В вавилонской поместной нумерации ту роль, которую у нас играет число 10, играет число 60, и потому эту нумерацию называют шестидесятиричной. Числа менее 60 обозначались с помощью двух знаков: для единицы, и для десятка. Они имели клинообразный вид, так как вавилоняне писали на глиняных табличках палочками треугольной формы. Эти знаки повторялись нужное число раз, например

- 3; - 20; - 32

а это число 59.

Вавилонский способ обозначения чисел больше 60 очень похож на наш: В этом случае цифры записываются по разрядам, с небольшими пробелами между:

Так записывается число 302, то есть 2+06×5

 

А это 5273 = 5+06×2+06×06×1

При отсутствии разряда вставлялся значек , игравший роль нуля.

это запись числа )3+60×60×7203 (2

Однако отсутствие низшего разряда не обозначалось, и поэтому число 60×180 = 3 записывалось так , а обозначать эта запись могла и 3, и 180, и 10800 60)×60×(3, и т. д. Различать эти числа можно было только по смыслу текста.

Шестидесятеричная система счисления появилась у вавилонян позже десятеричной, ибо числа до 60 записываются в ней по десятичному принципу. Но до сих пор неизвестно, когда и как возникла у вавилонян шестидесятеричная система. На этот счет строилось множество гипотез, но ни одна не доказана.

Шестидесятеричная запись целых чисел не получила широкого распространения за пределами Ассиро-вавилонского царства, но шестидесятеричные дроби проникли далеко за эти пределы: Ближний Восток, Средняя Азия, Северная Африка, Западная Европа пользовались ими. Они широко применялись, особенно в астрономии, вплоть до изобретения десятичных дробей, т. е. До начала XVII века. Следы шестидесятеричных дробей сохраняются и поныне в делении углового и дугового градуса (а также часа) на 60 минут и минуты на 60 секунд.

Древняя греческая нумерация

В древнейшее время в Греции была распространена так называемая Аттическая нумерация. В этой нумерации числа 1, 2, 3, 4 изображались соответствующим количеством вертикальных полосок: ,,,. Число 5 записывалось знаком (древнее начертание буквы "Пи", с которой начиналось слово "пять" - "пенте". Числа 6, 7, 8, 9 обозначались сочетаниями этих знаков: .

Число 10 обозначалось - заглавной "Дельта" от слова "дека" - "десять". Числа 100, 1 000 и 10 000 обозначались H, X, M. Числа 50, 500, 5 000 обозначались комбинациями чисел 5 и 10, 5 и 100, 5 и 1 000, а именно: .

Числа в пределах первого десятка тысяч записывались так:

 

Примерно в третьем веке до нашей эры аттическая нумерация в Греции была вытеснена другой, так называемой "Ионийской" системой. В ней числа 1 - 9 обозначаются первыми буквами греческого алфавита:

числа 10, 20, … 90 изображались следующими девятью буквами:ѓ

числа 100, 200, … 900 последними девятью буквами:

Для обозначения тысяч и десятков тысяч пользовались теми же цифрами, но только с добавлением особого значка '. Любая буква с этим значком сразу же становилась в тысячу раз больше.

Для отличия цифр и букв писали черточки над цифрами.

Примерно по такому же принципу организованную систему счисления имели в древности евреи, арабы и многие другие народы Ближнего Востока.

Шестидесятеричная система счисления

            Особый интерес представляет так называемая "вавилонская " , или шестидесятеричная, система счисления, весьма сложная система, существовавшая в Древнем Вавилоне. Мнения историков по поводу того, как именно возникла эта система счисления, расходятся. Существуют две гипотезы. Первая исходит из того, что произошло слияние двух племён, одно из которых пользовалось шестеричной, другое - десятичной. Шестидесятеричная система счисления в данном случае могла возникнуть в результате своеобразного политического компромисса. Суть второй гипотезы в том, что древние вавилоняне считали продолжительность года равной 360 суткам, что естественно связано с числом 60. Отголоски использования этой системы счисления дошли до наших дней. Например, 1 час = 60 минут, 1 градус = 60 минут . В целом шестидесятеричная система счисления громоздка и неудобна.

 

Двенадцатеричная система счисления

        Довольно широкое распространение имела двенадцатеричная система счисления. Происхождение её тоже связано со счётом на пальцах. Считали большой палец руки и фаланги остальных четырёх пальцев: всего их 12. Элементы двенадцатеричной системы счисления сохранились в Англии в системе мер (1 фут = 12 дюймам) и в денежной системе (1 шиллинг = 12 пенсам). Нередко и мы сталкиваемся в быту с двенадцатеричной системой счисления: чайные и столовые сервизы на 12 персон, комплект носовых платков - 12 штук. 

20

У ацтеков и майя народов, населявших в течение многих столетий обширные области Американского континента и создавших там высочайшую культуру, в том числе и математическую, была принята двадцатеричная система счисления. Также двадцатеричная система счисления была принята и у кельтов, населявших Западную Европу начиная со второго тысячелетия до нашей эры. Основу для счета в этой системе счисления составляли пальцы рук и ног. Некоторые следы двадцатеричной системы счисления кельтов сохранились во французской денежной системе: основная денежная единица, франк, делится на 20 (1 франк = 20 су).

 

 



Предварительный просмотр:

Круги Эйлера – задачи на пересечение или объединение множеств


Это новый тип задач, в которых требуется найти некоторое пересечение множеств или их объединение, соблюдая условия задачи.
Круги Эйлера — геометрическая схема, с помощью которой можно изобразить отношения между подмножествами, для наглядного представления.
Метод Эйлера является незаменимым при решении некоторых задач, а также упрощает рассуждения. Однако, прежде чем приступить к решению задачи, нужно проанализировать условие. Иногда с помощью арифметических действий решить задачу легче.

"Обитаемый остров" и "Стиляги"

Некоторые ребята из нашего класса любят ходить в кино. Известно, что 15 ребят смотрели фильм «Обитаемый остров», 11 человек – фильм «Стиляги», из них 6 смотрели и «Обитаемый остров», и «Стиляги». Сколько человек смотрели только фильм «Стиляги»?
Решение 

ЛюбимыеЧертим два множества таким образом:

http://logika.vobrazovanie.ru/image/14.PNG


6 человек, которые смотрели фильмы «Обитаемый остров» и «Стиляги», помещаем в пересечение множеств.
15 – 6 = 9 – человек, которые смотрели только «Обитаемый остров».
11 – 6 = 5 – человек, которые смотрели только «Стиляги».
Получаем:

http://logika.vobrazovanie.ru/image/15.PNG


Ответ. 5 человек смотрели только «Стиляги». 

Любимые  мультфильмы

Среди школьников шестого класса проводилось анкетирование по любимым мультфильмам. Самыми популярными оказались три мультфильма: «Белоснежка и семь гномов», «Губка Боб Квадратные Штаны», «Волк и теленок». Всего в классе 38 человек. «Белоснежку и семь гномов» выбрали 21 ученик, среди которых трое назвали еще «Волк и теленок», шестеро – «Губка Боб Квадратные Штаны», а один написал все три мультфильма. Мультфильм «Волк и теленок» назвали 13 ребят, среди которых пятеро выбрали сразу два мультфильма. Сколько человек выбрали мультфильм «Губка Боб Квадратные Штаны»?
Решение В этой задаче 3 множества, из условий задачи видно, что все они пересекаются между собой. Получаем такой чертеж:

http://logika.vobrazovanie.ru/image/16.PNG


Учитывая условие, что среди ребят, которые назвали мультфильм «Волк и теленок» пятеро выбрали сразу два мультфильма, получаем:

http://logika.vobrazovanie.ru/image/17.PNG


21 – 3 – 6 – 1 = 11 – ребят выбрали только «Белоснежку и семь гномов».
13 – 3 – 1 – 2 = 7 – ребят смотрят только «Волк и теленок».
Получаем:

http://logika.vobrazovanie.ru/image/18.PNG


38 – (11 + 3 + 1 + 6 + 2 + 7) = 8 – человек смотрят только «Губка Боб Квадратные Штаны».
Делаем вывод, что «Губка Боб Квадратные Штаны» выбрали 8 + 2 + 1 + 6 = 17 человек.
Ответ. 17 человек выбрали мультфильм «Губка Боб Квадратные Штаны».

«Мир музыки»

В магазин «Мир музыки» пришло 35 покупателей. Из них 20 человек купили новый диск певицы Максим, 11 – диск Земфиры, 10 человек не купили ни одного диска. Сколько человек купили диски и Максим, и Земфиры?
Решение Изобразим эти множества на кругах Эйлера.

http://logika.vobrazovanie.ru/image/19.PNG


Теперь посчитаем: Всего внутри большого круга 35 покупателей, внутри двух меньших 35–10=25 покупателей. По условию задачи 20 покупателей купили новый диск певицы Максим, следовательно, 25 – 20 = 5 покупателей купили только диск Земфиры. А в задаче сказано, что 11 покупателей купили диск Земфиры, значит 11 – 5 = 6 покупателей купили диски и Максим, и Земфиры:

http://logika.vobrazovanie.ru/image/20.PNG


Ответ: 6 покупателей купили диски и Максим, и Земфиры.

Гарри Поттер, Рон и Гермиона

На полке стояло 26 волшебных книг по заклинаниям, все они были прочитаны. Из них 4 прочитал и Гарри Поттер, и Рон. Гермиона прочитала 7 книг, которых не читали ни Гарри Поттер, ни Рон, и две книги, которые читал Гарри Поттер. Всего Гарри Поттер прочитал 11 книг. Сколько книг прочитал только Рон?
Решение Учитывая условия задачи, чертеж будет таков:

http://logika.vobrazovanie.ru/image/21.PNG


Так как Гарри Поттер всего прочитал 11 книг, из них 4 книги читал Рон и 2 книги – Гермиона, то 11 – 4 – 2 = 5 – книг прочитал только Гарри. Следовательно,
26 – 7 – 2 – 5 – 4 = 8 – книг прочитал только Рон.
Ответ. 8 книг прочитал только Рон.

Пионерский лагерь

В пионерском лагере 70 ребят. Из них 27 занимаются в драмкружке, 32 поют в хоре, 22 увлекаются спортом. В драмкружке 10 ребят из хора, в хоре 6 спортсменов, в драмкружке 8 спортсменов; 3 спортсмена посещают и драмкружок и хор. Сколько ребят не поют, не увлекаются спортом, не занимаются в драмкружке? Сколько ребят заняты только спортом?
Решение Изобразим множества следующим образом:

http://logika.vobrazovanie.ru/image/22.PNG


70 – (6 + 8 + 10 + 3 + 13 + 6 + 5) = 19 – ребят не поют, не увлекаются спортом, не занимаются в драмкружке. Только спортом заняты 5 человек.
Ответ. 5 человек заняты только спортом.

Экстрим

Из 100 ребят, отправляющихся в детский оздоровительный лагерь, кататься на сноуборде умеют 30 ребят, на скейтборде – 28, на роликах – 42. На скейтборде и на сноуборде умеют кататься 8 ребят, на скейтборде и на роликах – 10, на сноуборде и на роликах – 5, а на всех трех – 3. Сколько ребят не умеют кататься ни на сноуборде, ни на скейтборде, ни на роликах?
Решение http://logika.vobrazovanie.ru/image/23.PNG

Всеми тремя спортивными снарядами владеют три человека, значит, в общей части кругов вписываем число 3. На скейтборде и на роликах умеют кататься 10 человек, а 3 из них катаются еще и на сноуборде. Следовательно, кататься только на скейтборде и на роликах умеют 10-3=7 ребят. Аналогично получаем, что только на скейтборде и на сноуборде умеют кататься 8-3=5 ребят, а только на сноуборде и на роликах 5-3=2 человека. Внесем эти данные в соответствующие части. Определим теперь, сколько человек умеют кататься только на одном спортивном снаряде. Кататься на сноуборде умеют 30 человек, но 5+3+2=10 из них владеют и другими снарядами, следовательно, только на сноуборде умеют кататься 20 ребят. Аналогично получаем, что только на скейтборде умеют кататься 13 ребят, а только на роликах – 30 ребят. По условию задачи всего 100 ребят. 20+13+30+5+7+2+3=80 – ребят умеют кататься хотя бы на одном спортивном снаряде. Следовательно, 20 человек не умеют кататься ни на одном спортивном снаряде.
Ответ. 20 человек не умеют кататься ни на одном спортивном снаряде.



Предварительный просмотр:

Магические квадраты

Одно из самых загадочных произведений изобразительного искусства хранится в Кунстхалле города Карлсруэ. Речь о гравюре Альбрехта Дюрера «Меланхолия I» (1514).

Значимая деталь, изображенная на гравюре «Меланхолия I» – составленный впервые в европейском искусстве магический квадрат 4 Х 4. Сумма чисел в любой строке или столбце равна 34. Два средних числа в нижнем ряду указывают дату создания картины 1514 год.

Размерность квадрата 4*4. Он заполнен числами от 1 до 4*4(16) таким образом, что сумма чисел на любой горизонтали, вертикали и диагонали равна 34. Эта сумма также встречается во всех угловых квадратах 2×2, в центральном квадрате (10+11+6+7), в квадрате из угловых клеток (16+13+4+1), в квадратах, построенных «ходом коня» (2+8+9+15 и 3+5+12+14), в прямоугольниках, образованных парами средних клеток на противоположных сторонах (3+2+15+14 и 5+8+9+12).

Магические квадраты - это таблицы чисел, в которых суммы чисел в каждой строке, в каждом столбце и в каждой из двух диагоналей квадрата все равны между собой.

Магические квадраты были известны еще арабам, к которым вероятно, они перешли от индусов; затем они сделались достоянием математиков восточной части Римской империи и, наконец, появились в Западной Европе, где методами получения магических квадратов заинтересовались многие ученые. В средние века люди верили в магическую силу этих квадратов. Они использовались для изготовления талисманов, оберегающих от различных болезней.

Из всякого магического квадрата путем различных перестановок составляющих его чисел можно получить множество новых магических квадратов, обладающих теми же свойствами.

Известно, что магических квадратов 2х2 не существует (может быть, кто-нибудь это докажет?).

Магический квадрат 3х3 только один.

Магических квадратов 4х4, как на картине Дюрера, составлено уже 800, а количество магических квадратов 5х5 близко к четверти миллиона!

Магический квадрат – древнекитайского происхождения. Согласно легенде, во времена правления императора Ю (ок. 2200 до н.э.) из вод Хуанхэ (Желтой реки) всплыла священная черепаха, на панцире которой были начертаны таинственные иероглифы, и эти знаки известны под названием ло-шу. В 11 в. о магических квадратах узнали в Индии, а затем в Японии, где в 16 в. магическим квадратам была посвящена обширная литература. Европейцев с магическими квадратами познакомил в 15 в. византийский писатель Э.Мосхопулос. Первым квадратом, придуманным европейцем, считается квадрат А.Дюрера, изображенный на его знаменитой гравюре Меланхолия I.

Каждый элемент магического квадрата называется клеткой. Квадрат, сторона которого состоит из n клеток, содержит n² клеток и называется квадратом n-го порядка. В 16 в. Корнелий Генрих Агриппа построил квадраты 3-го, 4-го, 5-го, 6-го, 7-го, 8-го и 9-го порядков, которые были связаны с астрологией 7 планет. В 19 и 20 вв. интерес к магическим квадратам вспыхнул с новой силой. Их стали исследовать с помощью методов высшей алгебры и операционного исчисления.

Бытовало поверье, что выгравированный на серебре магический квадрат защищает от чумы. Магическим квадратам приписывали различные мистические свойства.  Даже сегодня среди атрибутов европейских прорицателей можно увидеть магические квадраты.

рис
Альбрехт Дюрер,
"Меланхолия "
рис

Квадрат Дюрера

рис

Рассмотрим удобный способ заполнения магического квадрата 3-го порядка. Наш квадрат разделен на 9 равных клеток. Необходимо расставить в этих клетках числа 1, 2, 3, 4, 5, 6, 7, 8, 9 так, чтобы сумма чисел в каждой строке и в каждом столбике равнялась 15.

рисрис рис рис рис
Рис. 1.                    Рис. 2.                     Рис. 3.                    Рис. 4.                    Рис. 5.

[Нажмите и удерживайте для перемещения. Вы можете раскрыть сразу несколько изображений.]


1. Добавим «крылышки» в средний столбец и в среднюю строку.
2. Выделим по диагоналям клетки, которые мы заполним числами.
3. Запишем в выделенные клетки числа от 1 до 9.
4. Перенесем числа из «крылышек» во внутреннюю часть квадрата, как показано на рисунках 3, 4, 5.
    Анимационный вариант решения в презентации.

рис

3

5

7

9

11

13

15

17

19

Используя рассмотренный алгоритм, можно решать занимательные задачи с магическими квадратами.


1. В клетках квадрата переставьте числа так, чтобы по любой вертикали, горизонтали и диагонали их суммы были равны между собой (рис. 6). Решение в презентации.

2. Даны числа: 5, 10, 15, 20, 25, 30, 35, 40, 45. Впишите их в клетки девятиклеточного квадрата так, чтобы получилось в сумме одно и то же число по любой вертикали, горизонтали и диагонали. Решение в презентации.

3. Разместите в свободных клетках квадрата еще числа 3, 4, 5, 6, 7, 8, 9 так, чтобы по любой вертикали, горизонтали и диагонали получилось в сумме одно и то же число (рис. 7).

 

рис. 6.

10

    

7

 

11

рис. 7.




Предварительный просмотр:

Математические ребусы

Математическими ребусами называют задания на восстановление записей вычислений. Условие математического ребуса содержит либо целиком зашифрованную запись (цифры заменены буквами), либо только часть записи(стертые цифры заменены точками).

Записи восстанавливаются на основании логических рассуждений. При этом нельзя ограничиваться отысканием только одного решения. Испытание нужно доводить до конца, чтобы убедиться, что нет других решений, или найти все решения. Есть математические ребусы, имеющие несколько решений.

Примеры решения

Восстановите поврежденные записи арифметических действий

Вариант А

**
+
*
-----
**8

Вариант Б

**
+
**
-----
*98

 

Рассматривая данную разновидность ребусов, нужно обратить свое внимание на то , что сумма двузначного и однозначного чисел является трехзначным числом, поэтому первая цифра в сумме будет 1. А число 1*8 может получиться только в сумме наибольшего двузначного числа и наибольшего однозначного. Аналогично во втором случае, сумма равна 198. А так как слагаемые двузначные числа и самое большое двузначное число будет 99, то решением будет 99 + 99 = 198.

Решите ребусы.

Вариант А

ДРАМА
+
ДРАМА
---------
ТЕАТР

Вариант Б

КОШКА
+
КОШКА
+
КОШКА
-----------
СОБАКА

Вариант C

ЧАЙ : АЙ = 5

 

Решение

Вариант А. Очевидно, Д < 4. В разряде тысяч имеем А + А = А, значит, А = 0 (без перехода) или А = 9(с переходом). Значение А = 0 не подходит, так как в разряде единиц  А + А =Р (получаем А = Р = 0). Значит, А = 9, Р= 8, Е = 7. Тогда 2М + 1 =10 + Т, Т < 9, значит, М = 5 или 6 (так как получается переход), а значения 7 и 8 уже заняты буквами Е и Р. При М = 6 получается решение:

18969
+
18969
---------
37938

 

Вариант Б. Так как КА +КА +КА оканчивается на КА, то КА = 50, а значит, К = 5, А = 0. Так как Ш + Ш + Ш + 1 оканчивается на 0, то Ш = 3. Так как сумма трех чисел, начинающихся на 5, может начинаться лишь с 1, то  С = 1. Рассматривая варианты для О, получаем, что О = 6 или О = 7, а значит, Б = 9 или Б = 2. Итак, получаем два варианта решения:

56350
+
56350
+
56350
----------
169050

57350
+
57350
+
57350
----------
172050

 

Вариант В. Этот пример является наиболее трудным. Для его решения лучше перейти от деления к умножению: 5*АЙ = ЧАЙ, значит Ч*100 + АЙ =АЙ*5 и тогда Ч*25 = АЙ. Так как АЙ – двузначное, то Ч = 1,2,3. Для каждого Ч находим решение: 125, 250, 375. Итак, получаем три решения:

125 : 25 = 5

250 : 50 = 5

375 : 75 = 5

 

Разминка

  1. Дайте добрый совет! Президент страны решил уволить своего премьер-министра, но не хотел его обижать, да и особого повода не было. Наконец он придумал вот что. Когда премьер-министр пришел к президенту, тот сказал ему: «Я положил в портфель 2 листа бумаги. На одном написано: «Останьтесь», на другом – «Уходите». Листок, который Вы, не глядя, вынете из портфеля, решит Вашу судьбу». Хитрый премьер-министр догадался, что на обоих листах написано «Уходите».
    Как ему избежать отставки?
  2. Два разбойника делят добычу. Каждый уверен, что мог бы поделить добычу на 2 равные части, но второй ему не доверяет.
    Как разбойникам разделить добычу, чтобы оба остались довольны?
  3. Можно ли в тетрадном листе прорезать дырку так, чтобы свозь нее мог пролезть любой из вас?

Соревнование

Задание 1.

Решите ребус:

6*
×
***
-----
**

Решите ребус:

**
+
**
-----
***6

 

Задание 2.

Решите ребус:

А
+
ББ
+
А
------
ССС

 

Задание 3.

Решите ребус:

СПОРТ
+
СПОРТ
+
СПОРТ
----------
КРОСС



Предварительный просмотр:

Математические фокусы

Введение

Обучаясь фокусам, человек развивает в себе артистизм, творческий потенциал. Математические фокусы нацеливают внимание детей на урок математики, благодаря развлекающей сути фокуса в сочетании с математической природой секрета (однажды показав фокус, ребенка можно стимулировать к активным действиям на уроке под предлогом раскрытия секрета). Вся суть при просмотре фокуса состоит в поиске отгадки и получением удовольствия от «магических действий».

Существует много математических фокусов различных видов, мы приведем лишь несколько примеров.

Фокусы:

День недели на ладони


Пронумеруем каждый день недели (понедельник – 1, вторник – 2 и т.д.). Любой ученик может загадать один из дней (число от 1 до 7), учитель предлагает умножить загаданное число на 2, затем прибавить 5, сумму умножить на 5, в конце приписать нуль. Классу сообщается результат, из которого вычитается 250. В итоге количество сотен будет соответствовать загаданному дню

Секрет фокуса: Подставим вместо номера дня «х»:

((2х+5)*5)*10=(10х+25)*10=100х+250

100х+250-250=100х. Следовательно, количество сотен всегда соответствует номеру дня.

Примечание: Фокусы подобного вида – самые распространенные из всех математических фокусов, поэтому не стоит заполнять мероприятие только ими.

Феноменальная память

Учитель пишет на листке очень длинный числовой ряд (22—26 чисел) и заявляет, что сможет по памяти перечислить все числа в ряду в том же порядке. Выполнив, можно повторить фокус, чтобы доказать, что числовой ряд абсолютно произвольный (в нем действительно не должно быть никакой закономерности).

Секрет фокуса: Все числа в ряду - всего-навсего хорошо знакомые номера телефонов (можно брать последние 4-7 чисел от каждого номера).

Примечание: Как видно из примера, в некоторых математических фокусах используется обыкновенная хитрость.

Интуиция, или магическая девятка

Один ученик (или все сразу) пишет число из 3 разных цифр, а рядом — число из этих же цифр, но в обратном порядке. Из большого числа вычитается меньшее. Не видя результата, учитель говорит, что в середине полученного ответа стоит девять (если в ответе двузначное число – то записать его в виде 0…). И действительно, девятка стоит, где и было предсказано учителем.

Секрет фокуса: Поскольку меняются местами только 1 и 3 цифры, то у большего числа, цифра в разряде единиц всегда будет меньше, значит, из разряда десятков нужно будет занять 1, а когда нужно будет вычитать десятки – из разряда сотен (чтобы понять – попробуйте решить столбиком). Например, 653-356=297.

Примечание: Секреты самых интересных математические фокусы обычно нельзя с первого взгляда угадать, а сам фокус сложно отнести к какой-либо подгруппе.

Число в конверте

Фокусник пишет на бумажке число 1089, вкладывает бумажку в конверт и заклеивает его. Предлагает кому-нибудь, дав ему этот конверт, написать на нем трехзначное число такое, чтобы крайние цифры в нем были различны и отличались бы друг от друга больше, чем на 1. Пусть затем он поменяет местами крайние цифры и вычтет из большего трехзначного числа меньшее. В результате пусть он снова переставит крайние цифры и получившееся трехзначное число прибавит к разности двух первых. Когда он получит сумму, фокусник предлагает ему вскрыть конверт. Там он найдет бумажку с числом 1089, которое у него и получилось.

Опять и опять ПЯТЬ

Простенький и коротенький фокус, где фокуснику даже не надо ничего считать и думать.
Просите задумать любое число (хоть 50-тизначное), затем просите прибавить к нему следующее по порядку, затем пусть прибавит к сумме 9, разделит полученное пополам, и вычесть из результата задуманное им число. Вы легко называете число которое у него получилось!

Секрет фокуса:

Вы легко угадываете сколько у него получилось, потому что какое бы он число не загадал после всех подсчетов у него всегда будет 5.
Пример: загадали 25 25+26=51, 51+9=60, 60/2=30, 30-25=5 загадали 564 564+565=1129, 1129 + 9 = 1138, 1138/2=569, 569-564=5 загадали 444444 444444+444445=888889, 888889+9=888898, 888898/2=444449, 444449-444444=5

Сумма нечетных

Просите зрителя за 1 минуту посчитать сумму всех нечетных чисел от 0 до 20 (без калькулятора). Скорее всего он не успеет. Говорите:
- Ну ты и черепаха, попробуй еще раз , только поживей, какова сумма нечетных от 0 до 45 включительно?
Скорее всего зритель отмахнется, мол я до 20 то не смог, а тут до 45 (тут уже можно и с калькулятором, но опять таки сделать ограничено во времени чтоб он ну никак не успел). Вы же легко считаете сумму всех нечетных, даже многозначных чисел (только кто проверять будет)

Секрет фокуса:

Нужно к последнему (заданному) нечетному числу прибавить 1, поделить на 2 и возвести в квадрат.
Пример: от 1 до 49 включительно 49+1=50, 50/2 = 25, 25*25 =625.
Если вас попросят сосчитать уж очень большое число, то вам придется таки воспользоваться калькулятором, но т.к. считать очень мало, вы это сделаете за 10-15 сек.

Все дороги ведут к нулю

Ученик загадывает двузначное число, выполняет определённые действия и в итоге у него получается ноль.

Секрет фокуса:
Ученик  загадывает любое двузначное число. к примеру 45. Затем он должен поменять цифры местами, получится 54. Полученный результат записывается 4 раза подряд. 54545454 ученик убирает 1-ю и последнюю цифры этого числа 454545. Полученное число умножается на 3. В данном случае ответ 1363635. Полученное число делим на 7 (получается 194805). Это число делим на 9 (получается 21645). Делим число на 13 (получается 1665). Полученное число делим на первоначально задуманное (45) ответ 37. Обратите внимание, что 37 получается всегда при любых первоначально загаданных числах. Итак для получения 0 Вам остается навычитать любыми вариантами 37.
Этот фокус может удивить даже сильных математиков.



Предварительный просмотр:

Приёмы быстрого счёта

  1.      Первая литература по способам счёта.
  2. Таблица умножения на пальцах.
  3. Люди – феномены быстрого счёта.
  4. Умножение на 11 числа, сумма цифр которого меньше 10.
  5. Умножение на 11 числа, сумма цифр которого 10 или больше 10.
  6.    Умножение на 11 (по Трахтенбергу).
  7. Умножение на 12 (по Трахтенбергу).
  8. Умножение на 111, 1111, 11111 и т.д.
  9. Умножение на 101.
  10. Умножение на 999.
  11. Умножение на 11 (по Берману).
  12. Умножение на 12 (по Берману).
  13. 13.Умножение на 6 (по Трахтенбергу).

1. Первая литература по способам счёта.

В книге В. Беллюстина « Как постепенно дошли люди до настоящей арифметики» (1914) изложено 27 способов умножения, причем автор замечает: «весьма возможно, что есть и еще (способы), скрытые в тайниках книгохранилищ, разбросанные в многочисленных, главным образом рукописных сборниках».Наш современный способ умножения описан там под названием «шахматного». Был так же и очень интересный, точный, лёгкий, но громоздкий способ «галерой» или «лодкой», названный так в силу того, что при делении чисел этим способом получается фигура, похожая на лодку или галеру. У нас такой способ употреблялся до середины XVIII века. («Арифметика» – старинный русский учебник математики, которую Ломоносов назвал «вратами своей учености») пользуется исключительно способом «галеры», не употребляя, впрочем, этого названия.

Упоминаются такие способы, как «загибанием», «решеткой», «задом наперед», «ромбом», «треугольником» и многие другие. Многие такие приемы для умножения чисел долгие и требуют обязательной проверки.

Интересно, что и наш способ умножения не является совершенным, можно придумать еще более быстрые и еще более надежные.

2. Таблица умножения на «пальцах».

Таблица умножения – те необходимые в жизни каждого человека знания, которые требуется элементарно заучить, что на первых школьных порах даётся совсем не элементарно. Это потом уже с легкостью мага мы «щелкаем» примеры на умножение: 2·3, 3·5, 4·6 и т.д., но со временем все чаще забываемся на множителях ближе к 9, особенно если счетной практики давно не ведали, отчего отдаемся во власть калькулятора или надеемся на свежесть знаний друга. Однако, овладев одной незамысловатой техникой «ручного» умножения, мы можем запросто отказаться от услуг калькулятора. Уточнение: речь идет о школьной таблице умножения, т.е. для чисел от 2 до 9, умножаемых на числа от 1 до 10.

Умножение для числа 9 – 9·1, 9·2 … 9·10 – легче выветривается из памяти и труднее пересчитывается вручную методом сложения, однако именно для числа 9 умножение легко воспроизводится» на пальцах». Растопырьте пальцы на обеих руках и поверните руки ладонями от себя. Мысленно присвойте пальцам последовательно числа от 1 до 10, начиная с мизинца левой руки и заканчивая мизинцем правой руки (это изображено на рисунке). Допустим, хотим умножить 9 на 7.  Загибаем палец с номером, равным числу, на которое мы будем умножать 9. В нашем примере нужно загнуть палец с номером 7. Количество пальцев слева от загнутого пальца показывает нам количество десятков в ответе, количество пальцев справа – количество единиц. Слева у нас 6 пальцев не загнуто, справа – 3 пальца. Таким образом, 9·7=63. Ниже на рисунке детально показан весь принцип «вычисления».

Еще пример: нужно вычислить 9·9=? По ходу дела скажем, что в качестве «счетной машинки» не обязательно могут выступать пальцы рук. Возьмите к примеру 10 клеточек в тетради. Зачеркиваем 9-ю клеточку. Слева осталось 8 клеточек, справа – 1 клеточка. Значит 9·9=81. Все очень просто.

Умножение для числа 8 – 8·1, 8·2 … 8·10 – действия здесь похожи на умножение для числа 9 за некоторыми изменениями. Во-первых, поскольку числу 8 не хватает уже двойки до круглого числа 10, нам необходимо каждый раз загибать сразу два пальца – с номером х и следующий палец с номером х+1. Во-вторых, тотчас же после загнутых пальцев мы должны загнуть еще столько пальцев, сколько осталось не загнутых пальцев слева. В-третьих, это напрямую работает при умножении на число от 1 до 5, а при умножении на число от 6 до 10 нужно отнять от числа х пятерку и выполнить расчёт как для числа от 1 до 5., а к ответу затем добавить число 40, потому что иначе придется выполнять переход через десяток, что не совсем удобно «на пальцах», хотя в принципе это не так сложно. Вообще надо заметить, что умножение для чисел ниже 9 тем неудобнее выполнять «на пальцах», чем ниже число расположено от 9.

Теперь рассмотрим пример умножения для числа 8. Допустим, хотим умножить 8 на 3. Загибаем палец с номером 3 и за ним палец с номером 4 (3+1). Слева у нас осталось 2 незагнутых пальца, значит нам необходимо загнуть еще 2 пальца после пальца с номером 4 (это будут пальцы с номерами 5, 6 и 7). Осталось 2 пальца не загнуто слева и 4 пальца – справа. Следовательно, 8·3=24.

Еще пример: вычислить 8·8=?  Как было сказано выше, при умножении на число от 6 до 10 нужно отнять от числа х пятерку, выполнить расчет с новым число х-5, а затем добавить к ответу число 40. У нас х=8, значит загибаем палец с номером 3 (8-5=3) и следующий палец с номером 4 (3+1). Слева два пальца остались не загнуты, значит загибаем еще два пальца (с номером 5,6). Получаем: слева 2 пальца не загнуты и справа – 4 пальца, что обозначает число 24. Но к этому числу нужно еще добавить 40: 24+40=64. В итоге 8·8=64.

3. Люди – феномен быстрого счёта.

Феномен особых способностей в устном счёте встречается с давних пор. Как известно, ими обладали многие ученые, в частности Андре Ампер и Карл Гаусс. Однако, умение быстро считать было присуще и многим людям, чья профессия была далека от математики и науки в целом.

До второй половины XX века на эстраде были популярны выступления специалистов в устном счёте. Иногда они устраивали показательные соревнования между собой. Известными российскими «суперсчетчиками» являются Арон Чиквашвили, Давид Гольдштейн, Юрий Горный, зарубежными – Борислав Гаджански, Вильям Клайн, Томас Фулер и другие.

Хотя некоторые специалисты уверяли, что дело во врожденных способностях, другие аргументировано доказывали обратное: «дело не только и не столько в каких-то исключительных «феноменальных» способностях, а в знании некоторых математических законов, позволяющих быстро производить вычисления» и охотно раскрывали эти законы.

Истина как обычно, оказалась на некоей «золотой середине» сочетания природных способностей и грамотного, трудолюбивого их пробуждения, взращивания и использования. Те, кто следуя Трофиму Лысенко уповают исключительно на волю и напористость, со всеми уже хорошо известными способами и приемами устного счёта обычно при всех стараниях не поднимаются выше очень и очень средних достижений. Более того, настойчивые попытки «хорошенько нагрузить» мозг такими занятиями как устный счёт, шахматы вслепую и т.п. легко могут привести к перенапряжению и заметному падению умственной работоспособности, памяти и самочувствия (а в наиболее тяжелых случаях – и к шизофрении). С другой стороны и одаренные люди при беспорядочном использовании своих талантов в такой области как устный счёт быстро «перегорают» и перестают быть в состоянии длительно и устойчиво показывать яркие достижения. Один из примеров удачного сочетания обоих условий (природной одаренности и большой грамотной работы над собой) показал наш соотечественник, уроженец Алтайского края Юрий Горный.

Пожалуй, единственная научно обоснованная и достаточно подробно разработанная система резкого повышения быстроты устного счёта создана была в годы второй мировой войны цюрихским профессором математики Я. Трахтенбергом.  Она известна под названием «Система быстрого счёта». История ее создания необычная. В 1941г. гитлеровцы бросили Трахтенберга в концлагерь. Чтобы уцелеть в нечеловеческих условиях и сохранить нормальной свою психику, Трахтенберг начал разрабатывать принципы ускоренного счета. За четыре страшных года пребывания в концлагере профессору удалось создать стройную систему ускоренного обучения детей и взрослых основам быстрого счёта. Уже с самого начала результаты были самые отрадные. Учащиеся радовались вновь приобретенным навыкам и с воодушевлением двигались вперед. Если раньше их отталкивала монотонность, то сейчас их привлекало разнообразие приёмов. Шаг за шагом, благодаря достигнутым ими успехам, рос интерес к занятиям. После войны Трахтенберг создал и возглавил Цюрихский математический институт, получивший мировую известность.

Также разработкой приёмов быстрого счёта занимались другие ученые: Яков Исидорович Перельман, Георгий Берман и другие.

4.  Умножение на 11 числа, сумма цифр которого не превышает 10.

Чтобы умножить на 11 число, сумма цифр которого 10 или меньше 10, надо мысленно раздвинуть цифры этого числа, поставить между ними сумму этих цифр, а затем к первой цифре прибавить 1, а вторую и последнюю (третью) цифру оставить без изменения.

72х11=7(7+2)2=792;

35х11=3(3+5)5=385;

5.  Умножение на 11 числа, сумма цифр которого больше 10.

Чтобы умножить на 11 число, сумма цифр которого 10 или больше 10, надо мысленно раздвинуть цифры этого числа, поставить между ними сумму этих цифр, а затем к первой цифре прибавить 1, а вторую и последнюю (третью) цифру оставить без изменения.

78х11=7(7+8)8=7(15)8=858;

94х11=9(9+4)4=9(13)4=1034;

6.  Умножение на одиннадцать (по Трахтенбергу).

Разберем на примере: 633 умножить на 11.

Ответ пишется под 633 по одной цифре справа налево, как указано в правилах.

Первое правило. Напишите последнюю цифру числа 633 в качестве правой цифры результата

633*11

3

Второе правило. Каждая последующая цифра числа 633 складывается со своим правым соседом и записывается в результат.3+3 будет 6. Перед тройкой записываем результат 6.

633*11

63

Применим правило еще раз: 6+3 будет 9. Записываем и эту цифру в результате:

633*11

963

Третье правило. Первая цифра числа 633, то есть 6, становится левой цифрой результата:

633*11

6963

Ответ: 6963.

7.  Умножение на двенадцать (по Трахтенбергу).

Правило умножения на 12: нужно удваивать поочередно каждую цифру и прибавлять к ней поочередно ее «соседа».

Пример: 63247*12

Необходимо записывать цифры множимого через интервал и каждую цифру результата писать точно под цифрой числа 63247, из которой она образовалась.

063247*12 дважды 7 будет = 14, переносим 1

4

063247*12 дважды 4+7+1=16, переносим 1

64

063247*12 дважды 2+4+1 = 9

964

Следующие шаги аналогичны.

Окончательный ответ: 063247*12

758964

8.  Умножение на число 111, 1111 и т. д., зная правила умножения двузначного числа на число 11.

Если сумма цифр первого множителя меньше 10, надо мысленно раздвинуть цифры этого числа на 2, 3 и т.д. шага, сложить цифры и записать соответствующее количество раз их сумму между раздвинутыми цифрами. Количество шагов всегда меньше количества единиц на 1.

Пример:

24х111=2(2+4) (2+4)4=2664 (количество шагов - 2)

24х1111=2(2+4)(2+4)(2+4)4=26664 (количество шагов - 3)

При умножении числа 72 на 111111 цифры 7 и 2 надо раздвинуть на 5 шагов. Эти вычисления можно легко произвести в уме.

72 х 111111 = 7999992 (количество шагов – 5)

Если единиц во втором множителе 7, то шагов будет на один меньше, т.е. 6.

Если единиц 8, то шагов будет 7 и т.д.

61 х 11111111 = 677777771

Эти вычисления можно легко произвести в уме.

Умножение двузначного числа на 111, 1111, 1111 и т.д., сумма цифр которого равна или больше 10.

Немного сложнее выполнить устное умножение, если сумма цифр первого множителя равна 10 или более 10.

Примеры:

48 х 111 = 4 (4+8) (4+8) = 4 (12) (12) 8 = (4+1) (2+1) 28 = 5328.

В этом случае к первой цифре нужно прибавить 1. получим 5.

Далее 2 + 1 = 3. А последние цифры 2 и 8 оставляем без изменения.

56 х 11111 = 5 (5+6) (5+6) (5+6) (5+6) 6 = 5 (11) (11) (11) (11) 6 = 622216

67 х 1111 = 6 (6+7)…7 = 6 (13)…7 = 74437

9.  Умножение двузначного числа на 101.

Пожалуй, самое простое правило: припишите ваше число к самому себе. Умножение закончено. Пример:

57 * 101 = 5757 57 5757                94 * 101 = 9494

быстрый счёт умножение число    59 * 101 = 5959

10.  Умножение трёхзначного числа на 999.

Любопытная особенность числа 999 проявляется при умножении на него всякого другого трёхзначного числа. Тогда получается шестизначное произведение: первые три цифры есть умножаемое число, только на уменьшенное на единицу, а остальные три цифры (кроме последней) – «дополнения» первых до 9. Например:

385 * 999 = 384615

573 * 999 = 572427                           943 * 999 = 942057

11.  Умножение по одиннадцать, число нужно умножить на 10 и прибавить то число, которое мы умножаем.

Пример:  110 * 11 = 110 * (10+1) = 110 * 10 + 110 * 1= 1100 + 110= 1210

Ответ: 1210

Пример: 123 * 11 = 123 * (10+1) = 123 * 10 + 123 * 1= 1230 + 123= 1353

Ответ: 1353.

12. Умножение на двенадцать (по Берману).

При умножении на 12 можно число умножить сначала на 6, а затем на 2. Шесть в свою очередь, можно разбить на 2 множителя – это 3 и 2.

Пример: 136 * 12 = 136* 6 * 2 = 816 * 2 = 1632 или

136 * 12 = 136 * 3 * 2 * 2 = 408 * 2 * 2 = 816 * 2 = 1632

13.  Умножение на шесть ( по Трахтенбергу)

Нужно прибавить к каждой цифре половину «соседа».

Пример: 0622084 * 6

0622084 * 6  4 является правой цифрой этого числа и, так 4 как «соседа» у неё нет, прибавлять нечего.

06222084 * 6  Вторая цифра  8, е «сосед» - 4. Мы берём 8 04 прибавляем половину 4 (2) и получаем 10, ноль пишем, 1 в перенос.

06222084 * 6  Следующая цифра ноль. Мы прибавляем к ней

504 половину «соседа» 8 (4), то есть 0 + 4 = 4 плюс

перенос (1).

Остальные цифры аналогичны.

Ответ: 06222084 * 6

3732504

Правило умножения на 6: является «сосед» чётным или не чётным – никакой роли не играет. Мы смотрим только на саму цифру: если она чётная, прибавляем к неё целую часть половины «соседа», если нечётная, то кроме половины «соседа» прибавляем еще 5.

Пример: 0443052 * 6

0443052 * 6 2 – чётная и не имеет «соседа», напишем её снизу

2

0443052 * 6 5 – нечётная: 5+5 и плюс половина «соседа» 2 (1)

12 будет 11. Запишем 1 и в перенос 1

0443052 * 6 половина от 5 будет 2, и прибавим перенос 1, то будет 3

312

0443052 * 6 3 – нечетная, 3 + 5 = 8

8312

0443052 * 6 4 + половина от 3 (1) будет 5

58312

0443052 * 6 4 + половина от 4 (2) будет 6

658312

0443052 * 6 ноль + половина от 4 (2) будет 2

2658312

Ответ: 2658312.

Как мы видим, быстрый счёт это уже не тайна за семью печатями, а научно разработанная система. Раз есть система, значит, её можно изучать, ей можно следовать, ею можно овладевать.



Предварительный просмотр:

Софизмы

Софизмом называется умышленно ложное умозаключение, которое имеет видимость правильного. Каков бы ни был софизм, он обязательно содержит одну или несколько замаскированных ошибок. Особенно часто в математических софизмах выполняются «запрещенные»   действия или не учитываются условия применения теорем, формул и правил. Иногда рассуждения ведутся с использованием ошибочного чертежа или опираются на приводящие к ошибочным заключениям «очевидности».

         Приведем примеры некоторых математических софизмов:

1.   Найти ошибку в рассуждении: Имеем верное числовое равенство: 4:4=5:5.

Вынесем за скобки в каждой части его общий множитель. Получим: 4(1:1)=5(1:1).

Числа в скобках равны, поэтому 4=5 или .

(Ошибка допущена  в левой и правой частях тождества 4:4=5:5 при вынесении общего множителя за скобки).

2. Тема: упрощение выражений  5 = 6. 

        Возьмём числовое тождество:

35 + 10 – 45 = 42 + 12 – 54

Вынесем общие множители левой и правой частей за скобки.

Получим:

5 (7 + 2 – 9) = 6 (7 + 2 – 9)

Разделим обе части этого равенства на общий множитель (заключённый в скобки).

Получаем  5 = 6

(Ошибка: общий множитель (7 + 2 – 9) равен 0, а делить на 0 нельзя).

3.  Тема: единицы измерений

 Один рубль не равен ста копейкам  

Возьмем верное равенство:

1 р. = 100 к.,

Возведем его по частям в квадрат, получим:

1 р. = 10000 к.  

Таким образом, один рубль не равен ста копейкам.

     (Ошибка: возведение в квадрат величин не имеет смысла, в квадрат возводятся только числа).

Логические софизмы

               Кроме математических софизмов, существует множество других. Понять абсурдность таких утверждений проще, но от этого они не становятся менее интересными. Очень многие софизмы выглядят как лишенная смысла и цели игра с языком; игра, опирающаяся на многозначность языковых выражений, их неполноту, недосказанность, зависимость их значений от контекста и т.д. Эти софизмы кажутся особенно наивными и несерьезными.

  1. Полупустое и полуполное

«Полупустое есть то же, что и полуполное. Если равны половины, значит равны и целые. Следовательно, пустое есть то же, что и полное».

Равен ли полный стакан пустому?

Пусть имеется стакан, наполненный водой до половины. Тогда можно сказать, что стакан, наполовину полный равен стакану, наполовину пустому. Увеличивая обе части равенства вдвое, получим, что стакан полный равен стакану пустому.

  1. Не знаешь то, что знаешь

«Знаешь ли ты то, о чём я хочу тебя спросить?» -  «Нет». – «Знаешь ли ты, что добродетель есть добро?» - «Знаю». – «Об этом я и хотел тебя спросить. А ты, выходит, не знаешь то, что знаешь».

  1. Лекарства

«Лекарство, принимаемое больным, есть добро. Чем больше делать добра, тем лучше. Значит, лекарств нужно принимать как можно больше».

  1. Вор

«Вор не желает приобрести ничего дурного. Приобретение хорошего есть дело хорошее. Следовательно, вор желает хорошего».

  1. Рогатый

«Что ты не терял, то имеешь. Рога ты не терял. Значит, у тебя рога».

  1. Апельсин- планета

Земля, Марс и т. д. - круглые. Значит, все планеты круглые. Апельсин тоже круглый, значит апельсин - планета?

  1. Сидящий стоит

«Сидящий встал; кто встал, тот стоит; следовательно, сидящий стоит».



Предварительный просмотр:

Старинные задачи на уроках математики

ИЗ КНИГИ «ТЫСЯЧА И ОДНА НОЧЬ»

Одна женщина отправилась в сад собирать яблоки. Чтобы выйти из сада, ей нужно было пройти через четыре двери, у каждой из которых стоял стражник. Стражнику у первых дверей женщина отдала половину сорванных ею яблок. Дойдя до второго стражника, женщина отдала ему половину оставшихся. Так же она поступила и с третьим стражником, а когда она поделилась яблоками с четвёртым стражником, у неё осталось 10 яблок. Сколько яблок она собрала в саду?

ИНДИЯ

Брахмагупта, около 600 г.

Слон, слониха и слонёнок пришли напиться к озеру, чтобы напиться воды. Слон может выпить озеро за 3ч, слониха - за 5ч, а слонёнок - за 6ч. За сколько времени они все вместе выпьют озеро?

ИЗ АКМИМСКОГО ПАПИРУСА

 (VI в.)

Некто взял из сокровищницы 1/13.   Из того, что осталось, другой взял 1/17. Оставил же в сокровищнице 192. Мы хотим узнать, сколько было в сокровищнице первоначально?

ИЗ КНИГИ «КОСС» Адама Ризе (XVI в.)

Трое выиграли некоторую сумму денег. На долю первого пришлось ¼ этой суммы, на долю второго -1/7, а долю третьего – 17 флоринов. Как велик весь выигрыш?

Старинная задача (Китай, II в.).

Дикая утка от южного моря до северного моря летит 7 дней. Дикий гусь от северного моря до южного моря летит 9 дней. Теперь дикая утка и дикий гусь вылетают одновременно. Через сколько дней они встретятся?

 

СТАРИННЫЕ РУССКИЕ ЗАДАЧИ

Здесь вы найдете занимательные задачи из русских учебников математики, опубликованных в России до 1800 года, в частности, из знаменитой "Арифметики" Л.Ф. Магницкого. Это задачи с интересным содержанием или интересными способами решения, задачи, касающиеся интересных свойств чисел, математические игры. Задачи пригодятся на уроках, в математических кружках. Элемент занимательности облегчит обучение, зарядка для ума украсит досуг.  

НА МЕЛЬНИЦЕ  

На мельнице имеется три жернова. На первом из них за сутки можно смолоть 60 четвертей зерна, на втором 54 четверти, а на третьем 48 четвертей. Некто хочет смолоть 81 четверть зерна за наименьшее время на этих трех жерновах.

За какое наименьшее время можно смолоть зерно и сколько для этого на каждый жернов надо зерна насыпать?

СКОЛЬКО ЯИЦ В ЛУКОШКЕ?  

Пришел крестьянин на базар и принес лукошко яиц. Торговцы его спросили: "Много ли у тебя в том лукошке яиц?" Крестьянин молвил им так: "Я всего не помню на перечень, сколько в том лукошке яиц. Только помню: перекладывал я те яйца в лукошко по 2 яйца, то одно яйцо лишнее осталось на земле; и я клал в лукошко по 3 яйца, то одно же яйцо осталось; и я клал по 4 яйца, то одно же яйцо осталось; и я их клал по 5 яиц, то одно же яйцо осталось; и я их клал по 6 яиц, то одно же яйцо осталось; и я клал их по 7 яиц, то ни одного не осталось. Сочти мне, сколько в том лукошке яиц было"?  

КАК УЗНАТЬ ДЕНЬ НЕДЕЛИ?  

Перенумеровав дни недели, начиная с понедельника, по порядку с 1 до 7, предложите кому-нибудь загадать некоторый день недели. Затем предложите порядковый номер задуманного дня увеличить в 2 раза и к этому произведению прибавить 5. Полученную сумму предложите умножить на 5, а затем то, что получится, умножить на 10. По объявленному результату вы называете день недели, который был загадан. Как узнать загаданный день недели?  

СКОЛЬКО РАЗ СОВМЕСТЯТСЯ СТРЕЛКИ?  

Часы пробили полночь. Сколько раз и в какие моменты времени до следующей полуночи часовая и минутная стрелки будут совмещаться?  

ЗА СКОЛЬКО МИНУТ?  

Ребята пилят бревна на метровые куски. Отпиливание одного такого куска занимает одну минуту.

За сколько минут они распилят бревно длиной 5 метров?  

ЧТО ЭТО ТАКОЕ?  

Что это такое две ноги сидели на трех, а когда пришли четыре и утащили одну, то две ноги, схватив три, бросили их в четыре, чтобы четыре оставили одну?  

КАК ЭТО МОГЛО БЫТЬ?  

У одного старика спросили, сколько ему лет. Он ответил, что ему сто лет и несколько месяцев, но дней рождения у него было всего 25.

Как это могло быть?  

ПУТНИК

Путник, догнав другого, спросил его: «Далеко ли до деревни, которая впереди?» Другой путник ответил: « Расстояние от деревни, из которой ты идёшь, равно трети всего расстояния меду деревнями. А если пройдёшь ещё две версты, будешь ровно посередине между деревнями. Сколько вёрст осталось идти первому путнику?

ВОЗ СЕНА

Лошадь съедает воз сено за месяц, коза - за два месяца, овца - за три месяца. За какое время лошадь, коза и овца вместе съедят такой же воз сена?

Словарь единиц измерения

Верста (поприще) – мера длины, равная 1,06 км

Аршин – мера длины, равная 0,71 м

Алтын – денежная единица, равная 3 копейкам

Полушка – денежная  единица, равная 0,25 рублей

Гривна – денежная единица, равная 10 копейкам

Фунт – мера веса, равная 453,6 г

Сажень – мера длины, равная 2,13 м

Четверть – мера, равная четвертой части какой-либо единицы

измерения

Пядь – расстояние между вытянутыми большим и указательным пальцами руки при их наибольшем удалении. (19 – 23 см)

Локоть – расстояние от конца вытянутого среднего пальца руки до локтевого сгиба. (38 – 46 см)

Вершок – мера длины, равная 4,5 см

Задача Л. Ф. Магницкого (из «Арифметики»). 

Некий торговец купил 112 баранов старых и молодых, дал 49 рублёв 20 алтын, за старого платил по 15 алтын и по 2 деньги, а за молодого по 10 алтын, и ведательно есть, колико старых и молодых баранов купил он.

Задача Л. Ф. Магницкого (из «Арифметики»). 

Спросил некто учителя скажи, сколько у тебя в классе учеников, так как хочу отдать к тебе в учение своего сына. Учитель ответил если придет еще учеников столько же, сколько имею, и полстолько, и четвертая часть, и твой сын, тогда будет у меня учеников 100. Спрашивается, сколько было у учителя учеников?

 Задача о гусях. 

Летело стадо гусей, навстречу им летит один гусь и говорит «Здравствуйте, сто гусей», а те ему отвечают «Нет, нас не сто гусей, а если бы нас было еще столько, сколько есть, да еще полстолько, да четверть столько, да еще ты, один гусь с нами, тогда нас было бы ровно сто гусей». Сколько их было?

 

Задача Л. Н. Толстого.

В рассказе Л. Н. Толстого «Много ли человеку земли нужно» крестьянину отводилось столько земли, сколько он успевал обежать в течение одного дня. По какому контуру ему выгоднее было бежать по квадратному, шестиугольному [правильный шестиугольник] или по кругу Указание: при равенстве периметров этих фигур какая имеет большую площадь?

Задача Л. Н. Толстого. 

Артели косцов надо было скосить два луга, один вдвое больше другого. Поло-

вину дня артель косила большой луг. После этого артель разделилась пополам первая половина осталась на большом лугу и докосила его к вечеру до конца; вторая же половина косила малый луг, на котором к вечеру еще остался участок, скошенный на другой день одним косцом за один день работы. Сколько косцов было в артели?

 Задача Л. Н. Толстого. 

На противоположных стенах комнаты определенной длины и ширины сидят муха и паук, муха — на полтора аршина от пола, паук — на полтора аршина от потолка. Какое между ними кратчайшее расстояние, которое мог бы проползти паук, чтобы достать муху?

Задача Льва Толстого

Продавец продает шапку. Стоит 10 р. Подходит покупатель, меряет и согласен взять, но у него есть только 25 р. Продавец отсылает мальчика с этими 25 р. к соседке разменять. Мальчик прибегает и отдает 10+10+5. Продавец отдает шапку и сдачу в 15 руб. Через какое то время приходит соседка и говорит, что 25 р. фальшивые, требует отдать ей деньги. Ну что делать. Продавец лезет в кассу и возвращает ей деньги.

На сколько обманули продавца?

Двенадцать человек

   Двенадцать человек несут 12 хлебов: каждый мужчина несет по 2 хлеба, женщина — по половине хлеба, а ребенок по четверти хлеба.

Сколько было мужчин, женщин и детей?            

Собака и заяц

   Собака усмотрела зайца в 150 саженях от себя. Заяц пробегает за 2 минуты 500 саженей, а собака — за 5 минут 1300 саженей.

За какое время собака догонит зайца?            

На охоте

   Пошел охотник на охоту с собакой. Идут они лесом, и вдруг собака увидала зайца. За сколько скачков собака догонит зайца, если расстояние от собаки до зайца равно 40 скачкам собаки и расстояние, которое пробегает собака за 5 скачков, заяц пробегает за 6 скачков?

(В задаче подразумевается, что скачки делаются одновременно и зайцем и собакой.)    

Бочонок

   Один человек выпивает бочонок кваса за 14 дней, а вместе с женой выпивает такой же бочонок кваса за 10 дней. Нужно узнать, за сколько дней жена одна выпивает такой же бочонок кваса.

Паромщик

   Паромщик многие годы зарабатывал на жизнь тем, что перевозил через реку людей, их скраб и животных. Платы он большой не взимал, поэтому люди охотно пользовались его услугами, но за всю жизнь так и не удалось ему скопить денег на новый большой паром. На его маленьком пароме хватало места только для самого паромщика и еще чего-нибудь одного. Однажды ему нужно было переправить через реку волка, козу и капусту. Но вот неудача - волка с козой без присмотра оставлять нельзя, козу с капустой тоже.

Объясните паромщику как в целости и сохранности переправить через реку капусту и животных?



Предварительный просмотр:

Числа-великаны и числа-малютки

ЧИСЛО - важнейшее математическое понятие. Возникнув в простейшем виде ещё в первобытном обществе, понятие числа изменялось на протяжении веков, постепенно обогащаясь содержанием по мере расширения сферы человеческой деятельности и связанного с ним расширения круга вопросов, требовавшего количеств, описания и исследования. На первых ступенях развития понятие числа. Определялось потребностями счёта и измерения, возникавшими в непосредственной практической деятельности человека. Затем число становится основным понятием математики, и дальнейшее развитие понятия числа определяется потребностями этой науки. В математике есть такие понятия как числа великаны и числа малютки.

Оперируя большими числами, ученые пользуются степенями 10 для того, чтобы избавиться от огромного количества нулей. Например, 19 160 000 000 000 миль можно записать как 1,916•1013 миль. Так же точно очень маленькое число, например 0,0000154324 г, может быть записано 1,54324•10–5 г. Из приставок, используемых перед числительными, самой малой величине соответствует атто, происходящая от датского или норвежского atten – восемнадцать. Приставка означает 10–18. Приставка экса (от греческого hexa, т.е. 6 групп по 3 нуля), или сокращенно Э, означает 1018.

Самым большим числом, встречающимся в толковых словарях и имеющим название – степенью 10, является центилион, впервые использованный в 1852 г. Это миллион в сотой степени, или единица с 600 нулями.

Самое маленькое число которое используют Микроме́тр (мкм, µm, от греч. μικρός — маленький и μέτρον — мера, измерение) — единица измерения длины, равная 10−6 метра (10−3 миллиметра). Также в 1879—1967 годах официально использовалось название микрон (мк, µ) (ныне устарело).

Для лучшего представления этой единицы длины можно привести некоторые данные: диаметр эритроцита составляет 7 мкм, толщина человеческого волоса (для жителей России) — в среднем 40 мкм.

Микрометр является стандартным допуском отклонений от заданного размера (по ГОСТу) в машиностроительном и практически почти в любом производстве изготовления каких-либо деталей, где требуется исключительная точность размеров. В микрометрах также измеряют длину волн инфракрасного излучения.

Для сокращения записи больших чисел давно используется система величин, в которой каждая из последующих в тысячу раз больше предыдущей:

Название

Значение

Короткая
шкала

Длинная
шкала

тысяча

103

103

миллион

106

106

миллиард

(109)

109

биллион

109

1012

биллиард

1015

триллион

1012

1018

триллиард

1021

квадриллион

1015

1024

квадриллиард

1027

квинтиллион

1018

1030

квинтиллиард

1033

секстиллион

1021

1036

секстиллиард

1039

септиллион

1024

1042

септиллиард

1045

октиллион

1027

1048

октиллиард

1051

нониллион

1030

1054

нониллиард

1057

дециллион

1033

1060

дециллиард

1063

Произношение чисел, идущих далее, часто различается.

Название

Значение

Короткая
шкала

Длинная
шкала

ундециллион

1036

1066

додециллион

1039

1072

тредециллион

1042

1078

кваттуордециллион

1045

1084

квиндециллион

1048

1090

седециллион

1051

1096

септдециллион

1054

10102

октодециллион

1057

10108

новемдециллион

1060

10114

вигинтиллион

1063

10120

анвигинтиллион

1066

10126

довигинтиллион

1069

10132

тревигинтиллион

1072

10138

кватторвигинтиллион

1075

10144

квинвигинтиллион

1078

10150

сексвигинтиллион

1081

10156

септемвигинтиллион

1084

10162

октовигинтиллион

1087

10168

новемвигинтиллион

1090

10174

тригинтиллион

1093

10180

антригинтиллион

1096

10186

дуотригинтиллион

1099

10192

гугол[1]

10100

10100

септдециллиард

10105

октодециллиард

10111

новемдециллиард

10117

вигинтиллиард

10123

квадрагинтиллион

10123

10240

анвигинтиллиард

10129

дуовигинтиллиард

10135

тревигинтиллиард

10141

кватторвигинтиллиард

10147

квинвигинтиллиард

10153

квинквагинтиллион

10153

10300

сексвигинтиллиард

10159

септемвигинтиллиард

10165

октовигинтиллиард

10171

новемвигинтиллиард

10177

тригинтиллиард

10183

сексагинтиллион

10183

10360

антригинтиллиард

10189

дуотригинтиллиард

10195

септуагинтиллион

10213

10420

квадрагинтиллиард

10243

октогинтиллион

10243

10480

нонагинтиллион

10273

10540

квинквагинтиллиард

10303

центиллион

10303

10600

Название

Значение

Короткая
шкала

Длинная
шкала

анцентиллион

10306

10606

дуоцентиллион

10309

10612

трецентиллион

10312

10618

кватторцентиллион

10315

10624

децицентиллион

10333

10660

ундецицентиллион

10336

10666

сексагинтиллиард

10363

вигинтицентиллион

10363

10720

третригинтацентиллион

10402

10798

септуагинтиллиард

10423

октогинтиллиард

10483

нонагинтиллиард

10543

центиллиард

10603

дуцентиллион

10603

101200

анцентиллиард

10609

дуоцентиллиард

10615

трецентиллиард

10621

кватторцентиллиард

10627

децицентиллиард

10663

ундецицентиллиард

10669

трицентиллион

10903

101800

квадрингентиллион

101203

102400

квингентиллион

101503

103000

сесцентиллион

101803

103600

септингентиллион

102103

104200

окстингентиллион

102403

104800

нонгентиллион

102703

105400

миллиллион (или милиаиллион)

103003

106000

дуомилиаллион

106003

1012000

тремиллиаллион

109003

1018000

квинквемилиаллион

1015003

1030000

дуцентдуомилианонгентновемдециллион

10308760

10617514

милиамилиаиллион

103000003

106000000

дуомилиамилиаиллион

106000003

1012000000

гуголплекс[1]

1010100

1010100

зиллион

103×n+3

106×n

Слово „миллиард" употребляется у нас в смысле тысячи миллионов и при денежных вычислениях и в точных науках. Но, например, в Германии и в Америке под миллиардом иногда имеют ввиду не тысячу, а всего сто мил-лионов. Этим, между прочим, можно объяснить то, что слово „миллиардер" было в ходу за океаном еще тогда, когда ни один из тамошних богачей не имел состояния в тысячу миллионов. Огромное состояние Рокфеллера незадолго до войны исчислялось „всего" 900 миллионов долларов, а остальных „миллиардеров"-меньшими числами. Только во время войны появились в Америке миллиардеры в нашем смысле слова.

Ощутить огромность этих биллионов и триллионов трудно даже человеку, привычному к обращению с миллионами. Великан-миллион — такой же: карлик рядом с сверхвеликаном-биллионом, как единица рядом с миллионом. Об этом взаимоотношении мы обыкновенно забываем и в своем воображении не делаем большой разницы между миллионом, биллионом и триллионом. Мы уподобляемся здесь тем первобытным народам, которые умеют считать только до 2 или 3, а все числа свыше их обозначают словом много.

Задачи

1. Сколько времени потребуется человеку, чтобы сосчитать миллиард зерен, если он в минуту будет считать по 100 зерен.

Решение: По нашему условию, сосчитать до миллиарда человеку потребуется 1 000 000000:100=10 000 000 мин. Или (10 000 000:60=166 667), т. е. Примерно 170 000 ч.или (170000:24=7000) около 7000 суток, т. е. Более 16 лет беспрерывного счета.

2. От Земли до Марса около 60млн.км. Сколько времени придется лететь на ракете от Земли до Марса, если скорость ракеты будет 10км/ч? Сколько времени потребовалось бы самолету, летящему со скоростью 1000км/ч, чтобы преодолеть это расстояние?

Решение: Ракета будет лететь 60:10=6млн.с, или около 1667ч, что составляет примерно 70 суток. Так как скорость самолета 1000км/ч, то на все расстояние ему потребуется 60 000 ч, или 2500 суток, т.е. примерно 7 лет.

3. В нашей стране проживают около 250 млн. человек.  Если все люди встанут в одну шеренгу, то какой  длины будет эта шеренга?  (Пусть каждый человек занимает место длиной в 50см).

Решение: 250 000 000•50 =12 500 000 000см, т.е. 125 000 км

4. Самая высокая гора на Земле – Джомолунгма. Её высота 8848м. Сколько этажей имел бы дом высотой с эту гору, если считать, что расстояние между этажами 4м. Решение: 8848:4=2212 этажей.



Предварительный просмотр:

Школьная математическая регата

        Предварительно создаются команды из учащихся 7-х, 8-х и 9-х классов. Всего 3 команды.  По одному учащемуся из каждого класса и учитель математики – судьи. В команде 6 человек. Команды придумывают себе название, вымпел,  на котором это название пишется. Для каждой команды выделяется стол – это «плавсредство».

        Ведущий объявляет о начале регаты и о том, что в программе ожидаются «заезды» «одиночек», «двоек», « четвёрок» и «шестёрок».

        -Прошу желающих делать ставки.

        Зрители на листочках с одной стороны ставят свою фамилию, с другой – указывают победителей каждого заезда, например: 1-й заезд -7, 2-й заезд -9, 3-й заезд – 8, 4-й заезд – 7.

        После того, как «ставки сделаны», демонстрируется приз (шоколад и т.п.), который получит тот, кто угадает победителей.

        Ведущий приглашает на старт участников 1-го «заезда». За 12 минут участник от каждой команды должен решить 2 задачи, каждая из которых оценивается в 6 баллов. Судьи проверяют решения, подсчитывают результат и объявляют победителя. Далее следует «заезд» «двоек», «четвёрок» , «шестёрок».

        Паузы можно заполнить концертными номерами или мини-викторинами.

«1-й заезд»

7 класс

  1. Если человек, стоящий в очереди перед Вами, был выше человека, стоящего после того человека, который стоял перед Вами, то был ли человек, стоящий перед Вами, выше Вас? (Да, потому что человек, стоящий после человека, который стоял передо мной, - это я сам).
  2. У Саши из 10 ответов 5 оказались правильными.  А у Алёши из 5 – 3. Чей результат лучше?  (У Алёши, т.к. ).

8 класс

  1. В месяце три воскресенья выпали на чётные числа. Какой день недели был седьмого числа этого месяца? (Пятница)
  2. Цену товара уменьшили на 10 %, а затем ещё на 10%. Станет ли он дешевле, если его цену снизить на 20%? (Да, на 1%)

9 класс

  1. Найдите последнюю цифру в записи числа . (4)
  2. Можно ли, имея два сосуда ёмкостью 3л и 2л, набрать из водонапорного крана 4л воды? (Можно.  Набираем  3л воды и переливаем в 2-х литровый сосуд, затем выливаем 2л и переливаем 1л в 2-литровый сосуд. Снова наполняем 3-х литровый сосуд, получаем 4л.)

«2-й заезд»

7 класс

  1. Найдите числа ребуса aa+b=bcc. (99+1=100)
  2. Баба-Яга варит волшебное зелье: к 1,5 кг мёда она добавила 100г волчих когтей, 100г дёгтя и 300г слёз кикиморы. Сколько процентов варева составляют слёзы кикиморы? (15%)

8 класс

  1. Расшифруйте: .
  2. Как разменять 5 рублей монетами по 20 копеек и 5 копеек, чтобы пятаков и двадцатикопеечных монет было поровну? (Взять по 20 монет).

9 класс

  1. Мама купила коробку кускового сахара. Дети съели верхний слой, состоящий из 77 кусочков. Затем они съели боковой слой, состоящий из 55 кусочков. Наконец, они съели передний слой. Сколько кусочков сахара осталось в коробке?  (300 кусочков).
  2. Расшифруйте:

abb

                                +  bab

                                bba

                                    ____

                                     bbb0     (a=8, b=1)

«3-й заезд»

7 класс

  1. В выражении  расставьте скобки так, чтобы в результате получилось 28. ()
  2. Лев съел овцу за один час, а волк съел овцу в 2 часа, а пёс съел овцу за 3 часа. За сколько они все вместе ту овцу съедят?  ( За  часа).

8 класс

  1. В классе 35 учеников. Из них 20 занимаются в математическом кружке, 11 – в спортивном. А 10 ребят не посещают кружков вообще. Сколько тех ребят, кто посещает оба кружка?  (6 человек).
  2. Собака усмотрела в 150 саженях зайца, который перебегает в 2 минуты по 500 сажен, а собака в 5 минут – 1300 сажен. За какое время собака догонит зайца? (За 15 минут)

9 класс

  1. Некто на вопрос о возрасте двух его сыновей отвечал: «Первый мой сын втрое старше второго, а обоим им вместе столько лет, сколько было мне 29 лет тому назад; мне теперь 45 лет». (4 года и 12 лет)
  2. В воскресенье 19 учеников нашего класса побывали в планетарии, 10 – в цирке, и 6 – на стадионе. Планетарий и цирк посетили 5 учеников; планетарий и стадион – 3, цирк и стадион – 1. Сколько учеников в нашем классе, если никто не успел посетить все три места, а 3 ученика не посетили ни одного места? (29 учеников)

«4-й заезд»

7 класс

  1. Все рёбра куба увеличили на 20%. На сколько процентов увеличился объём куба? (На 72, 8%)
  2. Некоторые клетки квадрата 4Х4 – белые, а остальные чёрные. Соседними считаются клетки, у которых есть общая сторона. Известно, что у каждой белой клетки ровно 3 чёрные соседки, а у каждой чёрной – ровно одна белая соседка. Восстановите раскраску по этим условиям.

C:\Users\Олег\Pictures\2014-01-19\Image0336.BMP

8 класс

  1. Разделите квадрат двумя прямолинейными разрезами на части, чтобы из них можно было сложить два квадрата.

C:\Users\Олег\Pictures\2014-01-19\Image0337.BMP

  1. Некоторые рёбра куба красные, а остальные чёрные. Известно. Что среди красных рёбер нет параллельных. Какое наибольшее число красных рёбер может быть у куба при этих условиях? (3 красных ребра)

9 класс

  1. Клетки квадрата 7Х7 раскрасьте в наименьшее число цветов, каждую одной краской так, чтобы у каждой клетки все четыре соседки (по стороне) были разных цветов.

C:\Users\Олег\Pictures\2014-01-19\Image0338.BMP (Потребуется не менее 4 цветов)

  1. Теннисный шарик и баскетбольный мяч обтянули проволокой по экватору. Длину проволоки увеличили на 1см.  Где зазор будет больше?  (Одинаков)

В то время, когда участники «заезда» решают свои задачи, их товарищи по команде, не участвующие в этом заезде также решают эти задачи. Если участники заезда не решили задачу, то команда может рассказать своё решение, если оно правильное, то засчитывается 3 балла.

Команда- победитель по сумме очков получает приз.

В то время, когда команды готовятся, ведущий организует работу с болельщиками.

  1. Сценку «3=7 и 2=5» показывают ученики 5 класса. 

Учитель: Ну что, Петров? Что же мне с тобой делать?

Петров: А что?

Учитель: Весь год ты ничего не делал, ничего не учил. Что тебе ставить в ведомости, прямо не знаю.

Петров (угрюмо глядя в пол): Я, Иван Иваныч, научным трудом занимался.

Учитель: Да что ты? Каким же?

Петров: Я решил, что вся математика наша неверна и ... доказал это!

Учитель: Ну и как же, товарищ Великий Петров, вы этого добились?

Петров: А-а, что там говорить, Иван Иваныч! Я же не виноват, что Пифагор ошибался и этот ... Архимед!

Учитель: Архимед?

Петров: И он тоже, Ведь говорили, что три равно только трём.

Учитель: А чему же ещё?

Петров (торжественно): Это неверно! Я доказал, что три равно семи!

Учитель: Как это?

Петров: А вот, смотрите: 15 -15 = 0. Верно?

Учитель: Верно.

Петров: 35 - 35 =0 - тоже верно. Значит, 15-15 = 35-35. Верно?

Учитель: Верно.

Петров: Выносим общие множители: 3(5-5) = 7(5-5). Верно?

Учитель: Точно.

Петров: Хе-хе! (5-5) = (5-5). Это тоже верно!

Учитель: Да.

Петров: Тогда всё вверх дном: 3 = 7!

Учитель: Ага! Так, Петров, дожили.

Петров: Я не хотел, Иван Иваныч. Но против науки ... не погрешишь!

Учитель: Понятно. Смотри: 20-20 = 0. Верно?

Петров: Точно!

Учитель: 8-8 = 0 - тоже верно. Тогда 20-20 = 8-8. Тоже верно?

Петров: Точно, Иван Иваныч, точно.

Учитель: Выносим общие множители: 5(4-4) = 2(4-4). Верно?

Петров: Верно!

Учитель: Тогда всё, Петров, ставлю тебе «2»!

Петров: За что, Иван Иваныч?

Учитель: А ты не расстраивайся, Петров, ведь если мы разделим обе части равенства на (4-4), то 2=5. Так ты делал?

Петров: Ну, допустим.

Учитель: Вот я и ставлю «2», не всё ли равно. А?

Петров: Нет, не всё равно, Иван Иваныч, «5» лучше.

Учитель: Возможно, лучше, Петров, но пока ты этого не докажешь, у тебя будет двойка за год, равная, по-твоему, пятёрке!

Затем ведущий рассказывает о математических софизмах. Презентация (Приложение)

  1. Сценку «Деление с проверкой» показывают ученики 5 класса.

На сцену выходят две девочки – Ксюша и Саша.

Ксюша (пританцовывая):

                                Я танцую руки в боки,

                                Приготовила уроки.

                                Тра-ля-ля, тра-ля-ля …

Саша (за сценой): А задачу ты разве решила?

Ксюша: Задачу я уже читала, задача очень лёгкая, и я сейчас её решу.

Саша: Скорее решай, а потом будешь отплясывать.

Ксюша садится за стол, берёт ручку, задачник, начинает читать задачу. Пишет, смотрит, снова пишет, и чем дальше, тем её движения становятся быстрее. В конце концов, девочка опускает голову на стол и громко плачет. Входит Саша.

Саша: Ксюша, что случилось? Что с тобой?

Ксюша (всхлипывая): Задача не выходит.

Саша: Как не выходит? Трудная она, что ли? Не знаешь, как делать?

Ксюша: Нет, знаю, да вот не выходит.

Саша: Как же так? Знаешь, как делать, а не выходит? Если не выходит, значит, делаешь неверно. Ну, показывай, что за задача такая? Читай условие.

Ксюша (читает): За 8 метров шёлка заплатили 40 рублей. Сколько стоит 1 м шёлка?

Саша: И эту задачу ты не сумела решить?

Ксюша: Если за 8 метров заплатили 40 рублей, то за 1 метр заплатили в 8 раз меньше. Я делила 40 на 8.

Саша: Верно, верно! Как же не выходит? Должно выйти. Сколько ты получила?

Ксюша: 41.

Саша: Что? 41! Ха-ха-ха! Ну и разделила, нечего сказать! А ну-ка, дели снова.

Катя: 40 делим на 8, получаем 4. Четырежды восемь – 32. Из сорока вычтем 32, будет 8. 8 разделим на 8, получим 1. Один метр будет стоить 41 рубль.

Все эти вычисления Ксюша показывает на доске.

Саша: Да ты сама пойми, 8м шёлка стоят 40 рублей, а 1м – 41 дубль. Разве это может быть?

Ксюша: Этого-то я и не понимаю. Объясни мне, Саша!

Саша: Как проверить деление?

Ксюша: Умножением. Нужно делитель умножить на частное.

Саша: Вот и умножай.

Ксюша (начинает умножать на доске): 8 умножить на 1, будет 8. Четырежды восемь – равно 32. 8 да 2 – равно 10, 0 пишем 1 в уме. 3 да 1 – равно 4. Всего 40. Верно …

Саша: Интересно! Ведь цифры ты получила верные!

Ксюша: Ну вот, верные! В ответе 5, а у меня 41! Как же это – верные?

Саша: А вот после того, как ты получишь 4 и 1, сложи их, и у тебя получится 5! Если непонятно, тебе ребята объяснят (показывает в зрительный зал), а мне некогда: надо уроки готовить …

Ведущий: Ребята, объясните Ксюше, почему можно 4 и 1 сложить? Укажите ей ошибки в делении и умножении.

Затем ведущий проводит

Блицтурнир для зрителей

  1. Найдите число, одна треть которого составляет 12.  (36)
  2. Какое число делится без остатка на любое целое число, отличное от нуля? (0)
  3. В семье у каждого из шести братьев есть сестра. Сколько детей в этой семье? (7)
  4. Три разных числа сначала сложили, а затем перемножили. Сумма и произведение оказались равными. Какие это числа?  (1; 2; 3)
  5. Как изменится дробь, если числитель увеличить на знаменатель?  (Увеличится на 1)
  6. Какой знак нужно поставить между числами 5 и 6, чтобы получилось число больше 5, но меньше 6? (Запятую, получится 5,6)
  7. Число 66 моментально увеличьте на половину этого числа.  (Перевернуть, получится 99)
  8. На какое число нужно разделить 3, чтобы получить 6?  (На )
  9. Который сейчас час, если оставшаяся часть суток вдвое больше прошедшей? (8 часов)
  10.  Одно яйцо варится 4 минуты. Сколько нужно минут, чтобы сварить 5 яиц?  (4 минуты)
  11. Шесть рыбаков съели шесть судаков за шесть дней. За сколько дней десять рыбаков съедят десять судаков?  ( за 6 дней)
  12.  Что больше: произведение всех цифр или их сумма?  (Сумма, так как произведение равно 0.)
  13. Когда делимое и частное равны между собой?  ( Когда делитель – 1)
  14. Будем условно считать, что если человек не будет семь суток есть или семь суток спать, то он умрет. Пусть человек неделю не ел и не спал. Что он должен сделать в первую очередь к концу седьмых суток: поесть или поспать, чтобы остаться в живых?
    (Несмотря на шутливый характер, задача имеет строгое и единственное решение.  
    Человек не может одновременно и спать и есть. Поэтому срок в семь суток после сна и после еды наступает в разное время. Человек должен сделать то, что неделю назад делал раньше: спал или ел.)

  1. Как можно одним мешком пшеницы, смоловши ее, наполнить два мешка, которые столь же велики, как и мешок, в котором находится пшеница? (Надо один из пустых мешков вложить в другой такой же, а затем в него насыпать смолотую пшеницу.)

  1.  В комнате горело 50 свечей, 20 из них задули. Сколько останется? ( Останется 20: задутые свечи не сгорят полностью.)
  2.  Если в 12 часов ночи идет дождь, то можно ли ожидать, что через 72 часа будет солнечная погода? ( Нет, - через 72 часа будет снова полночь.)
  3.  На край стола поставили жестяную банку, плотно закрытую крышкой, так, что 2/3 банки свисало со стола. Через некоторое время банка упала. Что было в банке? ( Кусок льда.)
  4. Один поезд едет из Москвы в С.-Петербург с опозданием 10 минут, а другой - из С.-Петербурга в Москву с опозданием 20 минут. Какой из этих поездов будет ближе к Москве, когда они встретятся? ( В момент встречи они будут на одинаковом расстоянии от Москвы.)
  5. Шли два отца и два сына, нашли три апельсина. Стали делить - всем по одному досталось. Как это могло быть? ( Это были дед, отец и сын.)
  6. Как правильно сказать: "9 и 7 будет 15" или "9 плюс 7 равно 15"? ( 9+7=16.)
  7. Представьте, что у вас в кармане коробок с одной-единственной спичкой. Вы вошли ночью в тёмную комнату, где есть свеча, керосиновая лампа и газовая плита. Что вы зажжёте в первую очередь? ( Спичку.)

  1. Н.И. Лобачевский – великий математик современности

Презентация (Приложение).

Обратить внимание на факты:

2 слайд: Николай Иванович потерял отца в 7 лет, и мать с тремя сыновьями уехали жить в Казань;

3 слайд:  Николай Лобачевский окончил гимназию в конце 1806 года, показав хорошие знания, особенно по математике и языкам — латинскому, немецкому, французскому;  при поступлении в Казанский университет Лобачевский намерен был посвятить себя медицине;  заслуга профессора Казанского университета И. Бартельса (Ма́ртин Фёдорович Ба́ртельс -Иоганн Христиан Мартин Бартельс, нем. Johann Christian Martin Bartels; 12 августа 1769, Брауншвейг — 20 декабря 1836, Тарту — немецкий, позже российский математик и педагог-друг и учитель великого немецкого математика Карла Фридриха Гаусса) в том, что Н.И. Лобачевский посвятил себя математике.

4, 5, 6 слайды: в 19 лет Лобачевский получил учёную степень магистра и был оставлен при университете; перед этим его заставили покаяться за «дурное поведение» и дать обещание впредь вести себя примерно.

7 слайд: в 24 года он стал профессором Казанского университета; 20 ноября 1845 года Лобачевский был в шестой раз избран ректором на новое четырёхлетие, причём единогласно;

8 слайд: 1846 год был для Лобачевского тяжелым. 8 февраля умирает его двухлетняя дочь Надежда. В этом же году, по истечении 30 лет службы, министерство, по уставу, должно было принять решение об оставлении Лобачевского профессором. 16 августа 1846 года Министерство «по указанию Правительствующего сената» отстранило Лобачевского не только от профессорской кафедры, но и от должности ректора. Вскоре Лобачевский разорился, дом в Казани и имение жены были проданы за долги. В 1852 году умер от туберкулёза старший сын Алексей, любимец Лобачевского. Здоровье его самого было подорвано, слабеет зрение. 

9, 10 слайды: Лобачевский считает аксиому параллельности Евклида произвольным ограничением. С его точки зрения, это требование слишком жёсткое, ограничивающее возможности теории, описывающей свойства пространства. В качестве альтернативы предлагает другую аксиому: на плоскости через точку, не лежащую на данной прямой, проходит более чем одна прямая, не пересекающая данную. Разработанная Лобачевским новая геометрия не включает в себя евклидову геометрию, однако евклидова геометрия может быть из неё получена предельным переходом (при стремлении кривизны пространства к нулю). В самой геометрии Лобачевского кривизна отрицательна. Уже в первой публикации Лобачевский детально разработал тригонометрию неевклидова пространства, дифференциальную геометрию (включая вычисление длин, площадей и объёмов).

Однако научные идеи Лобачевского не были поняты современниками. Его труд «О началах геометрии», представленный в 1832 году советом университета в Академию наук, получил у М. В. Остроградского отрицательную оценку. В иронически-язвительном отзыве на книгу Остроградский откровенно признался, что он ничего в ней не понял, кроме двух интегралов, один из которых, по его мнению, был вычислен неверно (на самом деле ошибся сам Остроградский). Среди других коллег также почти никто Лобачевского не поддержал, росли непонимание и невежественные насмешки. Не найдя понимания на Родине, Лобачевский попытался найти единомышленников за рубежом. В 1837 году статья Лобачевского «Воображаемая геометрия» на французском языке  появилась в авторитетном берлинском журнале Крелле, а в 1840 году Лобачевский опубликовал на немецком языке небольшую книгу «Геометрические исследования по теории параллельных», где содержится чёткое и систематическое изложение его основных идей. Два экземпляра получил Карл Фридрих Гаусс, «король математиков» той поры. Как много позже выяснилось, Гаусс и сам тайком развивал неевклидову геометрию, однако так и не решился опубликовать что-либо на эту тему, полагая, что научная общественность ещё не готова воспринять столь радикальные идеи. Ознакомившись с результатами Лобачевского, он восторженно отозвался о них, но лишь в своих дневниках и в письмах близким друзьям. Лобачевский умер непризнанным, не дожив до торжества своих идей всего 10-12 лет.

11 слайд: Осознание того, что у евклидовой геометрии имеется полноценная альтернатива, произвело огромное впечатление на научный мир и придало импульс другим новаторским идеям в математике и физике.



Предварительный просмотр:

Сценка «Деление с проверкой»

        На сцену выходят две девочки – Ксюша и Саша.

Ксюша (пританцовывая):

                                Я танцую руки в боки,

                                Приготовила уроки.

                                Тра-ля-ля, тра-ля-ля …

Саша (за сценой): А задачу ты разве решила?

Ксюша: Задачу я уже читала, задача очень лёгкая, и я сейчас её решу.

Саша: Скорее решай, а потом будешь отплясывать.

Ксюша садится за стол, берёт ручку, задачник, начинает читать задачу. Пишет, смотрит, снова пишет, и чем дальше, тем её движения становятся быстрее. В конце концов, девочка опускает голову на стол и громко плачет. Входит Саша.

Саша: Ксюша, что случилось? Что с тобой?

Ксюша (всхлипывая): Задача не выходит.

Саша: Как не выходит? Трудная она, что ли? Не знаешь, как делать?

Ксюша: Нет, знаю, да вот не выходит.

Саша: Как же так? Знаешь, как делать, а не выходит? Если не выходит, значит, делаешь неверно. Ну, показывай, что за задача такая? Читай условие.

Ксюша (читает): За 8 метров шёлка заплатили 40 рублей. Сколько стоит 1 м шёлка?

Саша: И эту задачу ты не сумела решить?

Ксюша: Если за 8 метров заплатили 40 рублей, то за 1 метр заплатили в 8 раз меньше. Я делила 40 на 8.

Саша: Верно, верно! Как же не выходит? Должно выйти. Сколько ты получила?

Ксюша: 41.

Саша: Что? 41! Ха-ха-ха! Ну и разделила, нечего сказать! А ну-ка, дели снова.

Катя: 40 делим на 8, получаем 4. Четырежды восемь – 32. Из сорока вычтем 32, будет 8. 8 разделим на 8, получим 1. Один метр будет стоить 41 рубль.

Все эти вычисления Ксюша показывает на доске.

Саша: Да ты сама пойми, 8м шёлка стоят 40 рублей, а 1м – 41 дубль. Разве это может быть?

Ксюша: Этого-то я и не понимаю. Объясни мне, Саша!

Саша: Как проверить деление?

Ксюша: Умножением. Нужно делитель умножить на частное.

Саша: Вот и умножай.

Ксюша (начинает умножать на доске): 8 умножить на 1, будет 8. Четырежды восемь – равно 32. 8 да 2 – равно 10, 0 пишем 1 в уме. 3 да 1 – равно 4. Всего 40. Верно …

Саша: Интересно! Ведь цифры ты получила верные!

Ксюша: Ну вот, верные! В ответе 5, а у меня 41! Как же это – верные?

Саша: А вот после того, как ты получишь 4 и 1, сложи их, и у тебя получится 5! Если непонятно, тебе ребята объяснят (показывает в зрительный зал), а мне некогда: надо уроки готовить …

Ребята, объясните Ксюше, почему можно 4 и 1 сложить? Укажите ей ошибки в делении и умножении.


Предварительный просмотр:


Подписи к слайдам:

Слайд 1

«Людям, которые желают идти верной дорогой, важно также знать и об отклонениях». Аристотель Софизм - это умышленно ложное умозаключение, которое имеет видимость правильного. Он обязательно содержит одну или несколько замаскированных ошибок.

Слайд 2

Софизмы появились еще в Древней Греции. Они тесно связаны с философской деятельностью софистов — платных учителей мудрости , учивших всех желающих философии, логике и, особенно, риторике (науке и искусству красноречия). Одна из основных задач софистов заключалась в том, чтобы научить человека доказывать (подтверждать или опровергать) все, что угодно, выходить победителем из любого интеллектуального состязания

Слайд 3

Вот один из древних софизмов («рогатый»), приписываемый Эвбулиду: « Что ты не терял, то имеешь. Рога ты не терял. Значит, у тебя есть рога». У одной кошки четыре ноги. У нуля кошек три ноги. Если перевести это предложение с «математического» языка на обычный, то получим «Не существует здоровой трехногой кошки». 1 кошка – 4 ноги 0 кошек – 3 ноги Т.к. равные величины, сложенные с равными, дают равные результаты 1+0 = 1, 4+3 = 7. Получили, что у одной кошки 7 ног.

Слайд 4

Дважды два – пять! Очевидно что: 4:4=5:5 вынесем общий множитель 4(1:1)=5(1:1) сократим общие множители 4=5 или 2х2=5

Слайд 5

Известно, что Значит, Откуда То есть -1=1

Слайд 6

Софизм Д ` Аламбера Пусть ad= bc , тогда a:b=c:d и если a>b, то c>d положим теперь a=d=1 и b=c=-1 Эти четыре числа удовлетворяют соотношению a d= bc и условию a>b з начит, c>d т .е. -1 >1


Предварительный просмотр:


Подписи к слайдам:

Слайд 1

Николай Иванович Лобачевский

Слайд 2

Первые годы жизни Н.И.Лобачевский родился 1 декабря 1792г . в Нижнем Новгороде в семье чиновника-геодезиста И.М.Лобачевского и его жены П.А.Лобачевской. Отец семейства умер в возрасте 40 лет, оставив вдову с тремя малолетними сыновьями Александром, Николаем и Алексеем, в бедственном материальном положении. Вскоре семья переехала в Казань. Ежегодное празднование дня рождения Н.И.Лобачевского

Слайд 3

Годы учёбы В 1802г. Прасковья Александровна отдала всех своих сыновей в гимназию. 14.02.1807г. Николай Лобачевский был зачислен в Казанский университет. Большое влияние на выбор дальнейшего пути Лобачевского оказал приглашённый профессор чистой математики Мартин Бартельс, друг и учитель великого математика Карла Фридриха Гаусса. Мартин Бартельс

Слайд 4

Начало преподавательской деятельности С 1812г. Н.И.Лобачевский преподаёт на курсах арифметики и геометрии для готовившихся к экзамену «на чин», а с 1814г. ведёт серьёзное университетское преподавание . Казанский Университет

Слайд 5

Работа в университете В 1816г . В возрасте 24 лет Лобачевский становится экстраординарным профессором. В 1819г. получает должность декана физико-математического факультета. В 1824г. по рекомендации К Ф Гаусса Н.И.Лобачевский был избран членом-корреспондентом Геттингенского Королевского научного общества .

Слайд 6

Научные труды 11.02.1826г. − день рождения неевклидовой геометрии (доклад «Сжатое изложение начал геометрии»); «О началах геометрии» (1829/30г.); «Воображаемая геометрия» (1835г.); «Геометрические исследования по теории параллельных» (1840г.); « Пангеометрия » (1855г.).

Слайд 7

На посту ректора В 1827г. Лобачевский избирается ректором Казанского университета и занимает этот пост 19 лет. За это время были построены новые корпуса, механические мастерские, лаборатории, обсерватория, стал издаваться «Казанский Вестник». Лобачевский вёл курсы геометрии, тригонометрии, алгебры, анализа, теории вероятностей, механики, физики, астрономии, гидравлики . Дом ректора

Слайд 8

Последние годы жизни В последние годы жизни Николай Иванович ослеп и своё последнее произведение «Пангеометрию» должен был диктовать своим ученикам. Умер он 24 .02.1856г . в возрасте 63 лет от паралича лёгких. 1855 год Герб Лобачевского

Слайд 9

Суть геометрии Лобачевского Это дедуктивная теория, исходящая из тех же понятий и аксиом, что и эвклидова геометрия, с единственным фундаментальным исключением − V постулат заменён аксиомой Лобачевского: «К данной прямой через данную точку, не лежащую на прямой, можно провести по крайней мере две параллельные прямые». При этом, в теории нет противоречий, все доказательства безупречны.

Слайд 10

Модели геометрии Лобачевского Наглядное представление геометрии Лобачевского: через точку М проходят две прямые, параллельные прямой D . В этой геометрии кривизна плоскости отрицательна. При изменении кривизны плоскости до нуля, получается геометрия Евклида. Простейшим примером поверхности с постоянной отрицательной кривизной служит псевдосфера, которую изучал в 1868г. итальянский математик Э.Бельтрами. Псевдосфера

Слайд 11

Памятные марки и медали Бюст Н.И.Лобачевского в Нижегородском университете

Слайд 12

1 декабря 1792 года 24 февраля 1856 года



Предварительный просмотр:

Блицтурнир для зрителей

  1. Найдите число, одна треть которого составляет 12.  (36)
  2. Какое число делится без остатка на любое целое число, отличное от нуля? (0)
  3. В семье у каждого из шести братьев есть сестра. Сколько детей в этой семье? (7)
  4. Три разных числа сначала сложили, а затем перемножили. Сумма и произведение оказались равными. Какие это числа?  (1; 2; 3)
  5. Как изменится дробь, если числитель увеличить на знаменатель?  (Увеличится на 1)
  6. Какой знак нужно поставить между числами 5 и 6, чтобы получилось число больше 5, но меньше 6? (Запятую, получится 5,6)
  7. Число 66 моментально увеличьте на половину этого числа.  (Перевернуть, получится 99)
  8. На какое число нужно разделить 3, чтобы получить 6?  (На )
  9. Который сейчас час, если оставшаяся часть суток вдвое больше прошедшей? (8 часов)
  10.  Одно яйцо варится 4 минуты. Сколько нужно минут, чтобы сварить 5 яиц?  (4 минуты)
  11. Шесть рыбаков съели шесть судаков за шесть дней. За сколько дней десять рыбаков съедят десять судаков?  ( за 6 дней)
  12.  Что больше: произведение всех цифр или их сумма?  (Сумма, так как произведение равно 0.)
  13. Когда делимое и частное равны между собой?  ( Когда делитель – 1)
  14. Будем условно считать, что если человек не будет семь суток есть или семь суток спать, то он умрет. Пусть человек неделю не ел и не спал. Что он должен сделать в первую очередь к концу седьмых суток: поесть или поспать, чтобы остаться в живых?
    (Несмотря на шутливый характер, задача имеет строгое и единственное решение.  
    Человек не может одновременно и спать и есть. Поэтому срок в семь суток после сна и после еды наступает в разное время. Человек должен сделать то, что неделю назад делал раньше: спал или ел.)
  15. Как можно одним мешком пшеницы, смоловши ее, наполнить два мешка, которые столь же велики, как и мешок, в котором находится пшеница? (Надо один из пустых мешков вложить в другой такой же, а затем в него насыпать смолотую пшеницу.)
  16.  В комнате горело 50 свечей, 20 из них задули. Сколько останется? ( Останется 20: задутые свечи не сгорят полностью.)
  17.  Если в 12 часов ночи идет дождь, то можно ли ожидать, что через 72 часа будет солнечная погода? ( Нет, - через 72 часа будет снова полночь.)
  18.  На край стола поставили жестяную банку, плотно закрытую крышкой, так, что 2/3 банки свисало со стола. Через некоторое время банка упала. Что было в банке? ( Кусок льда.)
  19. Один поезд едет из Москвы в С.-Петербург с опозданием 10 минут, а другой - из С.-Петербурга в Москву с опозданием 20 минут. Какой из этих поездов будет ближе к Москве, когда они встретятся? ( В момент встречи они будут на одинаковом расстоянии от Москвы.)
  20. Шли два отца и два сына, нашли три апельсина. Стали делить - всем по одному досталось. Как это могло быть? ( Это были дед, отец и сын.)
  21. Как правильно сказать: "9 и 7 будет 15" или "9 плюс 7 равно 15"? ( 9+7=16.)
  22. Представьте, что у вас в кармане коробок с одной-единственной спичкой. Вы вошли ночью в тёмную комнату, где есть свеча, керосиновая лампа и газовая плита. Что вы зажжёте в первую очередь? ( Спичку.)



Предварительный просмотр:

C:\Users\Олег\Pictures\2014-02-01\Image0353.BMP



Предварительный просмотр:

Математический КВН 5-6 классы 07.02.2014

Приветствие команды «ПУПС»

Команда «ПУПС» -Пусть Ум  Побеждает Силу

 Друзья, приветствовать мы рады

Вас всех на встрече в ВКН.

Другой награды нам не надо,

Победу мы хотим взамен.

Поют на мотив песни «Малиновка»:

Ведущего заслыша голосок,

Мы в КВН пришли без опозданья.

Потуже мы затянем поясок

И выполним все сложные заданья …

-А ты что не поёшь?

_ Думаю.

- О чём?

- Хотел угостить членов жюри яблоками. Но как разделить 2 яблока между членами жюри?

- И что решил?

- А … Сделаю компот!...

Поют на мотив песни «Наш сосед»:

В КВН играть мы рады

И в жару и в холода.

Пораженье иль награды –

Это вовсе не беда.

Мы – весёлые ребята,

Наше имечко «ПУПСята»,

Мы не любим лишних слов,

И к игре наш «ПУПС» готов!

Говорят по очереди:

- Клянёмся мы с первых минут КВН

Играть и шутить, чтоб понравилось всем.

- Соперникам нашим –

Все: Огромный привет!

_ Везенья и счастья,

Улыбок букет.

 - А судьи и строги, и уважаемы,

- И также любят шутки, юмор, смех.

- И пусть сильнейший победит сегодня.

- И это будет общий наш успех!



Предварительный просмотр:

Ф.И.__________________________________

Вариант 1.

  1. Любой прямоугольник является:

а) ромбом

б) квадратом

в) параллелограммом

г) нет правильного ответа

  1. Если в четырёхугольнике диагонали перпендикулярны, то этот четырёхугольник - …

а) ромб

б) квадрат

в) прямоугольник

г) нет правильного ответа

3. Ромб – это четырёхугольник, в котором …

а) диагонали точкой пересечения делятся пополам и равны

б) диагонали взаимно перпендикулярны и точкой пересечения делятся пополам

в) противоположные углы равны, а противолежащие стороны параллельны

г) нет правильного ответа

…………………………………………………………………………………         

     Ф.И.__________________________________

Вариант 2.

  1. Любой ромб является:

а) квадратом

б) прямоугольником

в) параллелограммом

г) нет правильного ответа

  1. Если в параллелограмме  диагонали перпендикулярны, то этот параллелограмм - …

а) ромб

б) квадрат

в) прямоугольник

г) нет правильного ответа

3.  Прямоугольник–  это  четырёхугольник, в котором …

а) противолежащие стороны параллельны, а диагонали равны

б) диагонали  точкой пересечения делятся пополам и являются биссектрисами его углов

в) два угла прямые и две стороны равны

г) нет правильного ответа



Предварительный просмотр:

Ф.И.__________________________________

Параллелограмм

Прямоугольник

Ромб

Квадрат

Противолежащие стороны параллельны и равны

Все стороны равны

Противолежащие углы равны, сумма соседних углов равна1800

Все углы прямые

Диагонали пересекаются и точкой пересечения делятся пополам

Диагонали равны

Диагонали взаимно перпендикулярны и являются биссектрисами углов

…………………………………………………………………………………………………………………………..

Ф.И.__________________________________

Параллелограмм

Прямоугольник

Ромб

Квадрат

Противолежащие стороны параллельны и равны

Все стороны равны

Противолежащие углы равны, сумма соседних углов равна1800

Все углы прямые

Диагонали пересекаются и точкой пересечения делятся пополам

Диагонали равны

Диагонали взаимно перпендикулярны и являются биссектрисами углов


Предварительный просмотр:


Предварительный просмотр:

Вариант 1

  1. Найдите сумму углов выпуклого одиннадцатиугольника.
  2. Найдите число сторон выпуклого n-угольника, каждый угол которого равен 1350.
  3. Найдите сторону ВС четырёхугольника ABCD, если его периметр равен 22см, сторона АВ на 2см больше стороны ВС и на 2см меньше каждой из сторон DA и CD.

…………………………………………………………………….

Вариант 2

  1. Найдите сумму углов выпуклого двадцатидвухугольника.
  2. Найдите число сторон выпуклого n-угольника, каждый угол которого равен 1500.
  3. Найдите сторону ВС четырёхугольника ABCD, если его периметр равен 26см, сторона АВ на 2см больше стороны ВС и на 2см меньше каждой из сторон DA и CD.

……………………………………………………………………..

Вариант 3

  1. Найдите сумму углов выпуклого двенадцатиугольника.
  2. Найдите число сторон выпуклого n-угольника, если сумма его углов равна 21600.
  3. В выпуклом пятиугольнике две стороны равны, третья сторона на 3см больше, а четвёртая в 2 раза больше первой стороны, пятая – на 4см меньше четвёртой. Найдите стороны пятиугольника, если известно, что периметр равен 34см.

Вариант 4

  1. Найдите сумму углов выпуклого тринадцатиугольника.
  2. Найдите число сторон выпуклого n-угольника, если сумма его углов равна 25200
  3. В выпуклом шестиугольнике три стороны равны, четвёртая сторона в 2 раза больше первой стороны, пятая – на 3см меньше четвёртой стороны, шестая – на 1см больше второй. Найдите стороны шестиугольника, если известно, что периметр равен 30см.

..........................................................................

Вариант 5

  1. Найдите сумму углов выпуклого семнадцатиугольника.
  2. Найдите число сторон выпуклого n-угольника, каждый угол которого равен 1200.
  3. Найдите сторону ВС четырёхугольника ABCD, если его периметр равен 18см, сторона АВ на 2см меньше стороны ВС и на 2см больше каждой из сторон DA и CD.

…………………………………………………………..

Вариант 6

  1. Найдите сумму углов выпуклого пятнадцатиугольника.
  2. Найдите число сторон выпуклого n-угольника, если сумма его углов равна 14400.
  3. Найдите сторону ВС четырёхугольника ABCD, если его периметр равен 22см, сторона АВ на 2см меньше стороны ВС и на 2см больше каждой из сторон DA и CD.


Предварительный просмотр:


Предварительный просмотр:

Час весёлых и находчивых

5 класс

2 команды

  1. Домашнее задание

Свитки из ватмана. На них большими цветными буквами написаны ребусы.

  1. Выходят команды и обмениваются приветствиями

Команда А

Этот турнир ждали мы.

По нему стосковались умы.

Дружно будем задачи решать-

Мы хотим математику знать.

Как же нам не веселиться?

Не смеяться, не шутить?

Ведь сегодня на турнире

Мы решили победить!

Команда Б

Сегодняшний турнир мы выиграть хотим,

И просто вам победу не дадим.

Придется попотеть и постараться.

За каждое очко мы будем драться.

Смекалку мы проявим и отвагу

И просим разгадать сию бумагу.

И если вдруг не повезёт-

Победа всех когда-нибудь найдёт.

  1. «Бумага», которую просила разгадать вторая команда, - это ребус. (Домашнее задание). Листы вывешиваются на доске. Каждая команда  разгадывает «бумагу» соперников.

Пока команды трудятся над ребусами, ведущий представляет жюри – Совет мудрейших.

        Затем ведущий обращается к болельщикам: «Для решения большинства задач недостаточно одних знаний. Необходима ещё и внимательность. С чего начинается решение задачи? Конечно, с условия. Но условие можно читать по-разному: прочтёшь невнимательно – вот и утеряна главная ниточка. Проверим, умеете ли вы быстро улавливать условие задачи. Ну-ка, кто из вас быстрее решит задачу Корнея Ивановича Чуковского:

«Шёл Кондрат в Ленинград, а навстречу двенадцать ребят,

У каждого по три лукошка. В каждом лукошке – кошка,

У каждой кошки – 12 котят,

У каждого котёнка в зубах по четыре мышонка.

И задумался старый Кондрат:

Сколько мышат и котят ребята несут в Ленинград?»

Ведущий не перебивает учащихся, но затем сообщает, что плохо выслушали условие и поэтому попали в положение Кондрата:

«Глупый, глупый Кондрат!

Он один и шагал в Ленинград,

А ребята с лукошками, с мышами и кошками шли навстречу ему –

В Кострому».

Болельщики могут помочь команде, за которую болеют, если будут активно участвовать в конкурсах для болельщиков. За каждый правильный ответ команде будет причислен балл.

Затем ведущий обращается к ребусам. Представители команд докладывают о том, в чём они увидели ключ к разгадке. За расшифрованный ребус присуждается балл.

  1. Весёлая рыбалка. На столе «озере» разбросаны в разных местах рыбки, вырезанные из бумаги. К каждой рыбке скрепками прикреплена задача. Ребята должны «удочкой» (палочка к ней прикреплена нить с магнитом) поймать рыбку. Для решения задачи «рыбаку» даётся время. За решение один балл.

Задачи:

  1. Спутник Земли делает оборот за 1ч 40 мин, а другой оборот за 100мин. Как это объяснить?
  2. Двое играли в шахматы 2 часа. Сколько времени играл каждый?
  3. Тройка лошадей пробежала 30км. Какое расстояние пробежала каждая лошадь?
  4. Три разных числа сначала сложили, затем их же перемножили. Сумма и произведение оказались равными. Какие это числа?
  5. От куска материи длиной в 200м каждый день отрезали по 20м. Через сколько дней отрезали последний кусок?(9дн.)
  6. По стеблю растения, высота которого 1м ползёт улитка. Днём она поднимается на 4дм, а ночью опускается на 2дм. На какой день улитка будет на вершине? (4-й)
  7. Книга в переплёте стоит 1р20к. Сколько стоит книга, если она на 1р. дороже переплёта?(1р.10к.)
  8. Одно число в 4 раза больше другого, сумма же этих чисел 20. Найдите меньшее число. (4)
  9. Кирпич весит 2 кг и ещё полкирпича. Сколько весит весь кирпич? (4кг)
  10. Арбуз весит 2 кг и ещё арбуза. Какова масса всего арбуза? (6кг)

  1. Конкурс «Рыбалка» очень возбуждает учащихся. Поэтому ведущий предлагает успокоиться и выслушать индусскую притчу, которую любил рассказывать один из создателей Московского художественного театра Константин Сергеевич Станиславский.

Магараджа выбирал себе министра. Он объявил, что возьмёт того, кто пройдёт по стене вокруг города с кувшином, доверху наполненным молоком, и не прольёт ни капли. Многие ходили, но по пути их отвлекали, и они проливали молоко. Но вот пошёл один. Вокруг него кричали, стреляли, его всячески пугали и отвлекали. Он не пролил молоко. «Ты слышал крики, выстрелы? – спросил его потом магараджа. – Ты видел, как тебя пугали?» - «Нет, повелитель, я смотрел на молоко».

        Не слышать и не видеть ничего постороннего - вот до какой степени может быть сосредоточено внимание. Теперь проверим внимание членов команд.

  1. 1) Слушай одновременно нескольких. От каждой команды выходят одному человеку. Начинается игра на проверку внимания.

Двое говорят одновременно два разных слова, а представители команд должны различить, кто какие слова сказал. Затем, трое говорят одновременно три разных слова, следом четверо – четыре слова. Выигрывает тот, кто различил больше слов.

2) Каждой руке = своё дело. Играющим дают лист бумаги и в каждую руку по карандашу. Задание: левой рукой начертить 3 треугольника, а правой 3 окружности. (Можно доска и мел)

3) Шагай – соображай. Участники стоят рядом с ведущим. Все делают первый шаг, и в это время ведущий называет какое-нибудь число, например, 7. При следующих шагах ребята должны назвать числа. Которые делятся на 7: 14, 21. .. На каждый шаг – по числу. Ведущий идёи с ними в ногу. Не давая замедлить шаг. Как только кто-то ошибся, он остаётся на месте до конца движения другого.

По результатам всех 3-х игр жюри определяет команду, победившую в конкурсе «Внимание».

  1. Конкурс Капитанов.

Вопросы капитанам (на размышление полминуты):

  1. В воде оказалась 10-я ступенька пароходной верёвочной лестницы. Начался прилив: вода в час поднимается на 30см. Между ступеньками лестницы 15см. Через сколько часов вода скроет 6-ю ступеньку? (Этого не будет. Пароход поднимется вместе с водой).
  2. Электропоезд идёт на запад со скоростью 60км/ч. В том направлении – с востока на запад – дует ветер, но со скоростью 50км/ч. В какую сторону отклонится дым поезда? (Электропоезд без дыма).
  3. В семье у каждого из шести братьев есть по сестре. Сколько детей в семье? (7)
  4. Петух, стоя на одной ноге, весит 5кг. Сколько он будет весить, если встанет на две ноги? (5кг)

Игра для капитанов «цепочка слов». Ведущий называет одно слово. Первый капитан повторяет это слово и называет своё. Второй капитан повторяет 2 первых слова и добавляет своё и  т.д. Выигрывает тот, кто больше назовёт слов.

  1. Конкурс для болельщиков. Учащимся показывают плакаты. Вопрос: Чьи болельщики быстрее поднимутся по ступенькам?»

Вопросы для болельщиков.

  1. Число разрядов в числе. (3)
  2. Формула площади прямоугольника со сторонами а и b?
  3. Как найти неизвестное делимое?
  4. Может ли при умножении получиться 0?
  5. Что легче 1кг ваты или 1кг железа?
  6.  Бежала тройка лошадей. Каждая лошадь пробежала 5км. Сколько километров проехал ямщик?
  7. К однозначному числу больше нуля приписали такую же цифру. Во сколько раз увеличилось число? (11раз)
  8. Как найти уменьшаемое?
  9. Площадь прямоугольника 36см2. Чему равна сторона квадрата с такой же площадью? (6см)
  10. Что больше 34или 43?

Заканчивается вечер выступлением представителя жюри, который называет победителей и поздравляет их.


Предварительный просмотр:

Предварительный просмотр:


Подписи к слайдам:

Слайд 1

Решение задач на составление уравнений На одной полке было в 3 раза больше книг, чем на другой. Когда с одной полки сняли 8 книг, а на вторую поставили 32 книги, то на полках стало книг поровну. Сколько книг было на каждой полке первоначально? 1 этап. Составление математической модели (уравнения) 1 полка 2 полка Было Стало х 3х 3х-8 Х+32 Зх-8=х+32

Слайд 2

2 этап. Работа с математической моделью (решение уравнения) 3х-8=х+32 3х-х=32+8 2х=40 Х=20 3 этап. Ответ на вопрос задачи б ыло на первой полке Ответ: 60 книг, 20 книг



Предварительный просмотр:

Ф.И.____________________________________

Координатная прямая

Название промежутка

Неравенство

Запись со скобками

отрезок

……………………………………………………………………………………………………………………………………………………………

Ф.И.____________________________________

Координатная прямая

Название промежутка

Неравенство

Запись со скобками

Открытый луч

………………………………………………………………………………………………………………………………………………………

Ф.И.____________________________________

Координатная прямая

Название промежутка

Неравенство

Запись со скобками

интервал

Ф.И.____________________________________

Координатная прямая

Название промежутка

Неравенство

Запись со скобками

луч

……………………………………………………………………………………………………………………………………………………………….

Ф.И.____________________________________

Координатная прямая

Название промежутка

Неравенство

Запись со скобками

отрезок

……………………………………………………………………………………………………………………………………………………………….

Ф.И.____________________________________

Координатная прямая

Название промежутка

Неравенство

Запись со скобками

интервал

 



Предварительный просмотр:

Вариант 1

  1. Вычислите: а) (-8)+(-2); б) -8+2; в) -8-(-2); г) 2-(-8).
  2. Найдите значение выражения: .
  3. Вычислите: 1,8-(-0,7)+(-2,5).

………………………………………………………………..

Вариант 2

  1. Вычислите: а) (-3)+(-7); б) -7+3; в) -7-(-3); г) 3-(-7).
  2. Найдите значение выражения: .
  3. Вычислите: 1,7-(-1,8)+(-3,5).

……………………………………………………………..

Вариант 3

  1. Вычислите: а) (-6)+(-4); б) -6+4; в) -6-(-4); г) 6-(-4).
  2. Найдите значение выражения: .
  3. Вычислите: 1,9-(-0,6)+(-2,5).

Вариант 4

  1. Вычислите: а) (-5)+(-6); б) -5+6; в) -5-(-6); г) 5-(-6).
  2. Найдите значение выражения: .
  3. Вычислите: 1,6-(-0,8)+(-2,4).

……………………………………………………………….

Вариант 5

  1. Вычислите: а) (-1)+(-7); б) -1+7; в) -1-(-7); г) 1-(-7).
  2. Найдите значение выражения: .
  3. Вычислите: 1,5-(-1,7)+(-3,2).

………………………………………………………………….

Вариант 6

  1. Вычислите: а) (-3)+(-4); б) -3+4; в) -3-(-4); г) 3-(-4).
  2. Найдите значение выражения: .
  3. Вычислите: 1,9-(-1,7)+(-3,6).



Предварительный просмотр:

(a+b)2= _______________

(c-d)2=  _______________

n2-m2= _______________

t3-l3=  ________________

(x+y)(x2-xy+y2)= ________

(2a-3b)2=4a2-  _______+____

(4x2+10xy+25y2)(_______)=8x3-125y3

(4c2+d3)2=______+8c2d3__________

(3+2n)(____________)=27____8n3

(5s+3t)(_______)=9t2-25s2

(a-d)2= _______________

(x+r)2=  _______________

y2-t2= _______________

t3+l3=  ________________

(x-z)(x2+xz+z2)= ________

(5a-3b)2=_____-  _______+9b2

(9x2-15xy+25y2)(_______)=27x3+125y3

(5c2-d3)2=______-10c2d3__________

(1-3n)(____________)=1____27n3

(s+3d)(_______)=9d2-s2

(n+m)2= _______________

(2-d)2=  _______________

n2-32= _______________

x3-y3=  ________________

(x+t)(x2-xt+t2)= ________

(3a-2b)2=9a2-  _______+____

(4x2+6xy+9y2)(_______)=8x3-27y3

(c2+2d3)2=______+4c2d3__________

(m+2n)(____________)=m3____8n3

(5x+t)(_______)=t2-25x2

(c-z)2= _______________

(d+3)2=  _______________

b2-a2= _______________

t3-23=  ________________

(z+y)(z2-zy+y2)= ________

(2a+5b)2=4a2+  _______+____

(4x2+2xy+y2)(_______)=8x3-y3

(4c3+d2)2=______+8c3d2__________

(3-5n)(____________)=27____125n3

(2s+3t)(_______)=9t2-4s2



Предварительный просмотр:

Фамилия, имя __________________________

Фамилия, имя __________________________

Фамилия, имя __________________________

Фамилия, имя __________________________

Фамилия, имя __________________________

Фамилия, имя __________________________

Фамилия, имя __________________________

Фамилия, имя __________________________

Фамилия, имя __________________________


Предварительный просмотр:


Подписи к слайдам:

Слайд 1

Статистика – дизайн информации 9 класс

Слайд 2

Статистика – дизайн информации У 50 работников городского предприятия попросили оценить время, которое они в среднем тратим на проезд от дома до работы. Получились следующие данные в минутах ( с точностью до 10 минут) 20 100 20 30 40 50 30 80 90 40 30 50 20 50 30 30 50 60 60 50 30 40 60 50 100 60 90 10 20 50 90 80 20 40 50 10 50 40 30 40 60 120 30 40 60 20 60 10 50 60 Близко: 10; 20 или 30 минут Недалеко: 40; 50 или 60 минут Далеко: более часа

Слайд 3

Варианта Сумма близко недалеко далеко Кратность 17 25 8 50 Частота, % 34 50 16 100 Опр.2. Если среди всех данных конкретного измерения одна из вариант встретилась ровно k раз, то число k называют кратностью этой варианты измерения. Опр. 1. Варианта измерения – один из результатов этого измерения Опр. 3. Частота

Слайд 5

Числовые характеристики данных измерений Размах измерения – разность между максимальной и минимальной вариантами 120 – 10=110 минут 2. Мода измерения – та варианта. Которая в измерении встретилась чаще других 50 минут (10 работников) 3 . Среднее значение – среднее арифметическое: Просуммировать все данные измерения; Полученную сумму разделить на количество данных или Каждую варианту умножить на её частоту ; Сложить полученные произведения 48 минут

Слайд 6

На вступительном экзамене по математике можно получить от 0 до 10 баллов. Сорок абитуриентов получили такие оценки: 6 7 7 8 9 2 10 6 5 6 7 3 7 9 9 2 3 2 6 6 6 7 8 8 2 6 7 9 7 5 9 8 2 6 6 3 7 7 6 6 а) Составить общий ряд данных; упорядочить и сгруппировать полученные оценки б) составить таблицы распределения данных и распределения частот в) Построить графики распределения данных и распределения частот г) Найти размах, моду и среднее измерения


Предварительный просмотр:


Подписи к слайдам:

Слайд 1

Комбинаторика 9 класс

Слайд 2

Комбинаторика Задачи о подсчёте числа возможных комбинаций называют комбинаторными. Задача 1. О числе выборок из нескольких множеств. Даны два множества предметов, в первом m элементов, второе множество содержит n элементов. Сколько можно составить пар элементов, выбирая по одному из каждого множества?

Слайд 3

Правило умножения Для того чтобы найти число всех возможных исходов независимого проведения двух испытаний А и В, следует перемножить число всех исходов испытания А и число всех исходов испытания В.

Слайд 4

Задача 1. На завтрак Вова может выбрать плюшку, бутерброд, пряник или кекс, а запить их он может кофе. Соком или кефиром. Сколько вариантов завтрака есть у Вовы? Задача 2. В коридоре три лампочки. Сколько имеется различных вариантов освещения, включая случай, когда все лампочки не горят?

Слайд 5

Задача 2. Перестановки. В семье шесть человек, а за столом в кухне шесть стульев. Было решено каждый вечер перед ужином рассаживаться на эти шесть стульев по-новому. Сколько дней члены семьи смогут делать это без повторений? Опр. Произведение подряд идущих первых n натуральных чисел обозначают n!

Слайд 6

n различных элементов можно расставить по одному на n различных мест ровно n! с пособами. -число всех перестановок множества из n элементов

Слайд 7

Задача 1. Сколькими способами четыре вора могут по одному разбегаться на все четыре стороны? Задача 2. В 11 классе в среду семь уроков: алгебра, геометрия, русский язык, литература., биология, английский язык, физкультура. Сколько вариантов расписания можно составить на среду?

Слайд 8

Простейшие вероятностные задачи Задача 1. Из цифр 1, 5, 9 случайным образом составляют трёхзначное число без повторяющихся цифр. Какова вероятность того, что получится число: а) больше 500 б)Квадратный корень из которого не больше 24 в) кратное трём г) кратное девяти? Классическая вероятностная схема Для нахождения случайного события А при проведении некоторого испытания следует: Найти число N всех возможных исходов данного испытания; Найти количество N(A) всех тех исходов испытания. В которых наступает событие А; Найти частное ; оно и будет равно вероятности события А.

Слайд 9

Задача 2. Монету подбрасывают три раза. Какова вероятность того, что: а) Все три раза выпадет «решка» б) «Решка» выпадет в 2 раза чаще, чем «Орёл» в) «Орёл» выпадет в 3 раза чаще, чем «Решка» г) При первом и третьем подбрасывании результаты будут различны? Задача. 17 точек из 50 покрашены в синий цвет, а 13 точек из оставшихся покрашены в оранжевый цвет. Какова вероятность того, что случайным образом выбранная точка окажется: а) синей; б) не оранжевой; в) окрашенной; г) неокрашенной

Слайд 10

Опр. Событие В называют противоположным событию А, если Событие В происходит тогда и только тогда, когда не происходит событие А. Обозначение: Опр. События А и В называют несовместными , если они не могут происходить одновременно. Если события А и В несовместны, то вероятность того, что наступит или А, или В, равна P(A)+P(B)

Слайд 11

Задача. Какова вероятность того, что при трёх последовательных бросаниях игрального кубика хотя бы один раз выпадет 6? А – событие: выпадение хотя бы одной шестёрки -противоположное событие: шестёрка вообще не выпадет ни в первый, ни во второй, ни в третий раз

Слайд 12

Задача 1. Какова вероятность того, что случайным образом выбранное двузначное число будет делится на 13? Задача 2. Какова вероятность того, что у случайным образом выбранного двузначного числа сумма цифр будет больше 15? Задача 3. Монету подбрасывают три раза. Какова вероятность того, что: а) в последний раз выпадет «Решка» б) ни разу не выпадет «Орёл» в) число выпадений «орла» в два раза больше числа выпадений «Решки» г) при первых двух подбрасываниях результаты будут одинаковы?

Слайд 13

Задача 4. Случайным образом выбрали двузначное число. Найдите вероятность того, что оно: а) оканчивается нулём б)Состоит из одинаковых цифр в) больше 27 и меньше 46 Г) не является кубом другого целого числа Задача 5. Найдите вероятность того, что при одном бросании игрального кубика выпадет: а) четвёрка б) чётное число очков в) число очков больше четырёх г) число очков, не кратно трём


Предварительный просмотр:


Подписи к слайдам:

Слайд 1

Экспериментальные данные и вероятность событий 9 класс

Слайд 2

Экспериментальные данные и вероятности событий На практических занятиях по обработке данных каждый из 20 школьников подбросил рублёвую монету 50 раз, подсчитал количество k выпадений «орла» и записал это количество в процентах от общего числа бросаний. Полученные данные были собраны в таблицу. № 1 2 3 4 5 6 7 8 9 10 11 12 13 14 15 16 17 18 19 20 k 24 27 23 26 28 25 24 25 26 22 23 23 22 26 27 24 23 29 30 21 % 48 54 46 52 56 50 48 50 52 44 46 46 44 52 54 48 46 58 60 42 № 1-2 3-4 5-6 7-8 9-10 11-12 13-14 15-16 17-18 19-20 k 51 49 53 49 48 46 48 51 52 51

Слайд 3

№ 1-4 5-8 9-12 13-16 17-20 k 100 102 94 99 103 % 50 51 47 49,5 51,5 100+102+94+99+103=498 49,8%

Слайд 4

Каждый из десяти игроков 50 раз подряд повторил одновременные бросания трёх игральных костей различного цвета и подсчитал количество k тех бросаний, в которых не выпала шестёрка. Получились такие результаты: № 1 2 3 4 5 6 7 8 9 10 k 28 32 24 30 31 25 29 29 27 28 а) Составьте таблицу частот (в %) невыпадения шестёрки для каждого игрока б) Составьте таблицу частот невыпадения шестёрки для результатов игроков 1-2, 3-4, …., 9-10 в) Какова частота невыпадения шестёрки для всех 500 проведённых бросаний? г) Найти вероятность невыпадения шестёрки при бросании трёх игральных костей.



Предварительный просмотр:

Широкая масленица

совместное мероприятие 1-го и 5-го классов

Цель: Познакомить с традициями праздника «Масленица».

Задачи:   1.Познакомить с праздником “Масленица”, дать представление учащимся о народных традициях, обрядах, верованиях, выявить связь жизни русского народа “со словом поющим” (масленичные песни);

2. Развивать эмоционально-волевую сферу, творческие и умственные способностей,вызвать положительные эмоции и праздничное настроение;

3. Воспитывать интерес к истории своего народа, желание поддерживать древние традиции, любовь к русской культуре, народной музыке, играм

Ведущий 1: Внимание, внимание!

 Веселое гуляние!

 Собирайся скорее народ!

 Вас что-то интересное ждет.

 Масленица, масленица!

 Широкая масленица!

Ведущий 2: Масленица!

 Будь здоровенька!

 Тебя встретим хорошенько!

 С блинами, с каравайцами, с варениками!

Ведущий 1:  Масляна неделя в гости прилетела.

                      На пенечек села, оладышек съела.

                      Другим закусила.

                      Домой потрусила.

Ведущий 2: Приглашаем на масленицу! И тихони и проказники!

Ведущий 1: Будут блин да калач.

                     Будут игры и смех.

 Будет пляска для всех.


Выходит фольклорная группа в русских народных костюмах, и исполняют русскую народную песню  «Блины».


Ведущий 2: 
А вы знаете, что Масленица предшествует Великому посту? Она длится всего неделю, и эту неделю можно плясать веселиться, кататься с горок, устраивать состязания и есть блины. А еще у каждого дня масленичной недели есть свое название. Давайте с вами узнаем, как называются каждый из дней масленицы.


Выходят 7 учеников.
1 ученик:
Понедельник первый вечно
и называется он «встреча»,
В этот день Весну встречают,
И блины печь начинают!
2 ученик:
Вторник – «заигрыш» зовется,
Ходят все друг к другу в гости,
С горок снежных все катаются,

И блинами наедаются.

3 ученик:
Среда – лакомка называется,
Вот тут все точно объедаются!
Живот себе все набивают,
И друг друга угощают!


4 ученик:
Ну, а что же с четвергом,
Четверг зовется «перелом»
С этого дня все меняется,
И разгулье начинается!

5 ученик:
«Тещины вечорки» пятница называется,
В этот день зятья к тещам в гости собираются!
И тещи для зятьев пекли блины,
И устраивали настоящий зятю пир!

6 ученик:
А суббота – день семейный,
Зовется «золовкины посиделки»
В этот день невеста родных в гости принимает,
А золовкам всем подарки дарит!

7 ученик:
У воскресенья званий много,
Но скажем лишь одно мы,
Прощенное зовется воскресенье,
И просят все друг у друга прощения!

Выкатывают чучело масленицы на санях. Девочки окружают его и приговаривают.

Девочки: Дорогая наша Масленица!

                Авдотья Ивановна!

                Дуня белая! Дуня румяная!

                Коса длинная, триаршинная!

                Лента алая, двуполтинная!

                Платок беленький, новомодненький!

                Брови черные, наведенные!

                Шуба синяя, заплатки красные!

                Лапти частые, головастые!

                Портянки белые, набеленные!

                Наша Масленица годовая.

                Она гостьюшка дорогая.

                Она пешею не ходит.

               Она на кониках разъезжает.

               Чтобы коники были вороные,

               Чтобы слуги были молодые.

               Располагайтесь, Авдотья Ивановна,

                А мы гулять будем.

                Масленица, Масленица!

                Широкая Масленица!

                Будем весело гулять.

                Масленицу прославлять.

Ведущий 1: А ну-ка, народ! Вставай в хоровод!

Все встают, водят хоровод вокруг Масленицы, (Песня «Кап-кап-кап, весна стучится»).


Ведущий 2:
Давайте теперь позовем к нам в гости весну! А чтобы она нас услышала, давайте споем весеннюю песенку.

Первоклассники поют песню о весне.


Выходит
Весна. 
Слышу, слышу, что зовете,
Вижу, вижу, что меня вы ждете.
Вот пришла я к вам друзья,
Наконец, я вас нашла!

Ребята, а теперь давайте поиграем.

                                     Игра – Горячий блин
Из альбомного листа вырезается круг. Это будет блин. Все участники встают в круг и делают руки ладонями вверх. Включается веселая музыка. Первому участнику кладется на ладони «блин», и участник должен как можно скорее передать этот блин на ладони другому участнику. В тот момент, когда музыка оборвется, участник, у кого в этот момент будет в руках «блин», выбывает. Так же выбывает участник, который уронит блин при передаче. Последний оставшийся получает приз.

Весна: Пришло времечко народ собирать

Мудреные загадки отгадать.

Кто остроту ума проявит,

Тот этот приз себе оставит.

Загадки:

  1. Праздник этот знаем мы –
    Время проводов зимы.
    Люди в эти дни должны
    Веселиться, печь блины.
    (Масленица.)

  1. Жёлтый, круглый, ароматный

И на вкус такой приятный

И с вареньем, и с медком,

 Со сгущенным молочком!  (Блины.)

  1. Дует тёплый южный ветер,
    Солнышко всё ярче светит.
    Снег худеет, мякнет, тает,
    Грач горластый прилетает.
    Что за месяц? Кто узнает?  (Март)

  1. Хоть сама – и снег, и лед,
    А уходит – слезы льет.  
    (Зима.)

  1. На горе-горушки

Стоит старушка

Руками машет,

Хлеба просит.

Дашь ей хлеба-

Назад отдает. (Мельница)

  1. Что за зверь:

В зиму ест,

А летом спит,

Тело теплое,

А крови нет

Сесть, на него сядешь,

А с места не свезет. (Печь)

  1. По - блошьи прыгает, а по-человечьи плавает. (Лягушка)

  1. Не король, а в короне,

     Не гусар, а при шпорах. (Петух)

Игра «Перетягивание каната»

Играющие делятся на 2 команды. Посередине площадки кладется канат. Середина каната отмечается ленточкой, под серединой каната проводится поперек черта. В 2-3 м. от нее проводится параллельно еще две черты. Играющие берутся за канат и поднимают его с пола. По сигналу играющие стараются перетянуть канат в свою сторону. Канат считается перетянутым, если его середина окажется в 2-3 м. от средней линии на стороне одной из команд. Перетягивать канат можно только после сигнала. Игру проводят несколько раз. Выигрывает команда, которая перетянет канат на свою сторону большее число раз.

 В программу Олимпийских игр 1900—1920 перетягивание каната входило как дисциплина лёгкой атлетики (на Олимпийских играх 1912 года, согласно официальному отчёту, фигурировало как самостоятельный вид спорта).

Игра «Прыжки в мешке»

Игра «Петушиный бой»

Игроки делятся на пары и встают друг напротив друга на расстоянии 3-5 шагов. Пары изображают дерущихся петухов: прыгая на одной ноге, они стараются толкнуть друг друга плечем. Тот, кто потерял равновесие и встал на пол двумя ногами, выходит из игры. Перед началом игры играющие договариваются, как они будут держать руки: на поясе, за спиной, крест-накрест, перед грудью или руками держать колено согнутой ноги.
Можно соревноваться командами: победит та из них, в которой осталось больше игроков, стоящих на одной ноге.


Весна: Почему блины стали символом масленицу, что они символизируют? (блин – символ солнца, а солнце – это начало начал у древних русичей, ему первому поклонялись).


Весна: А теперь давайте вместе с вами
Угостимся мы блинами,
Но сначала нашу Масленицу мы сожжем,
Чтоб зима и все наши беды прочь ушли с огнем!

Масленицу,  выносят на улицу и сжигают. Так же можно всем желающим предложить написать на листочках все плохое, например: болезни, плохое настроение, плохие оценки и т.д. и бросить эти листочки в костер.  


По теме: методические разработки, презентации и конспекты

Тема родительского собрания:"Организация учебной деятельности школьников при подготовке к ЕГЭ по математике"

Рассказывается о том, как помогут электронные ресурсы при подготовке к ЕГЭ...

Справка о проведении недели математики "И прекрасна и сильна- Математика страна"

Цели и задачи:1.     Формирование устойчивого интереса к математике и её приложениям;2.      Расширение и углубление знаний учащихся по программному матери...

Развитие математического мышления через применение информационных технологий на уроках математики

Развитие математического мышления через применение информационных коммуникационных технологий на уроках математики...

Рекомендации по проведению родительских собраний, сценарии

Рекомендации по проведению родительских собраний, сценарии...

Родительское собрание на тему: «Подготовка учащихся 9 класса к ОГЭ по математике»

В девятом классе ученики впервые сдают свои первые экзамены. Для них это важный этап в жизни. Очень важно подготовить ребят к этому ответственному этапу. Часть учеников теряется при виде тестовых...

Протокол №1 родительского собрания детского объединения «Технология обработки материалов»

ПРОТОКОЛ №1родительского собрания детского объединения «Технология обработки материалов»от 18 сентября 2021 г.«Знакомство с разделами программы. Правила поведения.  Выбор родите...